You are on page 1of 97

INSURANCE LAW of her fractured bones. Instead, the victim was brought to the U.S.T.

es. Instead, the victim was brought to the U.S.T. Hospital where she expired at 9:00
o'clock that same morning. Death was caused by "traumatic shock" as a result of the severe injuries she
1. Travellers Insurance & Surety Corp . v. CA (272 SCRA 536, GRN 82036, 22 May 1997) sustained . . .

HERMOSISIMA, JR., J.: . . . The evidence shows that at the moment the victim was bumped by the vehicle, the latter was running
fast, so much so that because of the strong impact the old woman was thrown away and she fell on the
The petition herein seeks the review and reversal of the decision of respondent Court of
1
pavement. . . . In truth, in that related criminal case against defendant Dumlao . . . the trial court found as
Appeals 2affirming in toto the judgment of the Regional Trial Court in an action for damages filed by
3 4 5
a fact that therein accused "was driving the subject taxicab in a careless, reckless and imprudent manner
private respondent Vicente Mendoza, Jr. as heir of his mother who was killed in a vehicular accident. and at a speed greater than what was reasonable and proper without taking the necessary precaution to
avoid accident to persons . . . considering the condition of the traffic at the place at the time
Before the trial court, the complainant lumped the erring taxicab driver, the owner of the taxicab, and the aforementioned" . . . Moreover, the driver fled from the scene of the accident and without rendering
alleged insurer of the vehicle which featured in the vehicular accident into one complaint. The erring assistance to the victim. . . .
taxicab was allegedly covered by a third-party liability insurance policy issued by petitioner Travellers
Insurance & Surety Corporation. . . . Three (3) witnesses who were at the scene at the time identified the taxi involved, though not
necessarily the driver thereof. Marvilla saw a lone taxi speeding away just after the bumping which, when
The evidence presented before the trial court established the following facts: it passed by him, said witness noticed to be a Lady Love Taxi with Plate No. 438, painted maroon, with
baggage bar attached on the baggage compartment and with an antenae [sic] attached at the right rear
At about 5:30 o'clock in the morning of July 20, 1980, a 78-year old woman by the name of Feliza Vineza
side. The same descriptions were revealed by Ernesto Lopez, who further described the taxi to have . . .
de Mendoza was on her way to hear mass at the Tayuman Cathedral. While walking along Tayuman
reflectorized decorations on the edges of the glass at the back . . . A third witness in the person of
corner Gregorio Perfecto Streets, she was bumped by a taxi that was running fast. Several persons
Eulogio Tabalno . . . made similar descriptions although, because of the fast speed of the taxi, he was
witnessed the accident, among whom were Rolando Marvilla, Ernesto Lopez and Eulogio Tabalno. After
only able to detect the last digit of the plate number which is "8". . . . [T]he police proceeded to the
the bumping, the old woman was seen sprawled on the pavement. Right away, the good Samaritan that
garage of Lady Love Taxi and then and there they took possession of such a taxi and later impounded it
he was, Mavilla ran towards the old woman and held her on his lap to inquire from her what had
in the impounding area of the agency concerned. . . . [T]he eyewitnesses . . . were unanimous in pointing
happened, but obviously she was already in shock and could not talk. At this moment, a private jeep
to that Lady Love Taxi with Plate No. 438, obviously the vehicle involved herein.
stopped. With the driver of that vehicle, the two helped board the old woman on the jeep and brought
her to the Mary Johnston Hospital in Tondo. . . . During the investigation, defendant Armando Abellon, the registered owner of Lady Love Taxi
bearing No. 438-HA Pilipinas Taxi 1980, certified to the fact "that the vehicle was driven last July 20, 1980
. . . Ernesto Lopez, a driver of a passenger jeepney plying along Tayuman Street from Pritil, Tondo, to
by one Rodrigo Dumlao. . ." . . . It was on the basis of this affidavit of the registered owner that caused
Rizal Avenue and vice-versa, also witnessed the incident. It was on his return trip from Rizal Avenue when
the police to apprehend Rodrigo Dumlao, and consequently to have him prosecuted and eventually
Lopez saw the plaintiff and his brother who were crying near the scene of the accident. Upon learning
convicted of the offense . . . . . . . [S]aid Dumlao absconded in that criminal case, specially at the time of
that the two were the sons of the old woman, Lopez told them what had happened. The Mendoza
the promulgation of the judgment therein so much so that he is now a fugitive from justice. 6
brothers were then able to trace their mother at the Mary Johnston Hospital where they were advised by
the attending physician that they should bring the patient to the National Orthopedic Hospital because

Page 1 of 97
Private respondent filed a complaint for damages against Armando Abellon as the owner of the Lady Petitioner mainly contends that it did not issue an insurance policy as compulsory insurer of the Lady
Love Taxi and Rodrigo Dumlao as the driver of the Lady Love taxicab that bumped private respondent's Love Taxi and that, assuming arguendo that it had indeed covered said taxicab for third-party liability
mother. Subsequently, private respondent amended his complaint to include petitioner as the insurance, private respondent failed to file a written notice of claim with petitioner as required by Section
compulsory insurer of the said taxicab under Certificate of Cover No. 1447785-3. 384 of P.D. No. 612, otherwise known as the Insurance Code.

After trial, the trial court rendered judgment in favor of private respondent, the dispositive portion of We find the petition to be meritorious.
which reads:
I
WHEREFORE, judgment is hereby rendered in favor of the plaintiff, or more particularly the "Heirs of the
late Feliza Vineza de Mendoza," and against defendants Rodrigo Dumlao, Armando Abellon and When private respondent filed his amended complaint to implead petitioner as party defendant and
Travellers Insurance and Surety Corporation, by ordering the latter to pay, jointly and severally, the therein alleged that petitioner was the third-party liability insurer of the Lady Love taxicab that fatally hit
former the following amounts: private respondent's mother, private respondent did not attach a copy of the insurance contract to the
amended complaint. Private respondent does not deny this omission.
(a) The sum of P2,924.70, as actual and compensatory damages, with interest thereon at the rate of 12%
per annum from October 17, 1980, when the complaint was filed, until the said amount is fully paid; It is significant to point out at this juncture that the right of a third person to sue the insurer depends on
whether the contract of insurance is intended to benefit third persons also or only the insured.
(b) P30,000.00 as death indemnity;
[A] policy . . . whereby the insurer agreed to indemnify the insured "against all sums . . . which the Insured
(c) P25,000.00 as moral damages; shall become legally liable to pay in respect of: a. death of or bodily injury to any person . . . is one for
indemnity against liability; from the fact then that the insured is liable to the third person, such third
(d) P10,000.00 as by way of corrective or exemplary damages; and person is entitled to sue the insurer.

(e) Another P10,000.00 by way of attorney's fees and other litigation expenses. The right of the person injured to sue the insurer of the party at fault (insured), depends on whether the
contract of insurance is intended to benefit third persons also or on the insured And the test applied has
Defendants are further ordered to pay, jointly and severally, the costs of this suit. been this: Where the contract provides for indemnity against liability to third persons, then third persons
to whom the insured is liable can sue the insurer. Where the contract is for indemnity against actual loss
SO ORDERED. 7
or payment, then third persons cannot proceed against the insurer, the contract being solely to
reimburse the insured for liability actually discharged by him thru payment to third persons, said third
Petitioner appealed from the aforecited decision to the respondent Court of Appeals. The decision of the
persons' recourse being thus limited to the insured alone. 10
trial court was affirmed by respondent appellate court. Petitioner's Motion for Reconsideration 8 of
September 22, 1987 was denied in a Resolution 9 dated February 9, 1988.
Since private respondent failed to attach a copy of the insurance contract to his complaint, the trial court
could not have been able to apprise itself of the real nature and pecuniary limits of petitioner's liability.
Hence this petition.
More importantly, the trial court could not have possibly ascertained the right of private respondent as

Page 2 of 97
third person to sue petitioner as insurer of the Lady Love taxicab because the trial court never saw nor In solidary obligation, the creditor may enforce the entire obligation against one of the solidary debtors.
read the insurance contract and learned of its terms and conditions. On the other hand, insurance is defined as "a contract whereby one undertakes for a consideration to
indemnify another against loss, damage or liability arising from an unknown or contingent event."
Petitioner, understandably, did not volunteer to present any insurance contract covering the Lady Love
taxicab that fatally hit private respondent's mother, considering that petitioner precisely presented the In the case at bar, the trial court held petitioner together with respondents Sio Choy and San Leon Rice
defense of lack of insurance coverage before the trial court. Neither did the trial court issue a Mills Inc. solidarily liable to respondent Vallejos for a total amount of P29,103.00, with the qualification
subpoena duces tecum to have the insurance contract produced before it under pain of contempt. that petitioner's liability is only up to P20,000.00. In the context of a solidary obligation, petitioner may be
compelled by respondent Vallejos to pay the entire obligation of P29,103.00, notwithstanding the
We thus find hardly a basis in the records for the trial court to have validly found petitioner liable jointly qualification made by the trial court. But, how can petitioner be obliged to pay the entire obligation
and severally with the owner and the driver of the Lady Love taxicab, for damages accruing to private when the amount stated in its insurance policy with respondent Sio Choy for indemnity against third-
respondent. party liability is only P20,000.00? Moreover, the qualification made in the decision of the trial court to the
effect that petitioner is sentenced to pay up to P20,000.00 only when the obligation to pay P29,103.00 is
Apparently, the trial court did not distinguish between the private respondent's cause of action against made solidary is an evident breach of the concept of a solidary obligation. 12
the owner and the driver of the Lady Love taxicab and his cause of action against petitioner. The former
is based on torts and quasi-delicts while the latter is based on contract. Confusing these two sources of The above principles take on more significance in the light of the counter-allegation of petitioner that,
obligations as they arise from the same act of the taxicab fatally hitting private respondent's mother, and assuming arguendo that it is the insurer of the Lady Love taxicab in question, its liability is limited to only
in the face of overwhelming evidence of the reckless imprudence of the driver of the Lady Love taxicab, P50,000.00, this being its standard amount of coverage in vehicle insurance policies. It bears repeating
the trial court brushed aside its ignorance of the terms and conditions of the insurance contract and that no copy of the insurance contract was ever proffered before the trial court by the private
forthwith found all three - the driver of the taxicab, the owner of the taxicab, and the alleged insurer of respondent, notwithstanding knowledge of the fact that the latter's complaint against petitioner is one
the taxicab - jointly and severally liable for actual, moral and exemplary damages as well as attorney's under a written contract. Thus, the trial court proceeded to hold petitioner liable for an award of
fees and litigation expenses. This is clearly a misapplication of the law by the trial court, and respondent damages exceeding its limited liability of P50,000.00. This only shows beyond doubt that the trial court
appellate court grievously erred in not having reversed the trial court on this ground. was under the erroneous presumption that petitioner could be found liable absent proof of the contract
and based merely on the proof of reckless imprudence on the part of the driver of the Lady Love taxicab
While it is true that where the insurance contract provides for indemnity against liability to third persons, that fatally hit private respondent's mother.
such third persons can directly sue the insurer, however, the direct liability of the insurer under indemnity
contracts against third-party liability does not mean that the insurer can be held solidarily liable with the II
insured and/or the other parties found at fault. The liability of the insurer is based on contract; that of the
insured is based on tort. 11 Petitioner did not tire in arguing before the trial court and the respondent appellate court that, assuming
arguendo that it had issued the insurance contract over the Lady Love taxicab, private respondent's
Applying this principle underlying solidary obligation and insurance contracts, we ruled in one case that: cause of action against petitioner did not successfully accrue because he failed to file with petitioner a
written notice of claim within six (6) months from the date of the accident as required by Section 384 of
the Insurance Code.

Page 3 of 97
At the time of the vehicular incident which resulted in the death of private respondent's mother, during It is significant to note that the aforecited Section 384 was amended by B.P. Blg. 874 to categorically
which time the Insurance Code had not yet been amended by Batas Pambansa (B.P.) Blg. 874, Section provide that "action or suit for recovery of damage due to loss or injury must be brought in proper cases,
384 provided as follows: with the Commissioner or the Courts within one year from denial of the claim, otherwise the claimant's
right of action shall prescribe" [emphasis ours]. 15
Any person having any claim upon the policy issued pursuant to this chapter shall, without any
unnecessary delay, present to the insurance company concerned a written notice of claim setting forth We have certainly ruled with consistency that the prescriptive period to bring suit in court under an
the amount of his loss, and/or the nature, extent and duration of the injuries sustained as certified by a insurance policy, begins to run from the date of the insurer's rejection of the claim filed by the insured,
duly licensed physician. Notice of claim must be filed within six months from date of the accident, the beneficiary or any person claiming under an insurance contract. This ruling is premised upon the
otherwise, the claim shall be deemed waived. Action or suit for recovery of damage due to loss or injury compliance by the persons suing under an insurance contract, with the indispensable requirement of
must be brought in proper cases, with the Commission or the Courts within one year from date of having filed the written claim mandated by Section 384 of the insurance Code before and after its
accident, otherwise the claimant's right of action shall prescribe [emphasis supplied]. amendment. Absent such written claim filed by the person suing under an insurance contract, no cause
of action accrues under such insurance contract, considering that it is the rejection of that claim that
In the landmark case of Summit Guaranty and Insurance Co., Inc. v. De Guzman, 13 we ruled that the one triggers the running of the one-year prescriptive period to bring suit in court, and there can be no
year prescription period to bring suit in court against the insurer should be counted from the time that opportunity for the insurer to even reject a claim if none has been filed in the first place, as in the instant
the insurer rejects the written claim filed therewith by the insured, the beneficiary or the third person case.
interested under the insurance policy. We explained:
The one-year period should instead be counted from the date of rejection by the insurer as this is the
It is very obvious that petitioner company is trying to use Section 384 of the Insurance Code as a cloak to time when the cause of action accrues. . . .
hide itself from its liabilities. The facts of these cases evidently reflect the deliberate efforts of petitioner
company to prevent the filing of a formal action against it. Bearing in mind that if it succeeds in doing so In Eagle Star Insurance Co., Ltd., et al. v. Chia Yu, this Court ruled:
until one year lapses from the date of the accident it could set up the defense of prescription, petitioner
company made private respondents believe that their claims would be settled in order that the latter will The plaintiff's cause of action did not accrue until his claim was finally rejected by the insurance company.
not find it necessary to immediately bring suit. In violation of its duties to adopt and implement This is because, before such final rejection, there was no real necessity for bringing suit.
reasonable standards for the prompt investigation of claims and to effectuate prompt, fair and equitable
settlement of claims, and with manifest bad faith, petitioner company devised means and ways of stalling The philosophy of the above pronouncement was pointed out in the case of ACCFA vs. Alpha Insurance

the settlement proceeding . . . [N]o steps were taken to process the claim and no rejection of said claim and Surety Co., viz:
was ever made even if private respondent had already complied with all the requirements. . . .
Since a cause of action requires, as essential elements, not only a legal right of the plaintiff and a

This Court has made the observation that some insurance companies have been inventing excuses to correlative obligation of the defendant but also an act or omission of the defendant in violation of said

avoid their just obligations and it is only the State that can give the protection which the insuring public legal right, the cause of action does not accrue until the party obligated refuses, expressly or impliedly, to

needs from possible abuses of the insurers. 14 comply with its duty. 16

Page 4 of 97
When petitioner asseverates, thus, that no written claim was filed by private respondent and rejected by (60) days from the occurrence thereof, in the Death of the Insured, the Company shall pay the
petitioner, and private respondent does not dispute such asseveration through a denial in his pleadings, amount set opposite such injury:
we are constrained to rule that respondent appellate court committed reversible error in finding
petitioner liable under an insurance contract the existence of which had not at all been proven in court. Section 1. Injury sustained other than those specified below unless
Even if there were such a contract, private respondent's cause of action can not prevail because he failed excepted hereinafter. . . . . . . . P1,000.00
to file the written claim mandated by Section 384 of the Insurance Code. He is deemed, under this legal
provision, to have waived his rights as against petitioner-insurer. Section 2. Injury sustained by the wrecking or disablement of a railroad
passenger car or street railway car in or on which the Insured is
WHEREFORE, the instant petition is HEREBY GRANTED. The decision of the Court of Appeals in CA-G.R. travelling as a farepaying passenger. . . . . . . . P1,500.00
CV No. 09416 and the decision of the Regional Trial Court in Civil Case No. 135486 are REVERSED and
Section 3. Injury sustained by the burning of a church, theatre, public
SET ASIDE insofar as Travelers Insurance & Surety Corporation was found jointly and severally liable to
library or municipal administration building while the Insured is therein
pay actual, moral and exemplary damages, death indemnity, attorney's fees and litigation expenses in
at the commencement of the fire. . . . . . . . P2,000.00
Civil Case No. 135486. The complaint against Travellers Insurance & Surety Corporation in said case is
hereby ordered dismissed. Section 4. Injury sustained by the wrecking or disablement of a regular
passenger elevator car in which the Insured is being conveyed as a
No pronouncement as to costs. passenger (Elevator in mines excluded) P2,500.00

SO ORDERED Section 5. Injury sustained by a stroke of lightning or by a cyclone. . . . .


... P3,000.00
2. Del Rosario V. Equitable Insurance & Casualty Co. (8 SCRA 343, G.R. No. L-16215, June
29, 1963) xxx xxx xxx

PAREDES, J.: Part VI. Exceptions

On February 7, 1957, the defendant Equitable Insurance and Casualty Co., Inc., issued Personal Accident This policy shall not cover disappearance of the Insured nor shall it cover Death, Disability,
Policy No. 7136 on the life of Francisco del Rosario, alias Paquito Bolero, son of herein plaintiff-appellee, Hospital fees, or Loss of Time, caused to the insured:
binding itself to pay the sum of P1,000.00 to P3,000.00, as indemnity for the death of the insured. The
pertinent provisions of the Policy, recite: . . . (h) By drowning except as a consequence of the wrecking or disablement in the Philippine
waters of a passenger steam or motor vessel in which the Insured is travelling as a farepaying
Part I. Indemnity For Death passenger; . . . .

If the insured sustains any bodily injury which is effected solely through violent, external, visible A rider to the Policy contained the following:
and accidental means, and which shall result, independently of all other causes and within sixty

Page 5 of 97
IV. DROWNING The amount payable under the policy, I believe should be P1,500.00 under the provision of
Section 2, part 1 of the policy, based on the rule of pari materia as the death of the insured
It is hereby declared and agreed that exemption clause Letter (h) embodied in PART VI of the policy is occurred under the circumstances similar to that provided under the aforecited section.
hereby waived by the company, and to form a part of the provision covered by the policy.
Defendant company, upon receipt of the letter, referred the matter to the Insurance Commissioner, who
On February 24, 1957, the insured Francisco del Rosario, alias Paquito Bolero, while on board the motor rendered an opinion that the liability of the company was only P1,000.00, pursuant to Section 1, Part I of
launch "ISLAMA" together with 33 others, including his beneficiary in the Policy, Remedios Jayme, were the Provisions of the policy (Exh. F, or 3). Because of the above opinion, defendant insurance company
forced to jump off said launch on account of fire which broke out on said vessel, resulting in the death of refused to pay more than P1,000.00. In the meantime, Atty. Vicente Francisco, in a subsequent letter to
drowning, of the insured and beneficiary in the waters of Jolo. 1äwphï1.ñët the insurance company, asked for P3,000.00 which the Company refused, to pay. Hence, a complaint for
the recovery of the balance of P2,000.00 more was instituted with the Court of First Instance of Rizal
On April 13, 1957, Simeon del Rosario, father of the insured, and as the sole heir, filed a claim for (Pasay City, Branch VII), praying for it further sum of P10,000.00 as attorney's fees, expenses of litigation
payment with defendant company, and on September 13, 1957, defendant company paid to him and costs.
(plaintiff) the sum of P1,000.00, pursuant to Section 1 of Part I of the policy. The receipt signed by plaintiff
reads — Defendant Insurance Company presented a Motion to Dismiss, alleging that the demand or claim is set
forth in the complaint had already been released, plaintiff having received the full amount due as
RECEIVED of the EQUITABLE INSURANCE & CASUALTY CO., INC., the sum of PESOS — appearing in policy and as per opinion of the Insurance Commissioner. An opposition to the motion to
ONE THOUSAND (P1,000.00) Philippine Currency, being settlement in full for all claims dismiss, was presented by plaintiff, and other pleadings were subsequently file by the parties. On
and demands against said Company as a result of an accident which occurred on December 28, 1957, the trial court deferred action on the motion to dismiss until termination of the trial
February 26, 1957, insured under out ACCIDENT Policy No. 7136, causing the death of of the case, it appearing that the ground thereof was not indubitable. In the Answer to the complaint,
the Assured. defendant company practically admitted all the allegations therein, denying only those which stated that
under the policy its liability was P3,000.00.
In view of the foregoing, this policy is hereby surrendered and CANCELLED.

On September 1, 1958, the trial court promulgated an Amended Decision, the pertinent portions of which
LOSS COMPUTATION read —

Amount of Insurance P1,000.00


xxx xxx xxx
__________
vvvvv Since the contemporaneous and subsequent acts of the parties show that it was not their
intention that the payment of P1,000.00 to the plaintiff and the signing of the loss receipt exhibit
On the same date (September 13, 1957), Atty. Vicente J. Francisco, wrote defendant company
"1" would be considered as releasing the defendant completely from its liability on the policy in
acknowledging receipt by his client (plaintiff herein), of the P1,000.00, but informing said company that
question, said intention of the parties should prevail over the contents of the loss receipt "1"
said amount was not the correct one. Atty. Francisco claimed —
(Articles 1370 and 1371, New Civil Code).

Page 6 of 97
". . . . Under the terms of this policy, defendant company agreed to pay P1,000.00 to P3,000.00 The plaintiff asks for an award of P10,000.00 as attorney's fees and expenses of litigation.
as indemnity for the death of the insured. The insured died of drowning. Death by drowning is However, since it is evident that the defendant had not acted in bad faith in refusing to pay
covered by the policy the pertinent provisions of which reads as follows: plaintiff's claim, the Court cannot award plaintiff's claim for attorney's fees and expenses of
litigation.
xxx xxx xxx
IN VIEW OF THE FOREGOING, the Court hereby reconsiders and sets aside its decision dated
"Part I of the policy fixes specific amounts as indemnities in case of death resulting July 21, 1958 and hereby renders judgment, ordering the defendant to pay plaintiff the sum of
from "bodily injury which is effected solely thru violence, external, visible and Two Thousand (P2,000.00) Pesos and to pay the costs.
accidental means" but, Part I of the Policy is not applicable in case of death by
drowning because death by drowning is not one resulting from "bodily injury which is The above judgment was appealed to the Court of Appeals on three (3) counts. Said Court, in a
affected solely thru violent, external, visible and accidental means" as "Bodily Injury" Resolution dated September 29, 1959, elevated the case to this Court, stating that the genuine issue is
means a cut, a bruise, or a wound and drowning is death due to suffocation and not to purely legal in nature.
any cut, bruise or wound."
All the parties agree that indemnity has to be paid. The conflict centers on how much should the
xxx xxx xxx indemnity be. We believe that under the proven facts and circumstances, the findings and conclusions of
the trial court, are well taken, for they are supported by the generally accepted principles or rulings on
Besides, on the face of the policy Exhibit "A" itself, death by drowning is a ground for recovery insurance, which enunciate that where there is an ambiguity with respect to the terms and conditions of
apart from the bodily injury because death by bodily injury is covered by Part I of the policy the policy, the same will be resolved against the one responsible thereof. It should be recalled in this
while death by drowning is covered by Part VI thereof. But while the policy mentions specific connection, that generally, the insured, has little, if any, participation in the preparation of the policy,
amounts that may be recovered for death for bodily injury, yet, there is not specific amount together with the drafting of its terms and Conditions. The interpretation of obscure stipulations in a
mentioned in the policy for death thru drowning although the latter is, under Part VI of the contract should not favor the party who cause the obscurity (Art. 1377, N.C.C.), which, in the case at bar,
policy, a ground for recovery thereunder. Since the defendant has bound itself to pay P1000.00 is the insurance company.
to P3,000.00 as indemnity for the death of the insured but the policy does not positively state
any definite amount that may be recovered in case of death by drowning, there is an ambiguity . . . . And so it has been generally held that the "terms in an insurance policy, which are
in this respect in the policy, which ambiguity must be interpreted in favor of the insured and ambiguous, equivocal or uncertain . . . are to be construed strictly against, the insurer, and
strictly against the insurer so as to allow greater indemnity. liberally in favor of the insured so as to effect the dominant purpose of indemnity or payment to
the insured, especially where a forfeiture is involved," (29 Am. Jur. 181) and the reason for this
xxx xxx xxx rule is that the "insured usually has no voice in the selection or arrangement of the words
employed and that the language of the contract is selected with great care and deliberation by
. . . plaintiff is therefore entitled to recover P3,000.00. The defendant had already paid the expert and legal advisers employed by, and acting exclusively in the interest of, the insurance
amount of P1,000.00 to the plaintiff so that there still remains a balance of P2,000.00 of the company" (44 C.J.S. 1174). Calanoc v. Court of Appeals, et al., G.R. No. L-8151, Dec. 16, 1955.
amount to which plaintiff is entitled to recover under the policy Exhibit "A".

Page 7 of 97
. . . . Where two interpretations, equally fair, of languages used in an insurance policy may be vehicle owner, he was induced by Fieldmen's Insurance Company Pampanga agent Benjamin Sambat to
made, that which allows the greater indemnity will prevail. (L'Engel v. Scotish Union & Nat. F. Ins. apply for a Common Carrier's Liability Insurance Policy covering his motor vehicle ... Upon paying an
Co., 48 Fla. 82, 37 So. 462, 67 LRA 581 111 Am. St. Rep. 70, 5 Ann. Cas. 749). annual premium of P16.50, defendant Fieldmen's Insurance Company, Inc. issued on September 19, 1960,
Common Carriers Accident Insurance Policy No. 45-HO- 4254 ... the duration of which will be for one (1)
At any event, the policy under consideration, covers death or disability by accidental means, and the year, effective September 15, 1960 to September 15, 1961. On September 22, 1961, the defendant
appellant insurance company agreed to pay P1,000.00 to P3,000.00. is indemnity for death of the insured. company, upon payment of the corresponding premium, renewed the policy by extending the coverage
from October 15, 1961 to October 15, 1962. This time Federico Songco's private jeepney carried Plate No.
In view of the conclusions reached, it would seem unnecessary to discuss the other issues raised in the J-68136-Pampanga-1961. ... On October 29, 1961, during the effectivity of the renewed policy, the insured
appeal. vehicle while being driven by Rodolfo Songco, a duly licensed driver and son of Federico (the vehicle
owner) collided with a car in the municipality of Calumpit, province of Bulacan, as a result of which
The judgment appealed from is hereby affirmed. Without costs.
mishap Federico Songco (father) and Rodolfo Songco (son) died, Carlos Songco (another son), the
latter's wife, Angelita Songco, and a family friend by the name of Jose Manuel sustained physical injuries
Padilla, Bautista Angelo, Labrador, Concepcion, Reyes, J.B.L., Barrera, Dizon and Regala, JJ., concur.
of varying degree." 1
Makalintal, J., reserves his vote.

It was further shown according to the decision of respondent Court of Appeals: "Amor Songco, 42-year-
old son of deceased Federico Songco, testifying as witness, declared that when insurance agent Benjamin
3. Fieldmen’s Insurance Co. Inc. V. Vda de Songco (25 SCRA 70, GRN L-24833, 23 Sept.
Sambat was inducing his father to insure his vehicle, he butted in saying: 'That cannot be, Mr. Sambat,
1968)
because our vehicle is an "owner" private vehicle and not for passengers,' to which agent Sambat replied:
'whether our vehicle was an "owner" type or for passengers it could be insured because their company is
FERNANDO, J.:
not owned by the Government and the Government has nothing to do with their company. So they
could do what they please whenever they believe a vehicle is insurable' ... In spite of the fact that the
An insurance firm, petitioner Fieldmen's Insurance Co., Inc., was not allowed to escape liability under a
present case was filed and tried in the CFI of Pampanga, the defendant company did not even care to
common carrier insurance policy on the pretext that what was insured, not once but twice, was a private
rebut Amor Songco's testimony by calling on the witness-stand agent Benjamin Sambat, its Pampanga
vehicle and not a common carrier, the policy being issued upon the insistence of its agent who
Field Representative." 2
discounted fears of the insured that his privately owned vehicle might not fall within its terms, the insured
moreover being "a man of scant education," finishing only the first grade. So it was held in a decision of
The plaintiffs in the lower court, likewise respondents here, were the surviving widow and children of the
the lower court thereafter affirmed by respondent Court of Appeals. Petitioner in seeking the review of
deceased Federico Songco as well as the injured passenger Jose Manuel. On the above facts they
the above decision of respondent Court of Appeals cannot be so sanguine as to entertain the belief that
prevailed, as had been mentioned, in the lower court and in the respondent Court of Appeals.1awphîl.nèt
a different outcome could be expected. To be more explicit, we sustain the Court of Appeals.

The basis for the favorable judgment is the doctrine announced in Qua Chee Gan v. Law Union and Rock
The facts as found by respondent Court of Appeals, binding upon us, follow: "This is a peculiar case.
Insurance Co., Ltd., 3 with Justice J. B. L. Reyes speaking for the Court. It is now beyond question that
Federico Songco of Floridablanca, Pampanga, a man of scant education being only a first grader ...,
where inequitable conduct is shown by an insurance firm, it is "estopped from enforcing forfeitures in its
owned a private jeepney with Plate No. 41-289 for the year 1960. On September 15, 1960, as such private
favor, in order to forestall fraud or imposition on the insured." 4

Page 8 of 97
As much, if not much more so than the Qua Chee Gan decision, this is a case where the doctrine of appliances called for in the insured premises, since its initial expression, 'the undernoted appliances for
estoppel undeniably calls for application. After petitioner Fieldmen's Insurance Co., Inc. had led the the extinction of fire being kept on the premises insured hereby, ... it is hereby warranted ...,' admits of
insured Federico Songco to believe that he could qualify under the common carrier liability insurance interpretation as an admission of the existence of such appliances which appellant cannot now
policy, and to enter into contract of insurance paying the premiums due, it could not, thereafter, in any contradict, should the parol evidence rule apply." 7
litigation arising out of such representation, be permitted to change its stand to the detriment of the
heirs of the insured. As estoppel is primarily based on the doctrine of good faith and the avoidance of To the same effect is the following citation from the same leading case: "This rigid application of the rule
harm that will befall the innocent party due to its injurious reliance, the failure to apply it in this case on ambiguities has become necessary in view of current business practices. The courts cannot ignore that
would result in a gross travesty of justice. nowadays monopolies, cartels and concentration of capital, endowed with overwhelming economic
power, manage to impose upon parties dealing with them cunningly prepared 'agreements' that the
That is all that needs be said insofar as the first alleged error of respondent Court of Appeals is weaker party may not change one whit, his participation in the 'agreement' being reduced to the
concerned, petitioner being adamant in its far-from-reasonable plea that estoppel could not be invoked alternative to 'take it or leave it' labelled since Raymond Saleilles 'contracts by adherence' (contrats
by the heirs of the insured as a bar to the alleged breach of warranty and condition in the policy. lt would d'adhesion), in contrast to those entered into by parties bargaining on an equal footing, such contracts
now rely on the fact that the insured owned a private vehicle, not a common carrier, something which it (of which policies of insurance and international bills of lading are prime examples) obviously call for
knew all along when not once but twice its agent, no doubt without any objection in its part, exerted the greater strictness and vigilance on the part of courts of justice with a view to protecting the weaker party
utmost pressure on the insured, a man of scant education, to enter into such a contract. from abuses and imposition, and prevent their becoming traps for the unwary (New Civil Code. Article
24; Sent. of Supreme Court of Spain, 13 Dec. 1934, 27 February 1942)." 8
Nor is there any merit to the second alleged error of respondent Court that no legal liability was incurred
under the policy by petitioner. Why liability under the terms of the policy 5 was inescapable was set forth The last error assigned which would find fault with the decision of respondent Court of Appeals insofar as
in the decision of respondent Court of Appeals. Thus: "Since some of the conditions contained in the it affirmed the lower court award for exemplary damages as well as attorney's fees is, on its face, of no
policy issued by the defendant-appellant were impossible to comply with under the existing conditions at persuasive force at all.
the time and 'inconsistent with the known facts,' the insurer 'is estopped from asserting breach of such
conditions.' From this jurisprudence, we find no valid reason to deviate and consequently hold that the The conclusion that inescapably emerges from the above is the correctness of the decision of respondent
decision appealed from should be affirmed. The injured parties, to wit, Carlos Songco, Angelito Songco Court of Appeals sought to be reviewed. For, to borrow once again from the language of the Qua Chee
and Jose Manuel, for whose hospital and medical expenses the defendant company was being made Gan opinion: "The contract of insurance is one of perfect good faith (uberima fides) not for the insured
liable, were passengers of the jeepney at the time of the occurrence, and Rodolfo Songco, for whose alone,but equally so for the insurer; in fact, it is more so for the latter, since its dominant bargaining
burial expenses the defendant company was also being made liable was the driver of the vehicle in position carries with it stricter responsibility."9
question. Except for the fact, that they were not fare paying passengers, their status as beneficiaries
under the policy is recognized therein." 6 This is merely to stress that while the morality of the business world is not the morality of institutions of
rectitude like the pulpit and the academe, it cannot descend so low as to be another name for guile or
Even if it be assumed that there was an ambiguity, an excerpt from the Qua Chee Gan decision would deception. Moreover, should it happen thus, no court of justice should allow itself to lend its approval
reveal anew the weakness of petitioner's contention. Thus: "Moreover, taking into account the well and support.1awphîl.nèt
known rule that ambiguities or obscurities must be strictly interpreted against the party that caused
them, the 'memo of warranty' invoked by appellant bars the latter from questioning the existence of the

Page 9 of 97
We have no choice but to recognize the monetary responsibility of petitioner Fieldmen's Insurance Co., corporation has ceased to be effective because of the outbreak of the war between the United States
Inc. It did not succeed in its persistent effort to avoid complying with its obligation in the lower court and and Germany on December 10, 1941, and that the payment made by the petitioner to the respondent
the Court of Appeals. Much less should it find any receptivity from us for its unwarranted and unjustified corporation during the Japanese military occupation was under pressure. After trial, the Court of First
plea to escape from its liability. Instance of Manila dismissed the action without pronouncement as to costs. Upon appeal to the Court of
Appeals, the judgment of the Court of First Instance of Manila was affirmed, with costs. The case is now
WHEREFORE, the decision of respondent Court of Appeals of July 20, 1965, is affirmed in its entirety. before us on appeal by certiorari from the decision of the Court of Appeals.
Costs against petitioner Fieldmen's Insurance Co., Inc.
The Court of Appeals overruled the contention of the petitioner that the respondent corporation became
Concepcion, C.J., Reyes, J.B.L., Dizon, Makalintal, Zaldivar, Sanchez, Castro and Angeles, JJ., concur. an enemy when the United States declared war against Germany, relying on English and American cases
which held that a corporation is a citizen of the country or state by and under the laws of which it was
created or organized. It rejected the theory that nationality of private corporation is determine by the
4. Filipinas Cia de Seguros V. Christern Huenefeld & Co, Inc. (89 Phil. 54, GRN L-2294, 25 character or citizenship of its controlling stockholders.
May 1951)
There is no question that majority of the stockholders of the respondent corporation were German
PARAS, C.J.: subjects. This being so, we have to rule that said respondent became an enemy corporation upon the
outbreak of the war between the United States and Germany. The English and American cases relied
On October 1, 1941, the respondent corporation, Christern Huenefeld, & Co., Inc., after payment of
upon by the Court of Appeals have lost their force in view of the latest decision of the Supreme Court of
corresponding premium, obtained from the petitioner ,Filipinas Cia. de Seguros, fire policy No. 29333 in
the United States in Clark vs. Uebersee Finanz Korporation, decided on December 8, 1947, 92 Law. Ed.
the sum of P1000,000, covering merchandise contained in a building located at No. 711 Roman Street,
Advance Opinions, No. 4, pp. 148-153, in which the controls test has been adopted. In "Enemy
Binondo Manila. On February 27, 1942, or during the Japanese military occupation, the building and
Corporation" by Martin Domke, a paper presented to the Second International Conference of the Legal
insured merchandise were burned. In due time the respondent submitted to the petitioner its claim
Profession held at the Hague (Netherlands) in August. 1948 the following enlightening passages appear:
under the policy. The salvage goods were sold at public auction and, after deducting their value, the total
loss suffered by the respondent was fixed at P92,650. The petitioner refused to pay the claim on the Since World War I, the determination of enemy nationality of corporations has been discussion
ground that the policy in favor of the respondent had ceased to be in force on the date the United States in many countries, belligerent and neutral. A corporation was subject to enemy legislation when
declared war against Germany, the respondent Corporation (though organized under and by virtue of it was controlled by enemies, namely managed under the influence of individuals or
the laws of the Philippines) being controlled by the German subjects and the petitioner being a company corporations, themselves considered as enemies. It was the English courts which first
under American jurisdiction when said policy was issued on October 1, 1941. The petitioner, however, in the Daimler case applied this new concept of "piercing the corporate veil," which was adopted
pursuance of the order of the Director of Bureau of Financing, Philippine Executive Commission, dated by the peace of Treaties of 1919 and the Mixed Arbitral established after the First World War.
April 9, 1943, paid to the respondent the sum of P92,650 on April 19, 1943.
The United States of America did not adopt the control test during the First World War. Courts
The present action was filed on August 6, 1946, in the Court of First Instance of Manila for the purpose of refused to recognized the concept whereby American-registered corporations could be
recovering from the respondent the sum of P92,650 above mentioned. The theory of the petitioner is considered as enemies and thus subject to domestic legislation and administrative measures
that the insured merchandise were burned up after the policy issued in 1941 in favor of the respondent regarding enemy property.

Page 10 of 97
World War II revived the problem again. It was known that German and other enemy interests The Philippine Insurance Law (Act No. 2427, as amended,) in section 8, provides that "anyone except a
were cloaked by domestic corporation structure. It was not only by legal ownership of shares public enemy may be insured." It stands to reason that an insurance policy ceases to be allowable as
that a material influence could be exercised on the management of the corporation but also by soon as an insured becomes a public enemy.
long term loans and other factual situations. For that reason, legislation on enemy property
enacted in various countries during World War II adopted by statutory provisions to the control Effect of war, generally. — All intercourse between citizens of belligerent powers which is
test and determined, to various degrees, the incidents of control. Court decisions were rendered inconsistent with a state of war is prohibited by the law of nations. Such prohibition includes all
on the basis of such newly enacted statutory provisions in determining enemy character of negotiations, commerce, or trading with the enemy; all acts which will increase, or tend to
domestic corporation. increase, its income or resources; all acts of voluntary submission to it; or receiving its
protection; also all acts concerning the transmission of money or goods; and all contracts
The United States did not, in the amendments of the Trading with the Enemy Act during the last relating thereto are thereby nullified. It further prohibits insurance upon trade with or by the
war, include as did other legislations the applications of the control test and again, as in World enemy, upon the life or lives of aliens engaged in service with the enemy; this for the reason
War I, courts refused to apply this concept whereby the enemy character of an American or that the subjects of one country cannot be permitted to lend their assistance to protect by
neutral-registered corporation is determined by the enemy nationality of the controlling insurance the commerce or property of belligerent, alien subjects, or to do anything detrimental
stockholders. too their country's interest. The purpose of war is to cripple the power and exhaust the
resources of the enemy, and it is inconsistent that one country should destroy its enemy's
Measures of blocking foreign funds, the so called freezing regulations, and other administrative property and repay in insurance the value of what has been so destroyed, or that it should in
practice in the treatment of foreign-owned property in the United States allowed to large such manner increase the resources of the enemy, or render it aid, and the commencement of
degree the determination of enemy interest in domestic corporations and thus the application war determines, for like reasons, all trading intercourse with the enemy, which prior thereto may
of the control test. Court decisions sanctioned such administrative practice enacted under the have been lawful. All individuals therefore, who compose the belligerent powers, exist, as to
First War Powers Act of 1941, and more recently, on December 8, 1947, the Supreme Court of each other, in a state of utter exclusion, and are public enemies. (6 Couch, Cyc. of Ins. Law, pp.
the United States definitely approved of the control theory. In Clark vs. Uebersee Finanz 5352-5353.)
Korporation, A. G., dealing with a Swiss corporation allegedly controlled by German interest, the
Court: "The property of all foreign interest was placed within the reach of the vesting power (of In the case of an ordinary fire policy, which grants insurance only from year, or for some other
the Alien Property Custodian) not to appropriate friendly or neutral assets but to reach enemy specified term it is plain that when the parties become alien enemies, the contractual tie is
interest which masqueraded under those innocent fronts. . . . The power of seizure and vesting broken and the contractual rights of the parties, so far as not vested. lost. (Vance, the Law on
was extended to all property of any foreign country or national so that no innocent appearing Insurance, Sec. 44, p. 112.)
device could become a Trojan horse."
The respondent having become an enemy corporation on December 10, 1941, the insurance policy issued
It becomes unnecessary, therefore, to dwell at length on the authorities cited in support of the appealed in its favor on October 1, 1941, by the petitioner (a Philippine corporation) had ceased to be valid and
decision. However, we may add that, in Haw Pia vs. China Banking Corporation,* 45 Off Gaz., (Supp. 9) enforcible, and since the insured goods were burned after December 10, 1941, and during the war, the
299, we already held that China Banking Corporation came within the meaning of the word "enemy" as respondent was not entitled to any indemnity under said policy from the petitioner. However, elementary
used in the Trading with the Enemy Acts of civilized countries not only because it was incorporated under rules of justice (in the absence of specific provision in the Insurance Law) require that the premium paid
the laws of an enemy country but because it was controlled by enemies.

Page 11 of 97
by the respondent for the period covered by its policy from December 11, 1941, should be returned by 5. Geagonio V. CA (241 SCRA 152, GRN 114427, 6 February 1995)
the petitioner.
DAVIDE, JR., J.:
The Court of Appeals, in deciding the case, stated that the main issue hinges on the question of whether
the policy in question became null and void upon the declaration of war between the United States and Four our review under Rule 45 of the Rules of Court is the decision1 of the Court of Appeals in CA-G.R. SP
Germany on December 10, 1941, and its judgment in favor of the respondent corporation was predicated No. 31916, entitled "Country Bankers Insurance Corporation versus Armando Geagonia," reversing the
on its conclusion that the policy did not cease to be in force. The Court of Appeals necessarily assumed decision of the Insurance Commission in I.C. Case No. 3340 which awarded the claim of petitioner
that, even if the payment by the petitioner to the respondent was involuntary, its action is not tenable in Armando Geagonia against private respondent Country Bankers Insurance Corporation.
view of the ruling on the validity of the policy. As a matter of fact, the Court of Appeals held that "any
intimidation resorted to by the appellee was not unjust but the exercise of its lawful right to claim for and The petitioner is the owner of Norman's Mart located in the public market of San Francisco, Agusan del

received the payment of the insurance policy," and that the ruling of the Bureau of Financing to the Sur. On 22 December 1989, he obtained from the private respondent fire insurance policy No. F-

effect that "the appellee was entitled to payment from the appellant was, well founded." Factually, there 146222 for P100,000.00. The period of the policy was from 22 December 1989 to 22 December 1990 and

can be no doubt that the Director of the Bureau of Financing, in ordering the petitioner to pay the claim covered the following: "Stock-in-trade consisting principally of dry goods such as RTW's for men and

of the respondent, merely obeyed the instruction of the Japanese Military Administration, as may be seen women wear and other usual to assured's business."

from the following: "In view of the findings and conclusion of this office contained in its decision on
The petitioner declared in the policy under the subheading entitled CO-INSURANCE that Mercantile
Administrative Case dated February 9, 1943 copy of which was sent to your office and the concurrence
Insurance Co., Inc. was the co-insurer for P50,000.00. From 1989 to 1990, the petitioner had in his
therein of the Financial Department of the Japanese Military Administration, and following the instruction
inventory stocks amounting to P392,130.50, itemized as follows:
of said authority, you are hereby ordered to pay the claim of Messrs. Christern, Huenefeld & Co., Inc. The
payment of said claim, however, should be made by means of crossed check." (Emphasis supplied.)
Zenco Sales, Inc. P55,698.00

It results that the petitioner is entitled to recover what paid to the respondent under the circumstances F. Legaspi Gen. Merchandise 86,432.50
on this case. However, the petitioner will be entitled to recover only the equivalent, in actual Philippines Cebu Tesing Textiles 250,000.00 (on credit)
currency of P92,650 paid on April 19, 1943, in accordance with the rate fixed in the Ballantyne scale.
—————

P392,130.50
Wherefore, the appealed decision is hereby reversed and the respondent corporation is ordered to pay
to the petitioner the sum of P77,208.33, Philippine currency, less the amount of the premium, in
The policy contained the following condition:
Philippine currency, that should be returned by the petitioner for the unexpired term of the policy in
question, beginning December 11, 1941. Without costs. So ordered.
3. The insured shall give notice to the Company of any insurance or insurances already
affected, or which may subsequently be effected, covering any of the property or
Feria, Pablo, Bengzon, Tuason, Montemayor, Jugo and Bautista Angelo, JJ., concur.
properties consisting of stocks in trade, goods in process and/or inventories only
hereby insured, and unless such notice be given and the particulars of such insurance
or insurances be stated therein or endorsed in this policy pursuant to Section 50 of the

Page 12 of 97
Insurance Code, by or on behalf of the Company before the occurrence of any loss or In its answer,7 the private respondent specifically denied the allegations in the complaint and set up as its
damage, all benefits under this policy shall be deemed forfeited, provided however, principal defense the violation of Condition 3 of the policy.
that this condition shall not apply when the total insurance or insurances in force at the
time of the loss or damage is not more than P200,000.00. In its decision of 21 June 1993,8 the Insurance Commission found that the petitioner did not violate
Condition 3 as he had no knowledge of the existence of the two fire insurance policies obtained from the
On 27 May 1990, fire of accidental origin broke out at around 7:30 p.m. at the public market of San PFIC; that it was Cebu Tesing Textiles which procured the PFIC policies without informing him or securing
Francisco, Agusan del Sur. The petitioner's insured stock-in-trade were completely destroyed prompting his consent; and that Cebu Tesing Textile, as his creditor, had insurable interest on the stocks. These
him to file with the private respondent a claim under the policy. On 28 December 1990, the private findings were based on the petitioner's testimony that he came to know of the PFIC policies only when
respondent denied the claim because it found that at the time of the loss the petitioner's stocks-in-trade he filed his claim with the private respondent and that Cebu Tesing Textile obtained them and paid for
were likewise covered by fire insurance policies No. GA-28146 and No. GA-28144, for P100,000.00 each, their premiums without informing him thereof. The Insurance Commission then decreed:
issued by the Cebu Branch of the Philippines First Insurance Co., Inc. (hereinafter PFIC). These policies
3

indicate that the insured was "Messrs. Discount Mart (Mr. Armando Geagonia, Prop.)" with a mortgage WHEREFORE, judgment is hereby rendered ordering the respondent company to pay
clause reading: complainant the sum of P100,000.00 with legal interest from the time the complaint
was filed until fully satisfied plus the amount of P10,000.00 as attorney's fees. With
MORTGAGE: Loss, if any shall be payable to Messrs. Cebu Tesing Textiles, Cebu City as costs. The compulsory counterclaim of respondent is hereby dismissed.
their interest may appear subject to the terms of this policy. CO-INSURANCE
DECLARED: P100,000. — Phils. First CEB/F 24758.4 Its motion for the reconsideration of the decision 9 having been denied by the Insurance Commission in
its resolution of 20 August 1993, 10 the private respondent appealed to the Court of Appeals by way of a
The basis of the private respondent's denial was the petitioner's alleged violation of Condition 3 of the petition for review. The petition was docketed as CA-G.R. SP No. 31916.
policy.
In its decision of 29 December 1993, 11 the Court of Appeals reversed the decision of the Insurance
The petitioner then filed a complaint 5 against the private respondent with the Insurance Commission Commission because it found that the petitioner knew of the existence of the two other policies issued
(Case No. 3340) for the recovery of P100,000.00 under fire insurance policy No. F-14622 and for by the PFIC. It said:
attorney's fees and costs of litigation. He attached as Annex "AM"6 thereof his letter of 18 January 1991
which asked for the reconsideration of the denial. He admitted in the said letter that at the time he It is apparent from the face of Fire Policy GA 28146/Fire Policy No. 28144 that the
obtained the private respondent's fire insurance policy he knew that the two policies issued by the PFIC insurance was taken in the name of private respondent [petitioner herein]. The policy
were already in existence; however, he had no knowledge of the provision in the private respondent's states that "DISCOUNT MART (MR. ARMANDO GEAGONIA, PROP)" was the assured
policy requiring him to inform it of the prior policies; this requirement was not mentioned to him by the and that "TESING TEXTILES" [was] only the mortgagee of the goods.
private respondent's agent; and had it been mentioned, he would not have withheld such information.
He further asserted that the total of the amounts claimed under the three policies was below the actual In addition, the premiums on both policies were paid for by private respondent, not by

value of his stocks at the time of loss, which was P1,000,000.00. the Tesing Textiles which is alleged to have taken out the other insurance without the
knowledge of private respondent. This is shown by Premium Invoices nos. 46632 and
46630. (Annexes M and N). In both invoices, Tesing Textiles is indicated to be only the

Page 13 of 97
mortgagee of the goods insured but the party to which they were issued were the support my claim that the amount claimed under the three policies
"DISCOUNT MART (MR. ARMANDO GEAGONIA)." are much below the value of my stocks lost.

In is clear that it was the private respondent [petitioner herein] who took out the xxx xxx xxx
policies on the same property subject of the insurance with petitioner. Hence, in failing
to disclose the existence of these insurances private respondent violated Condition No. The letter contradicts private respondent's pretension that he did not know that there
3 of Fire Policy No. 1462. . . . were other insurances taken on the stock-in-trade and seriously puts in question his
credibility.
Indeed private respondent's allegation of lack of knowledge of the provisions
insurances is belied by his letter to petitioner [of 18 January 1991. The body of the letter His motion to reconsider the adverse decision having been denied, the petitioner filed the instant
reads as follows;] petition. He contends therein that the Court of Appeals acted with grave abuse of discretion amounting
to lack or excess of jurisdiction:
xxx xxx xxx
A — . . . WHEN IT REVERSED THE FINDINGS OF FACTS OF THE INSURANCE
Please be informed that I have no knowledge of the provision COMMISSION, A QUASI-JUDICIAL BODY CHARGED WITH THE DUTY OF
requiring me to inform your office about my DETERMINING INSURANCE CLAIM AND WHOSE DECISION IS ACCORDED RESPECT
prior insurance under FGA-28146 and F-CEB-24758. Your AND EVEN FINALITY BY THE COURTS;
representative did not mention about said requirement at the time
he was convincing me to insure with you. If he only die or even B — . . . WHEN IT CONSIDERED AS EVIDENCE MATTERS WHICH WERE NOT
inquired if I had other existing policies covering my establishment, I PRESENTED AS EVIDENCE DURING THE HEARING OR TRIAL; AND
would have told him so. You will note that at the time he talked to
me until I decided to insure with your company the two policies C — . . . WHEN IT DISMISSED THE CLAIM OF THE PETITIONER HEREIN AGAINST THE

aforementioned were already in effect. Therefore I would have no PRIVATE RESPONDENT.

reason to withhold such information and I would have desisted to


The chief issues that crop up from the first and third grounds are (a) whether the petitioner had prior
part with my hard earned peso to pay the insurance premiums [if] I
knowledge of the two insurance policies issued by the PFIC when he obtained the fire insurance policy
know I could not recover anything.
from the private respondent, thereby, for not disclosing such fact, violating Condition 3 of the policy, and

Sir, I am only an ordinary businessman interested in protecting my (b) if he had, whether he is precluded from recovering therefrom.

investments. The actual value of my stocks damaged by the fire was


The second ground, which is based on the Court of Appeals' reliance on the petitioner's letter of
estimated by the Police Department to be P1,000,000.00 (Please see
reconsideration of 18 January 1991, is without merit. The petitioner claims that the said letter was not
xerox copy of Police Report Annex "A"). My Income Statement as of
offered in evidence and thus should not have been considered in deciding the case. However, as
December 31, 1989 or five months before the fire, shows my
correctly pointed out by the Court of Appeals, a copy of this letter was attached to the petitioner's
merchandise inventory was already some P595,455.75. . . . These will

Page 14 of 97
complaint in I.C. Case No. 3440 as Annex "M" thereof and made integral part of the complaint. 12 It has property.19 The mortgagee's insurable interest is to the extent of the debt, since the property is relied
attained the status of a judicial admission and since its due execution and authenticity was not denied by upon as security thereof, and in insuring he is not insuring the property but his interest or lien thereon.
the other party, the petitioner is bound by it even if it were not introduced as an independent His insurable interest is prima facie the value mortgaged and extends only to the amount of the debt, not
evidence. 13 exceeding the value of the mortgaged property. 20 Thus, separate insurances covering different insurable
interests may be obtained by the mortgagor and the mortgagee.
As to the first issue, the Insurance Commission found that the petitioner had no knowledge of the
previous two policies. The Court of Appeals disagreed and found otherwise in view of the explicit A mortgagor may, however, take out insurance for the benefit of the mortgagee, which is the usual
admission by the petitioner in his letter to the private respondent of 18 January 1991, which was quoted in practice. The mortgagee may be made the beneficial payee in several ways. He may become the
the challenged decision of the Court of Appeals. These divergent findings of fact constitute an exception assignee of the policy with the consent of the insurer; or the mere pledgee without such consent; or the
to the general rule that in petitions for review under Rule 45, only questions of law are involved and original policy may contain a mortgage clause; or a rider making the policy payable to the mortgagee "as
findings of fact by the Court of Appeals are conclusive and binding upon this Court. 14 his interest may appear" may be attached; or a "standard mortgage clause," containing a collateral
independent contract between the mortgagee and insurer, may be attached; or the policy, though by its
We agree with the Court of Appeals that the petitioner knew of the prior policies issued by the PFIC. His terms payable absolutely to the mortgagor, may have been procured by a mortgagor under a contract
letter of 18 January 1991 to the private respondent conclusively proves this knowledge. His testimony to duty to insure for the mortgagee's benefit, in which case the mortgagee acquires an equitable lien upon
the contrary before the Insurance Commissioner and which the latter relied upon cannot prevail over a the proceeds. 21
written admission made ante litem motam. It was, indeed, incredible that he did not know about the
prior policies since these policies were not new or original. Policy No. GA-28144 was a renewal of Policy In the policy obtained by the mortgagor with loss payable clause in favor of the mortgagee as his interest
No. F-24758, while Policy No. GA-28146 had been renewed twice, the previous policy being F-24792. may appear, the mortgagee is only a beneficiary under the contract, and recognized as such by the
insurer but not made a party to the contract himself. Hence, any act of the mortgagor which defeats his
Condition 3 of the private respondent's Policy No. F-14622 is a condition which is not proscribed by law. right will also defeat the right of the mortgagee. 22 This kind of policy covers only such interest as the
Its incorporation in the policy is allowed by Section 75 of the Insurance Code 15 which provides that "[a] mortgagee has at the issuing of the policy.23
policy may declare that a violation of specified provisions thereof shall avoid it, otherwise the breach of
an immaterial provision does not avoid the policy." Such a condition is a provision which invariably On the other hand, a mortgagee may also procure a policy as a contracting party in accordance with the
appears in fire insurance policies and is intended to prevent an increase in the moral hazard. It is terms of an agreement by which the mortgagor is to pay the premiums upon such insurance. 24 It has
commonly known as the additional or "other insurance" clause and has been upheld as valid and as a been noted, however, that although the mortgagee is himself the insured, as where he applies for a
warranty that no other insurance exists. Its violation would thus avoid the policy, fully informs the authorized agent of his interest, pays the premiums, and obtains on the
policy. 16 However, in order to constitute a violation, the other insurance must be upon same subject assurance that it insures him, the policy is in fact in the form used to insure a mortgagor with loss
matter, the same interest therein, and the same risk.17 payable clause. 25

As to a mortgaged property, the mortgagor and the mortgagee have each an independent insurable The fire insurance policies issued by the PFIC name the petitioner as the assured and contain a mortgage
interest therein and both interests may be one policy, or each may take out a separate policy covering clause which reads:
his interest, either at the same or at separate times. 18 The mortgagor's insurable interest covers the full
value of the mortgaged property, even though the mortgage debt is equivalent to the full value of the

Page 15 of 97
Loss, if any, shall be payable to MESSRS. TESING TEXTILES, Cebu City as their interest for this is that, except for riders which may later be inserted, the insured sees the contract already in its
may appear subject to the terms of this policy. final form and has had no voice in the selection or arrangement of the words employed therein. On the
other hand, the language of the contract was carefully chosen and deliberated upon by experts and legal
This is clearly a simple loss payable clause, not a standard mortgage clause. advisers who had acted exclusively in the interest of the insurers and the technical language employed
therein is rarely understood by ordinary laymen. 31
It must, however, be underscored that unlike the "other insurance" clauses involved in General Insurance
and Surety Corp. vs. Ng Hua 26 or in Pioneer Insurance & Surety Corp. vs. Yap, 27 which read: With these principles in mind, we are of the opinion that Condition 3 of the subject policy is not totally
free from ambiguity and must, perforce, be meticulously analyzed. Such analysis leads us to conclude
The insured shall give notice to the company of any insurance or insurances already that (a) the prohibition applies only to double insurance, and (b) the nullity of the policy shall only be to
effected, or which may subsequently be effected covering any of the property hereby the extent exceeding P200,000.00 of the total policies obtained.
insured, and unless such notice be given and the particulars of such insurance or
insurances be stated in or endorsed on this Policy by or on behalf of the Company The first conclusion is supported by the portion of the condition referring to other insurance "covering
before the occurrence of any loss or damage, all benefits under this Policy shall be any of the property or properties consisting of stocks in trade, goods in process and/or inventories only
forfeited. hereby insured," and the portion regarding the insured's declaration on the subheading CO-INSURANCE
that the co-insurer is Mercantile Insurance Co., Inc. in the sum of P50,000.00. A double insurance exists
or in the 1930 case of Santa Ana vs. Commercial Union Assurance where the same person is insured by several insurers separately in respect of the same subject and
Co. 28 which provided "that any outstanding insurance upon the whole or a portion of the interest. As earlier stated, the insurable interests of a mortgagor and a mortgagee on the mortgaged
objects thereby assured must be declared by the insured in writing and he must cause the property are distinct and separate. Since the two policies of the PFIC do not cover the same interest as
company to add or insert it in the policy, without which such policy shall be null and void, and that covered by the policy of the private respondent, no double insurance exists. The non-disclosure then
the insured will not be entitled to indemnity in case of loss," Condition 3 in the private of the former policies was not fatal to the petitioner's right to recover on the private respondent's policy.
respondent's policy No. F-14622 does not absolutely declare void any violation thereof. It
expressly provides that the condition "shall not apply when the total insurance or insurances in Furthermore, by stating within Condition 3 itself that such condition shall not apply if the total insurance
force at the time of the loss or damage is not more than P200,000.00." in force at the time of loss does not exceed P200,000.00, the private respondent was amenable to
assume a co-insurer's liability up to a loss not exceeding P200,000.00. What it had in mind was to
It is a cardinal rule on insurance that a policy or insurance contract is to be interpreted liberally in favor of discourage over-insurance. Indeed, the rationale behind the incorporation of "other insurance" clause in
the insured and strictly against the company, the reason being, undoubtedly, to afford the greatest fire policies is to prevent over-insurance and thus avert the perpetration of fraud. When a property
protection which the insured was endeavoring to secure when he applied for insurance. It is also a owner obtains insurance policies from two or more insurers in a total amount that exceeds the property's
cardinal principle of law that forfeitures are not favored and that any construction which would result in value, the insured may have an inducement to destroy the property for the purpose of collecting the
the forfeiture of the policy benefits for the person claiming thereunder, will be avoided, if it is possible to insurance. The public as well as the insurer is interested in preventing a situation in which a fire would be
construe the policy in a manner which would permit recovery, as, for example, by finding a waiver for profitable to the insured.32
such forfeiture. 29 Stated differently, provisions, conditions or exceptions in policies which tend to work a
forfeiture of insurance policies should be construed most strictly against those for whose benefits they WHEREFORE, the instant petition is hereby GRANTED. The decision of the Court of Appeals in CA-G.R. SP
are inserted, and most favorably toward those against whom they are intended to operate. 30 The reason No. 31916 is SET ASIDE and the decision of the Insurance Commission in Case No. 3340 is REINSTATED.

Page 16 of 97
Costs against private respondent Country Bankers Insurance Corporation. 5. That the plaintiff charged as expenses of its business all the said premiums and deducted the
same from its gross incomes as reported in its annual income tax returns, which deductions
SO ORDERED. were allowed by the defendant upon a showing made by the plaintiff that such premiums were
legitimate expenses of its (plaintiff's) business.
6. El Oriente and Posadas (56 Phil 147, GRN 34774, 21 Sept. 1931)\
6. That the said A. Velhagen, the insured, had no interest or participation in the proceeds of said
MALCOLM, J.: life insurance policy.

The issue in this case is whether the proceeds of insurance taken by a corporation on the life of an 7. That upon the death of said A. Velhagen in the year 1929, the plaintiff received all the
important official to indemnify it against loss in case of his death, are taxable as income under the proceeds of the said life insurance policy, together with the interests and the dividends accruing
Philippine Income Tax Law. thereon, aggregating P104,957.88.

The parties submitted the case to the Court of First Instance of Manila for decision upon the following 8. That over the protest of the plaintiff, which claimed exemption under section 4 of the Income
agreed statement of facts: Tax Law, the defendant Collector of Internal Revenue assessed and levied the sum of P3,148.74
as income tax on the proceeds of the insurance policy mentioned in the preceding paragraph,
1. That the plaintiff is a domestic corporation duly organized and existing under and by virtue of
which tax the plaintiff paid under instant protest on July 2, 1930; and that defendant overruled
the laws of the Philippine Islands, having its principal office at No. 732 Calle Evangelista, Manila,
said protest on July 9, 1930.
P.I.; and that the defendant is the duly appointed, qualified and acting Collector of Internal
Revenue of the Philippine Islands. Thereupon, a decision was handed down which absolved the defendant from the complaint, with costs
against the plaintiff. From this judgment, the plaintiff appealed, and its counsel now allege that:
2. That on March 18, 1925, plaintiff, in order to protect itself against the loss that it might suffer
by reason of the death of its manager, A. Velhagen, who had had more than thirty-five (35) 1. That trial court erred in holding that section 4 of the Income Tax Law (Act No. 2833) is not
years of experience in the manufacture of cigars in the Philippine Islands, and whose death applicable to the present case.
would be a serious loss to the plaintiff, procured from the Manufacturers Life Insurance Co., of
Toronto, Canada, thru its local agent E.E. Elser, an insurance policy on the life of the said A. 2. The trial court erred in reading into the law certain exceptions and distinctions not warranted
Velhagen for the sum of $50,000, United States currency. by its clear and unequivocal provisions.

3. That the plaintiff, El Oriente, Fabrica de Tabacos, Inc., designated itself as the sole beneficiary 3. The trial court erred in assuming that the proceeds of the life insurance policy in question
of said policy on the life of its said manager. represented a net profit to the plaintiff when, as a matter of fact, it merely represented an
indemnity, for the loss suffered by it thru the death of its manager, the insured.
4. That during the time the life insurance policy hereinbefore referred to was in force and effect
plaintiff paid from its funds all the insurance premiums due thereon. 4. The trial court erred in refusing to hold that the proceeds of the life insurance policy in
question is not taxable income, and in absolving the defendant from the complaint.

Page 17 of 97
The Income Tax Law for the Philippines is Act No. 2833, as amended. It is divided into four chapters: It will be recalled that El Oriente, Fabrica de Tabacos, Inc., took out the insurance on the life of its
Chapter I On Individuals, Chapter II On Corporations, Chapter III General Administrative Provisions, and manager, who had had more than thirty-five years' experience in the manufacture of cigars in the
Chapter IV General Provisions. In chapter I On Individuals, is to be found section 4 which provides that, Philippines, to protect itself against the loss it might suffer by reason of the death of its manager. We do
"The following incomes shall be exempt from the provisions of this law: ( a) The proceeds of life insurance not believe that this fact signifies that when the plaintiff received P104,957.88 from the insurance on the
policies paid to beneficiaries upon the death of the insured ... ." Section 10, as amended, in Chapter II On life of its manager, it thereby realized a net profit in this amount. It is true that the Income Tax Law, in
Corporations, provides that, There shall be levied, assessed, collected, and paid annually upon the total exempting individual beneficiaries, speaks of the proceeds of life insurance policies as income, but this is
net income received in the preceding calendar year from all sources by every corporation ... a tax of a very slight indication of legislative intention. In reality, what the plaintiff received was in the nature of an
three per centum upon such income ... ." Section 11 in the same chapter, provides the exemptions under indemnity for the loss which it actually suffered because of the death of its manager.
the law, but neither here nor in any other section is reference made to the provisions of section 4 in
Chapter I. To quote the exact words in the cited case of Chief Justice Taft delivering the opinion of the court:

Under the view we take of the case, it is sufficient for our purposes to direct attention to the anomalous It is earnestly pressed upon us that proceeds of life insurance paid on the death of the insured
and vague condition of the law. It is certain that the proceeds of life insurance policies are exempt. It is are in fact capital, and cannot be taxed as income under the Sixteenth Amendment. Eisner vs.
not so certain that the proceeds of life insurance policies paid to corporate beneficiaries upon the death Macomber, 252 U.S., 189, 207; Merchants' Loan & Trust Co. vs. Smietanka, 255 U.S., 509, 518.
of the insured are likewise exempt. But at least, it may be said that the law is indefinite in phraseology We are not required to meet this question. It is enough to sustain our construction of the act to
and does not permit us unequivocally to hold that the proceeds of life insurance policies received by say that proceeds of a life insurance policy paid on the death of the insured are not usually
corporations constitute income which is taxable. classed as income.

The situation will be better elucidated by a brief reference to laws on the same subject in the United . . . Life insurance in such a case is like that of fire and marine insurance, — a contract of
States. The Income Tax Law of 1916 extended to the Philippine Legislature, when it came to enact Act No. indemnity. Central Nat. Bank vs. Hume, 128 U.S., 195. The benefit to be gained by death has no
2833, to copy the American statute. Subsequently, the Congress of the United States enacted its Income periodicity. It is a substitution of money value for something permanently lost, either in a house,
Tax Law of 1919, in which certain doubtful subjects were clarified. Thus, as to the point before us, it was a ship, or a life. Assuming, without deciding, that Congress could call the proceeds of such
made clear, when not only in the part of the law concerning individuals were exemptions provided for indemnity income, and validly tax it as such, we think that, in view of the popular conception of
beneficiaries, but also in the part concerning corporations, specific reference was made to the the life insurance as resulting in a single addition of a total sum to the resources of the
exemptions in favor of individuals, thereby making the same applicable to corporations. This was beneficiary, and not in a periodical return, such a purpose on its part should be express, as it
authoritatively pointed out and decided by the United States Supreme Court in the case of United certainly is not here.
States vs. Supplee-Biddle Hardware Co. ( [1924], 265 U.S., 189), which involved facts quite similar to those
before us. We do not think the decision of the higher court in this case is necessarily controlling on Considering, therefore, the purport of the stipulated facts, considering the uncertainty of Philippine law,

account of the divergences noted in the federal statute and the local statute, but we find in the decision and considering the lack of express legislative intention to tax the proceeds of life insurance policies paid

certain language of a general nature which appears to furnish the clue to the correct disposition of the to corporate beneficiaries, particularly when in the exemption in favor of individual beneficiaries in the

instant appeal. Conceding, therefore, without necessarily having to decide, the assignments of error Nos. chapter on this subject, the clause is inserted "exempt from the provisions of this law," we deem it

1 and 2 are not well taken, we would turn to the third assignment of error. reasonable to hold the proceeds of the life insurance policy in question as representing an indemnity and
not taxable income.

Page 18 of 97
The foregoing pronouncement will result in the judgment being reversed and in another judgment being Motion, thus allowing the private respondent to adduce evidence, the consequence of which was the
rendered in favor of the plaintiff and against the defendant for the sum of P3,148.74. So ordered, without issuance of the questioned Order granting the petition.
costs in either instance.
Petitioner promptly filed a Motion for Reconsideration but the same was denied in an Order June 10,
Avanceña, C.J., Street, Villamor, Ostrand, Romualdez, Villa-Real, and Imperial, JJ., concur. 1980. Hence, this petition raising the following issues for resolution:chanrob1es virtual 1aw library

7. Philippine American Life Insurance Co. V. Pineda (155 SCRA 416, GRN L-54216, 19 July WHETHER OR NOT THE DESIGNATION OF THE IRREVOCABLE BENEFICIARIES COULD BE CHANGED OR
1989) AMENDED WITHOUT THE CONSENT OF ALL THE IRREVOCABLE BENEFICIARIES.

II
PARAS, J.:
WHETHER OR NOT THE IRREVOCABLE BENEFICIARIES HEREIN, ONE OF WHOM IS ALREADY DECEASED
Challenged before Us in this petition for review on certiorari are the Orders of the respondent Judge
WHILE THE OTHERS ARE ALL MINORS, COULD VALIDLY GIVE CONSENT TO THE CHANGE OR
dated March 19, 1980 and June 10, 1980 granting the prayer in the petition in Sp. Proc. No. 9210 and
AMENDMENT IN THE DESIGNATION OF THE IRREVOCABLE BENEFICIARIES.
denying petitioner’s Motion for Reconsideration, respectively.

The undisputed facts are as follows:chanrob1es virtual 1aw library


We are of the opinion that his Honor, the respondent Judge, was in error in issuing the questioned
On January 15, 1968, private respondent procured an ordinary life insurance policy from the petitioner
Orders.
company and designated his wife and children as irrevocable beneficiaries of said policy.
Needless to say, the applicable law in the instant case is the Insurance Act, otherwise known as Act No.
Under date February 22, 1980 private respondent filed a petition which was docketed as Civil Case No.
2427 as amended, the policy having been procured in 1968. Under the said law, the beneficiary
9210 of the then Court of First Instance of Rizal to amend the designation of the beneficiaries in his life
designated in a life insurance contract cannot be changed without the consent of the beneficiary because
policy from irrevocable to revocable.
he has a vested interest in the policy (Gercio v. Sun Life Ins. Co. of Canada, 48 Phil. 53; Go v. Redfern and

Petitioner, on March 10, 1980 filed an Urgent Motion to Reset Hearing. Also on the same date, petitioner the International Assurance Co., Ltd., 72 Phil. 71).

filed its Comment and/or Opposition to Petition.


In this regard, it is worth noting that the Beneficiary Designation Indorsement in the policy which forms

When the petition was called for hearing on March 19, 1980, the respondent Judge Gregorio G. Pineda, part of Policy Number 0794461 in the name of Rodolfo Cailles Dimayuga states that the designation of

presiding Judge of the then Court of First Instance of Rizal, Pasig Branch XXI, denied petitioner’s Urgent the beneficiaries is irrevocable (Annex "A" of Petition in Sp. Proc. No. 9210, Annex "C" of the Petition for
Review on Certiorari), to wit:chanrob1es virtual 1aw library

Page 19 of 97
It is hereby understood and agreed that, notwithstanding the provisions of this policy to the contrary, Of equal importance is the well-settled rule that the contract between the parties is the law binding on
inasmuch as the designation of the primary/contingent beneficiary/beneficiaries in this Policy has been both of them and for so many times, this court has consistently issued pronouncements upholding the
made without reserving the right to change said beneficiary/beneficiaries, such designation may not be validity and effectivity of contracts. Where there is nothing in the contract which is contrary to law, good

surrendered to the Company, released or assigned; and no right or privilege under the Policy may be morals, good customs, public policy or public order the validity of the contract must be sustained.
exercised, or agreement made with the Company to any change in or amendment to the Policy, without Likewise, contracts which are the private laws of the contracting parties should be fulfilled according to
the consent of the said beneficiary/beneficiaries. (Petitioner’s Memorandum, p. 72, Rollo) the literal sense of their stipulations, if their terms are clear and leave no room for doubt as to the
intention of the contracting parties, for contracts are obligatory, no matter in what form they may be,
Be it noted that the foregoing is a fact which the private respondent did not bother to disprove.
whenever the essential requisites for their validity are present (Phoenix Assurance Co., Ltd. v. United

Inevitably therefore, based on the aforequoted provision of the contract, not to mention the law then States Lines, 22 SCRA 675, Phil. American General Insurance Co., Inc. v. Mutuc, 61 SCRA 22.)

applicable, it is only with the consent of all the beneficiaries that any change or amendment in the policy
In the recent case of Francisco Herrera v. Petrophil Corporation, 146 SCRA 385, this Court ruled that:
concerning the irrevocable beneficiaries may be legally and validly effected. Both the law and the policy
do not provide for any other exception, thus, abrogating the contention of the private respondent that ". . . it is settled that the parties may establish such stipulations, clauses, terms, and conditions as they

said designation can be amended if the Court finds a just, reasonable ground to do so. may want to include; and as long as such agreements are not contrary to law, good morals, good
customs, public policy or public order, they shall have the force of law between them."
Similarly, the alleged acquiescence of the six (6) children beneficiaries of the policy (the beneficiary-wife
predeceased the insured) cannot be considered an effective ratification to the change of the beneficiaries Undeniably, the contract in the case at bar, contains the indispensable elements for its validity and does
from irrevocable to revocable. Indubitable is the fact that all the six (6) children named as beneficiaries not in any way violate the law, morals, customs, orders, etc. leaving no reason for Us to deny sanction

were minors at the time, ** for which reason, they could not validly give their consent. Neither could they thereto.
act through their father-insured since their interests are quite divergent from one another. In point is an
Finally, the fact that the contract of insurance does not contain a contingency when the change in the
excerpt from the Notes and Cases on Insurance Law by Campos and Campos, 1960, reading —
designation of beneficiaries could be validly effected means that it was never within the contemplation of
"The insured . . . can do nothing to divest the beneficiary of his rights without his consent. He cannot the parties. The lower court, in gratuitously providing for such contingency, made a new contract for

assign his policy, nor even take its cash surrender value without the consent of the beneficiary. Neither them, a proceeding which we cannot tolerate. Ergo, We cannot help but conclude that the lower court
can the insured’s creditors seize the policy or any right thereunder. The insured may not even add acted in excess of its authority when it issued the Order dated March 19, 1980 amending the designation
another beneficiary because by doing so, he diminishes the amount which the beneficiary may recover of the beneficiaries from "irrevocable" to "revocable" over the disapprobation of the petitioner insurance
and this he cannot do without the beneficiary’s consent." company.

Therefore, the parent-insured cannot exercise rights and/or privileges pertaining to the insurance WHEREFORE, premises considered, the questioned Orders of the respondent Judge are hereby nullified
contract, for otherwise, the vested rights of the irrevocable beneficiaries would be rendered and set aside.
inconsequential.
Page 20 of 97
SO ORDERED. February 11, 1980, with interest thereon at the rate of 12% per annum on each of the said
monthly amounts of P10,000.00 from the time the same became due until it is paid;
Melencio-Herrera (Chairman), Sarmiento and Regalado, JJ., concur.
5. Ordering the plaintiff to pay the defendant through the injunction bond, the sum of
Padilla, J., No part in the deliberations.
P100,000.00, representing the P10,000.00 monthly increase in rentals which the defendant failed
to realize from February to November 1980 result from the injunction, with legal interest
8. Harvadian Colleges V. Country Bankers Insurance Corp. (1 SCRA 1, GRN 85161, 9 Sept.
thereon from the finality of this decision until fully paid;
1991)

6. Ordering the plaintiff to pay to the defendant the sum equivalent to ten per centum (10%) of
MEDIALDEA, J.:p
the above-mentioned amounts of P289,534.78, P100,000.00 and P100,000.00, as and for
attorney's fees; and
Petitioners seek a review on certiorari of the decision of the Court of Appeals in CA-G.R. CV No. 09504
"Enrique Sy and Country Bankers Insurance Corporation v. Oscar Ventanilla Enterprises Corporation"
7. Ordering the plaintiff to pay the costs. (pp. 94-95, Rollo)
affirming in toto the decision of the Regional Trial Court, Cabanatuan City, Branch XXV, to wit:

The antecedent facts of the case are as follows:


WHEREFORE, the complaint of the plaintiff Enrique F. Sy is dismissed, and on the counterclaim of the
defendant O. Ventanilla Enterprises Corporation, judgment is hereby rendered:
Respondent Oscar Ventanilla Enterprises Corporation (OVEC), as lessor, and the petitioner Enrique F. Sy,
as lessee, entered into a lease agreement over the Avenue, Broadway and Capitol Theaters and the land
1. Declaring as lawful, the cancellation and termination of the Lease Agreement (Exh. A) and the
on which they are situated in Cabanatuan City, including their air-conditioning systems, projectors and
defendant's re-entry and repossession of the Avenue, Broadway and Capitol theaters under
accessories needed for showing the films or motion pictures. The term of the lease was for six (6) years
lease on February 11, 1980;
commencing from June 13, 1977 and ending June 12,1983. After more than two (2) years of operation of
the Avenue, Broadway and Capitol Theaters, the lessor OVEC made demands for the repossession of the
2. Declaring as lawful, the forfeiture clause under paragraph 12 of the Id Lease Agreement, and
said leased properties in view of the Sy's arrears in monthly rentals and non-payment of amusement
confirming the forfeiture of the plaintiffs remaining cash deposit of P290,000.00 in favor of the
taxes. On August 8,1979, OVEC and Sy had a conference and by reason of Sy's request for
defendant thereunder, as of February 11, 1980;
reconsideration of OVECs demand for repossession of the three (3) theaters, the former was allowed to

3. Ordering the plaintiff to pay the defendant the sum of P289,534.78, representing arrears in continue operating the leased premises upon his conformity to certain conditions imposed by the latter

rentals, unremitted amounts for amusement tax delinquency and accrued interest thereon, with in a supplemental agreement dated August 13, 1979.

further interest on said amounts at the rate of 12% per annum (per lease agreement) from
In pursuance of their latter agreement, Sy's arrears in rental in the amount of P125,455.76 (as of July 31,
December 1, 1980 until the same is fully paid;
1979) was reduced to P71,028.91 as of December 31, 1979. However, the accrued amusement tax liability

4. Ordering the plaintiff to pay the defendant the amount of P100,000.00, representing the of the three (3) theaters to the City Government of Cabanatuan City had accumulated to P84,000.00

P10,000.00 portion of the monthly lease rental which were not deducted from the cash deposit despite the fact that Sy had been deducting the amount of P4,000.00 from his monthly rental with the

of the plaintiff from February to November, 1980, after the forfeiture of the said cash deposit on obligation to remit the said deductions to the city government. Hence, letters of demand dated January

Page 21 of 97
7, 1980 and February 3, 1980 were sent to Sy demanding payment of the arrears in rentals and OVEC had thus become forfeited; that OVEC would be losing P50,000.00 for every month that the
amusement tax delinquency. The latter demand was with warning that OVEC will re-enter and repossess possession and operation of said three theaters remain with Sy and that OVEC incurred P500,000.00 for
the Avenue, Broadway and Capital Theaters on February 11, 1980 in pursuance of the pertinent provisions attorney's service.
of their lease contract of June 11, 1977 and their supplemental letter-agreement of August 13, 1979. But
notwithstanding the said demands and warnings SY failed to pay the above-mentioned amounts in full The trial court arrived at the conclusions that Sy is not entitled to the reformation of the lease agreement;
Consequently, OVEC padlocked the gates of the three theaters under lease and took possession thereof that the repossession of the leased premises by OVEC after the cancellation and termination of the lease
in the morning of February 11, 1980 by posting its men around the premises of the Id movie houses and was in accordance with the stipulation of the parties in the said agreement and the law applicable thereto
preventing the lessee's employees from entering the same. and that the consequent forfeiture of Sy's cash deposit in favor of OVEC was clearly agreed upon by
them in the lease agreement. The trial court further concluded that Sy was not entitled to the writ of
Sy, through his counsel, filed the present action for reformation of the lease agreement, damages and preliminary injunction issued in his favor after the commencement of the action and that the injunction
injunction late in the afternoon of the same day. And by virtue of a restraining order dated February 12, bond filed by Sy is liable for whatever damages OVEC may have suffered by reason of the injunction.
1980 followed by an order directing the issuance of a writ of preliminary injunction issued in said case, Sy
regained possession and operation of the Avenue, Broadway and Capital theaters. On the counterclaim of OVEC the trial court found that the said lessor was deprived of the possession
and enjoyment of the leased premises and also suffered damages as a result of the filing of the case by
As first cause of action, Sy alleged that the amount of deposit — P600,000.00 as agreed upon, Sy and his violation of the terms and conditions of the lease agreement. Hence, it held that OVEC is
P300,000.00 of which was to be paid on June 13, 1977 and the balance on December 13, 1977 — was too entitled to recover the said damages in addition to the arrears in rentals and amusement tax delinquency
big; and that OVEC had assured him that said forfeiture will not come to pass. By way of second cause of of Sy and the accrued interest thereon. From the evidence presented, it found that as of the end of
action, Sy sought to recover from OVEC the sums of P100,000.00 which Sy allegedly spent in making November, 1980, when OVEC finally regained the possession of the three (3) theaters under lease, Sy's
"major repairs" on Broadway Theater and the application of which to Sy's due rentals; (2) P48,000.00 unpaid rentals and amusement tax liability amounted to P289,534.78. In addition, it held that Sy was
covering the cost of electrical current allegedly used by OVEC in its alleged "illegal connection" to Capitol under obligation to pay P10,000.00 every month from February to November, 1980 or the total amount
Theater and (3) P31,000.00 also for the cost of electrical current allegedly used by OVEC for its alleged of P100,000.00 with interest on each amount of P10,000.00 from the time the same became due. This
"illegal connection" to Broadway Theater and for damages suffered by Sy as a result of such connection. P10,000.00 portion of the monthly lease rental was supposed to come from the remaining cash deposit
Under the third cause of action, it is alleged in the complaint that on February 11, 1980, OVEC had the of Sy but with the consequent forfeiture of the remaining cash deposit of P290,000.00, there was no
three theaters padlocked with the use of force, and that as a result, Sy suffered damages at the rate of more cash deposit from which said amount could be deducted. Further, it adjudged Sy to pay attorney's
P5,000.00 a day, in view of his failure to go thru the contracts he had entered into with movie and fees equivalent to 10% of the amounts above-mentioned.
booking companies for the showing of movies at ABC. As fourth cause of action, Sy prayed for the
issuance of a restraining order/preliminary injunction to enjoin OVEC and all persons employed by it Finally, the trial court held Sy through the injunction bond liable to pay the sum of P10,000.00 every
from entering and taking possession of the three theaters, conditioned upon Sy's filing of a P500,000.00 month from February to November, 1980. The amount represents the supposed increase in rental from
bond supplied by Country Bankers Insurance Corporation (CBISCO). P50,000.00 to P60,000.00 in view of the offer of one RTG Productions, Inc. to lease the three theaters
involved for P60,000.00 a month.
OVEC on the other hand, alleged in its answer by way of counterclaims, that by reason of Sy's violation of
the terms of the subject lease agreement, OVEC became authorized to enter and possess the three
theaters in question and to terminate said agreement and the balance of the deposits given by Sy to

Page 22 of 97
From this decision of the trial court, Sy and (CBISCO) appealed the decision in toto while OVEC appealed RESPONDENT COURT OF APPEALS FURTHER COMMITTED SERIOUS ERROR OF LAW AND
insofar as the decision failed to hold the injunction bond liable for an damages awarded by the trial GRAVE ABUSE OF DISCRETION IN NOT DISMISSING PRIVATE RESPONDENTS COUNTER-CLAIM
court. FOR FAILURE TO PAY THE NECESSARY DOCKET FEE. (p. 10, Rollo)

The respondent Court of Appeals found no ambiguity in the provisions of the lease agreement. It held We find no merit in petitioners' argument that the forfeiture clause stipulated in the lease agreement
that the provisions are fair and reasonable and therefore, should be respected and enforced as the law would unjustly enrich the respondent OVEC at the expense of Sy and CBISCO — contrary to law, morals,
between the parties. It held that the cancellation or termination of the agreement prior to its expiration good customs, public order or public policy. A provision which calls for the forfeiture of the remaining
period is justified as it was brought about by Sy's own default in his compliance with the terms of the deposit still in the possession of the lessor, without prejudice to any other obligation still owing, in the
agreement and not "motivated by fraud or greed." It also affirmed the award to OVEC of the amount of event of the termination or cancellation of the agreement by reason of the lessee's violation of any of the
P100,000.00 chargeable against the injunction bond posted by CBISCO which was soundly and amply terms and conditions of the agreement is a penal clause that may be validly entered into. A penal clause
justified by the trial court. is an accessory obligation which the parties attach to a principal obligation for the purpose of insuring
the performance thereof by imposing on the debtor a special presentation (generally consisting in the
The respondent Court likewise found no merit in OVECS appeal and held that the trial court did not err in payment of a sum of money) in case the obligation is not fulfilled or is irregularly or inadequately fulfilled.
not charging and holding the injunction bond posted by Sy liable for all the awards as the undertaking of (Eduardo P. Caguioa, Comments and Cases on Civil Law, Vol. IV, First Edition, pp. 199-200) As a general
CBISCO under the bond referred only to damages which OVEC may suffer as a result of the injunction. rule, in obligations with a penal clause, the penalty shall substitute the indemnity for damages and the
payment of interests in case of non-compliance. This is specifically provided for in Article 1226, par. 1,
From this decision, CBISCO and Sy filed this instant petition on the following grounds: New Civil Code. In such case, proof of actual damages suffered by the creditor is not necessary in order
that the penalty may be demanded (Article 1228, New Civil Code). However, there are exceptions to the
A
rule that the penalty shall substitute the indemnity for damages and the payment of interests in case of
non-compliance with the principal obligation. They are first, when there is a stipulation to the contrary;
PRIVATE RESPONDENT SHOULD NOT BE ALLOWED TO UNJUSTLY ENRICH OR BE BENEFITTED
second, when the obligor is sued for refusal to pay the agreed penalty; and third, when the obligor is
AT THE EXPENSE OF THE PETITIONERS.
guilty of fraud (Article 1226, par. 1, New Civil Code). It is evident that in all said cases, the purpose of the
penalty is to punish the obligor. Therefore, the obligee can recover from the obligor not only the penalty
B
but also the damages resulting from the non-fulfillment or defective performance of the principal
obligation.
RESPONDENT COURT OF APPEALS CO D SERIOUS ERROR OF LAW AND GRAVE ABUSE OF
DISCRETION IN NOT SETTING OFF THE P100,000.00 SUPPOSED DAMAGE RESULTING FROM
In the case at bar, inasmuch as the forfeiture clause provides that the deposit shall be deemed forfeited,
THE INJUNCTION AGAINST THE P290,000.00 REMAINING CASH DEPOSIT OF PETITIONER
without prejudice to any other obligation still owing by the lessee to the lessor, the penalty cannot
ENRIQUE SY.
substitute for the P100,000.00 supposed damage resulting from the issuance of the injunction against the
P290,000.00 remaining cash deposit. This supposed damage suffered by OVEC was the alleged
C
P10,000.00 a month increase in rental from P50,000.00 to P60,000,00), which OVEC failed to realize for
ten months from February to November, 1980 in the total sum of P100,000.00. This opportunity cost
which was duly proven before the trial court, was correctly made chargeable by the said court against the
Page 23 of 97
injunction bond posted by CBISCO. The undertaking assumed by CBISCO under subject injunction refers or transaction within the limitation on counterclaims imposed by the statutes depends on a consideration
to "all such damages as such party may sustain by reason of the injunction if the Court should finally of all the facts brought forth by the parties and on a determination of whether there is some legal or
decide that the Plaintiff was/were not entitled thereto." (Rollo, p. 101) Thus, the respondent Court equitable relationship between the ground of recovery alleged in the counterclaim and the matters
correctly sustained the trial court in holding that the bond shall and may answer only for damages which alleged as the cause of action by the plaintiff (80 C.J.S. 48). As the counterclaims of OVEC arise from or
OVEC may suffer as a result of the injunction. The arrears in rental, the unmeritted amounts of the are necessarily connected with the facts alleged in the complaint for reformation of instrument of Sy, it is
amusement tax delinquency, the amount of P100,000.00 (P10,000.00 portions of each monthly rental clear that said counterclaims are compulsory.
which were not deducted from plaintiffs cash deposit from February to November, 1980 after the
forfeiture of said cash deposit on February 11, 1980) and attorney's fees which were all charged against Sy ACCORDINGLY, finding no merit in the grounds relied upon by petitioners in their petition, the same is
were correctly considered by the respondent Court as damages which OVEC sustained not as a result of hereby DENIED and the decision dated June 15, 1988 and the resolution dated September 21, 1988, both
the injunction. of the respondent Court of Appeals are AFFIRMED.

There is likewise no merit to the claim of petitioners that respondent Court committed serious error of SO ORDERED.
law and grave abuse of discretion in not dismissing private respondent's counterclaim for failure to pay
the necessary docket fee, which is an issue raised for the first time in this petition. Petitioners rely on the
rule in Manchester Development Corporation v. Court of Appeals, G.R. No. 75919, May 7, 1987, 149 SCRA
9. Traders Insurance & Surety Co. V. Golangco (95 Phil 826, GRN L-6442, 21 Sept. 1954)
562 to the effect that all the proceedings held in connection with a case where the correct docket fees
are not paid should be peremptorily be considered null and void because, for all legal purposes, the trial
court never acquired jurisdiction over the case. It should be remembered however, that in Davao Light
(NOTE: The original text is in Spanish. This was directly translated to English through Google)
and Power Co., Inc. v. Dinopol, G.R. 75195, August 19, 1988, 164 SCRA 748, this Court took note of the
fact that the assailed order of the trial court was issued prior to the resolution in the Manchester case and SYLLABUS
held that its strict application to the case at bar would therefore be unduly harsh. Thus, We allowed the
amendment of the complaint by specifying the amount of damages within a non-extendible period of
five (5) days from notice and the re-assessment of the filing fees. Then, in Sun Insurance Office, Ltd. v. 1. APPEAL; ERROR RELATIONSHIP; THE COURT OF APPEAL IS OBLIGED TO CONSIDER ALL THE ERRORS
Asuncion, G.R. 79937-38, February 3, 1989, 170 SCRA 274, We held that where the filing of the initiatory SIGNED IN THE ERROR RELATIONSHIP, BUT NOT ALL THAT ARE MADE IN THE COURSE OF
pleading is not accompanied by payment of the docket fee, the court may allow payment of the fee ARGUMENTS. - Law No. 296 was discussed and approved in English; We have to adopt the English text
within a reasonable time but in no case beyond the applicable prescriptive or reglemen tary period. and not the translation into Spanish. Article 22 of said law, talking about issues that arise properly before
the Court of Appeal does not mean that this has to resolve all issues that the arguments. That is not the
Nevertheless, OVEC's counterclaims are compulsory so no docket fees are required as the following proper place; We must raise it in the relation of errors. Note that it uses "properly raised" and not raised
circumstances are present: (a) they arise out of or are necessarily connected with the transaction or only. It is not enough to raise the question: it is necessary to raise it in the right time and place.
occurrence that is subject matter of the opposing party's claim; (b) they do not require for their
adjudication the presence of third parties of whom the court cannot acquire jurisdiction; and (c) the court
DECISION
has jurisdiction to entertain the claim (see Javier v. Intermediate Appellate Court, G.R. 75379, March 31,
1989, 171 SCRA 605). Whether the respective claims asserted by the parties arise out of the same contract
Page 24 of 97
"THE LOWER COURT ERRED IN HOLDING THAT THE APPELLEE HAD INSURABLE INTEREST CONSISTING
OF A RIGHT TO RECEIVE RENTALS BOTH AT THE TIME WHEN THE INSURANCE TOOK EFFECT AND
PAUL, M .:
WHEN THE LOSS OCCURRED."

It involves charging a fire insurance policy for a value of P10,000. The plaintiff was entitled to receive
monthly P1,100 from Melitona Estrella as rents for building No. 34 Plaza Sta. Cruz, Manila, for a period of
five years. The plaintiff secured this right, paid the premium and the defendant issued the corresponding In resolving these issues, the Court of Appeal, after transcribing the entire decision of the Court of First

policy. Two months after the building burned down, the plaintiff required Traders Insurance & Surety Co. Instance, said the following in its decision:

to pay the insurance amount. As the defendant refused to pay it, the plaintiff went to the Court of First
and it is particularly so because the policy prepared and issued by the very defendant specifically states
Instance of Manila, which issued a ruling in its favor. The Court of Appeal confirmed the sentence.
that all insurance covered under said policy, includes the 'rent or other subject matter of insurance in

In certiorari resource, Traders Insurance & Surety Co. goes to this Court, alleging that the Court of Appeal respect of or in connection with any building or any property contained in any building '. Under the

made two errors: 1 (a) in issuing a judgment without a complete conclusion of the facts of all the issues evidence on record We cannot alter in the least the decision aforequoted which is hereby adopted by

raised, as required by Article 33 of the Law No. 296, and 1 (b) by not making factual conclusions this Court. "

regarding the application of the oral test rule; and (2) when issuing judgment without factual conclusions
The findings of fact of the Court of First Instance (not including the list of facts) that were adopted by the
regarding Exhibits 10-H and 10-I.
Court of Appeal are the following:

As for error 1 (a). What are the issues that had been properly raised before the Court of Appeal? The best
"After considering the manner of testifying of these witnesses, the evasiveness of the witness Limpe, the
answer is the first page of the appellant's allegation (now recurring) that reads: virtual chanrob1es 1aw
improbability of his testimony, and the failure of defense to present Antonio Paredes, the clerk who
library
admittedly investigated the premises in question, the Court finds that plaintiff's version is more credible;

BRIEF FOR THE APPELLANT that, before the policy (Exhibit A) was issued, plaintiff made full and clear exposal of his interests in the

premises; and that the said fire policy, (Exhibit A) covers all of plaintiff's interests in the premises No. 34
"ASSIGNMENT OF ERRORS
Plaza Sta. Cruz, Manila, especially his right to collect rentals therefrom under the decision of this Court in
Civil Case No. 6306 (Exhibit C) This finding is further strengthened by the fact that paragraph 4 of the
I
said fire policy (Exhibit A), above quoted, includes insurance 'on rent';and accords with rule that a policy
"THE LOWER COURT ERRED IN HOLDING THAT THE FIRE INSURANCE POLICY EXHIBIT 'A' COVERS ALL is to be interpreted in favor of the assured.
APPELLEE'S INTERESTS IN THE PREMISES NO. 34 PLAZA STA. CRUZ, MANILA, ESPECIALLY HIS RIGHT TO
"The argument of the defendant that, under section 49 of the Insurance Law, a policy of insurance must
COLLECT RENTALS THEREFROM,
specify the interest of the insured in the property insured, if he is not the absolute owner thereof, is not
II meritorious because it was the defendant, not plaintiff, who prepared that policy, and it cannot take

Page 25 of 97
advantage of its own acts to plaintiff's detriment; and, in any case, this provision was substantially application of the rule on oral evidence is also unfounded. If the appellant wanted to provoke the
complied with by plaintiff when I made a full and clear statement of his interests to defendant's manager. improper admission of the plaintiff's oral testimony, or if he wanted to request the discarding of said
testimony, he must have indicated it in the relation of errors, saying that the judge erred in admitting the
Both at the time of the issuance of the policy and at the time of the fire, plaintiff Golangco was in legal
plaintiff's oral testimony, or that the Judge failed to discard the plaintiff's oral testimony despite the
possession of the premises, collecting rentals from its occupant (tr., Nov. 7, 1950, pp. 8, 10). It seems plain
petition duly submitted.
that if the premises were destroyed - as they were - by fire, Golangco would be, as he was, directly
damnified accordingly; and hence he had an insurable interest therein (section 12, Insurance Law).

"Defendant's contrary contentions are without merit. The contract between Lianco and the Archbishop In paragraph 3 of the arguments in support of the first error, under the title of Argument, (page 13 of the
only forbade Lianco from transferring 'his rights as LESSEE' (Exhibit 4-D); but the contracts Lianco made allegation presented in the Court of Appeal), it is when such a question was first raised, instead of raising
in favor of Kaw Eng Si (Exhibit D) and plaintiff Golangco (Exhibit C) did not transfer such rights; and hence it in the relationship of errors: it was not raised, then, in its proper place and in a timely manner. (Rule 48,
no written consent thereto was necessary. At worst, the contract would be voidable, but not a void art. 17). The Court of Appeal is not a diver who has to look in the arguments of the allegation which are
contract, at the option of the Archbishop; but this would not deprive Golangco of his insurable interest the mistakes made.

until such option were exercised; and it does not appear that it was ever exercised.
Regarding the second error, that is, that the Court of Appeal has not established conclusions of fact on
"The ejectment case filed by the Archbishop against Lianco did not remove nor destroy plaintiff's Exhibits 10-H and 10-I, we believe that it is deprived of merit, for two reasons: 1st because it was not
insurable interest: first, because plaintiff was not a party thereto and cannot be bound conseque; and raised in the relation of errors, and 2.a because in the decision there are such conclusions, although they
second, because the judgment of the Municipal Court, at least as late as February 14, 1950, had not been are not liked by the appellant. She says in her plea, page 31:

executed so far as possession of the premises were concerned (Exhibit G-10). In fact, not even
"The Court of Appeals adopted the finding of the trial court that 'not even garnishment was issued
garnishments were issued against Melitona Estrella, So Eng Si (her husband) or plaintiff Golangco, the
against Melitona Estrella, So Eng Si (her husband), or plaintiff Golangco The decision of the Court of
actual and legal possessors of the premises (Exhibit F); so that, as far as plaintiff Golangco was
Appeals should therefore contain the following finding of fact: chanrob1es virtual 1aw library
concerned, his right to the premises and to the rentals thereon continued to exist on June 5, 1949 when
the fire took place. " (a) That Exhibits '10 -H 'and '10 -I' clearly prove that notice of garnishment was served on No. 34 Plaza
Sta. Cruz (the property in question) and the occupants of No. 34 Plaza Sta. Cruz made return to the said
The findings of fact adopted by the Court of Appeal establish that the plaintiff insured his interest in the
garnishment. "
building No. 34 Plaza Sta. Cruz, consisting of the right to collect rents and that said insurable interest
existed at the time of the insurance and when the fire occurred . Apparently, the appellant wishes this Court to amend the factual findings of the Court of Appeal on
Exhibits 10-H and 10-I, which we cannot do. We do not review the evidence "Whether or not the Court of
The contention of the appellant that the decision of the Court of Appeal does not contain factual
Appeal has erred in these conclusions, it is not in us to make them straight. We must not interfere in the
conclusions of the issues properly raised is therefore unfounded. Error 1 (b). The contention of the
functions that, by provision of the law, correspond to said court.
appellant that the Court of Appeal must have stated in its decision the factual findings regarding the
Page 26 of 97
The law cited by the appellant reads as follows: Kiok, Et. Al. V. Macario Tiacho, 45 Off. Gaz., 2466, 79 Phil., 696; and Villareal v. The People of the
Philippines, 47 Off. Gaz., 191, 84 Phil., 264.) v. Macario Tiacho, 45 Off. Gaz., 2466, 79 Phil., 696; and
"Every decision of the Court of Appeals shall contain complete findings of fact on all issues properly
Villareal v. The People of the Philippines, 47 Off. Gaz., 191, 84 Phil., 264.) v. Macario Tiacho, 45 Off. Gaz.,
raised before it."
2466, 79 Phil., 696; and Villareal v. The People of the Philippines, 47 Off. Gaz., 191, 84 Phil., 264.)

But in Spanish it says like this:

"Where an appeal is taken to this court from any court, the appellant shall file with the clerk of the court

"Every decision of the Court of Appeals will contain a complete account of the facts of all the issues that below, with his petition for appeal, an assignment of errors, which shall set out separately and particularly

arise before it." each error asserted. No appeal shall be allowed unless such an assignment of errors shall accompany the
petition. " (Rule 9, Revised Rules of the Supreme Court of the United States, 11 US Supreme Court Report
The translation into Spanish is not accurate. If we stick to the translation into Spanish, the decision of the
Digest.)
Court of Appeal should not contain more than a "complete account of the facts of all issues" that may
have arisen therein. Something would be missing, namely the factual findings of the court. The "complete "The Supreme Court of the United States will not consider a question not raised below, not discussed by

relation of the facts" is not equivalent to "the complete relation of the conclusions of fact." The relation of the lower court, and not included in the assignment of errors. " (Pacific States Box & Basket Co. v. ST

facts "statement of facts" comes to be the narration of the facts of the matter so that it is known what it White, Et Al., 80 L ed., 138.)

is. The law in English speaks of complete findings of fact or complete conclusions of fact. The findings of
We could list a long list of decisions of the Supreme Courts of the states of the American Union that have
fact are those that the court adopts after studying, discussing and considering the contradictory evidence
regulations similar to ours; but the cases cited are enough.
presented at trial.
"Error not indicated in the relation of errors in a civil case is considered an error consented by the
Article 33 of said law, when talking about issues that arise properly before the Court of Appeal, does not
interested party." (Vitug v. Montemayor, 49 Off. Gaz., 5350.)
mean that it has to resolve all issues that arise in any state of the case or in the course of the arguments.
That is not the appropriate place: it must be raised in the relation of errors. Note that he uses the words We declare that an issue not specified in the error relationship is not properly raised, and the Court of
"properly raised" and not raised only. It is not enough to raise the question: it is necessary to raise it in a Appeal is not required to resolve it.
timely manner. It is a well established forensic practice that no matter that has not been properly raised
The petition is denied with costs against the appellant.
in the relationship of errors will be considered, unless it is a lack of jurisdiction, which may arise in any

state of the matter. (Enriquez et al. Against Enriquez et al., 8 Jur. Fil., 574; Capellania de Tambobong 10. Tai Tong Chuache & Co. V. Insurance Commission (158 SCRA 366, GRN L-55397, 29
against Antonio, 8 Jur. Fil., 693; Paternal vs. City of Manila, 17 Jur. Fil., 26; Santiago v. Felix, 24 Jur. Fil., 391; Feb. 1988)
Tan Me Nio v Customs Administrator, 34 Jur. Fil., 992; Granados y Granados v. Bandelaria, 45 Jur. Fil.,
530; Gemora v. Municipal Council of Ilog, 58 Jur. Fil., 377; Sanchez v. Land Director, 63 Jur. Fil., 403; Tan Si
GANCAYCO, J.:
Page 27 of 97
This petition for review on certiorari seeks the reversal of the decision of the Insurance Commission in IC British
Case #367[1] dismissing the complaint[2] for recovery of the alleged unpaid balance of the proceeds of Assco. Co.
the Fire Insurance Policies issued by herein respondent insurance company in favor of petitioner- Inc. FFF & F5 50,000 39, 186. 10
intervenor. FIC-15381 SSS
Accredited
The facts of the case as found by respondent Insurance Commission are as follows: Group of
"Complainants acquired from a certain Rolando Gonzales a parcel of land and a building located at San Insurers Building P25, 000 P8, 805.47
Rafael Village, Davao City. Complainants assumed the mortgage of the building in favor of S.S.S., which Totals P195, 000 P 90, 257.81
building was insured with respondent S.S.S. Accredited Group of Insurers for P25,000.00.
We are showing hereunder another apportionment of the loss which includes the Travellers
On April 19, 1975, Azucena Palomo obtained a loan from Tai Tong Chuache, Inc. in the amount of Multi?Indemnity policy for reference purposes.
P100,000.00. To secure the payment of the loan, a mortgage was executed over the land and the building
in favor of Tai Tong Chuache & Co. (Exhibit "1" and "1-A"). On April 25, 1975, Arsenio Chua, representative Policy No. Company Risk Insures Pays
of Thai Tong Chuache & Co. insured the latter's interest with Travellers Multi-Indemnity Corporation for MIRO/ Zenith
P100,000.00 (P70,000.00 for the building and P30,000.00 for the contents thereof) (Exhibit "A-a", contents F-02500 Insurance
thereof) (Exhibit "A-a"). Corp. Building P50, 000 P 11, 877.14
F-84590 Phil.
On June 11, 1975, Pedro Palomo secured a Fire Insurance Policy No. F-02500 (Exhibit "A"), covering the British
building for P50,000.00 with respondent Zenith Insurance Corporation. On July 16, 1975, another Fire Assco. Co. I-Buliding 70, 000 16, 628.00
Insurance Policy No. 8459 (Exhibit "B") was procured from respondent Philippine British Assurance II-Building
Company, covering the same building for P50,000.00 and the contents thereof for P70,000.00. FFF & P.E. 50, 000 24, 918. 79
PVC-15181 SSS
On July 31, 1975, the building and the contents were totally razed by fire. Adjustment Standard Accredited

Corporation submitted a report as follow Group of Building 25,000 5, 938.50


Insurers
xxx F-599 DV Insurers 1-Ref 30,000 14,467.31
Multi II-Building 70,000 16,628.00
x x x Thus the apportioned share of each company is as follows: Totals P295, 000 P90, 257. 81
Policy No. Company Risk Insures Pays
MIRO/ Zenith Building P50, 000 P 7,610.93 Based on the computation of the loss, including the Travellers Multi-Indemnity, respondents, Zenith
F-02500 Insurance Insurance, Phil. British Assurance and S.S.S. Accredited Group of Insurers, paid their corresponding shares
Corp.
of the loss. Complainants were paid the following: P41,546.79 by Philippine British Assurance Co.,
F-84590 Phil. Household 70,000 24,655.31
Page 28 of 97
P11,877.14 by Zenith Insurance Corporation, and P5,936.57 by S.S.S. Group of Accredited Insurers (Par. 6, As adverted to above respondent Insurance Commission dismissed spouses Palomos' complaint on the
Amended Complaint). Demand was made from respondent Travellers Multi-Indemnity for its share in the ground that the insurance policy subject of the complaint was taken out by Tai Tong Chuache &
loss but the same was refused. Hence, complainants demanded from the other three (3) respondents the Company, petitioner herein, for its own interest only as mortgagee of the insured property and thus

balance of each share in the loss based on the computation of the Adjustment Standards Report complainants as mortgagors of the insured property have no right of action against herein respondent. It
excluding Travellers Multi-Indemnity in the amount of P30,894.31 (P5,732.79 - Zenith Insurance: likewise dismissed petitioner's complaint in intervention in the following words:
P22,294.62, Phil. British: and P2,866.90, SSS Accredited) but the same was refused, hence, this action.
"We move on the issue of liability of respondent Travellers Multi-Indemnity to the Intervenor-mortgagee.
In their answers, Philippine British and Zenith Insurance Corporation admitted the material allegations in The complainant testified that she was still indebted to Intervenor in the amount of P100,000.00. Such

the complaint, but denied liability on the ground that the claim of the complainants had already been allegation has not however, been sufficiently proven by documentary evidence. The certification (Exhibit
waived, extinguished or paid. Both companies set up counterclaim in the total amount of P91,546.79. 'E-e') issued by the Court of First Instance of Davao, Branch 11, indicate that the complainant was Antonio
Lopez Chua and not Tai Tong Chuache & Company."[4]
Instead of filing an answer, SSS Accredited Group of Insurers informed the Commission in its letter of July
22, 1977 that the herein claim of complainants for the balance had been paid in the amount of P5,938.57 From the above decision, only intervenor Tai Tong Chuache filed a motion for reconsideration but it was

in full, based on the Adjustment Standards Corporation Report of September 22, 1975. likewise denied hence, the present petition.

Travellers Insurance, on its part, admitted the issuance of the Policy No. 599 DV and alleged as its special It is the contention of the petitioner that respondent Insurance Commission decided an issue not raised
and affirmative defenses the following, to wit: that Fire Policy No. 599 DV, covering the furniture and in the pleadings of the parties in that it ruled that a certain Arsenio Lopez Chua is the one entitled to the
building of complainants was secured by a certain Arsenio Chua, mortgage creditor, for the purpose of insurance proceeds and not Tai Tong Chuache & Company.

protecting his mortgage credit against the complainants; that the said policy was issued in the name of
This Court cannot fault petitioner for the above erroneous interpretation of the decision appealed from
Azucena Palomo, only to indicate that she owns the insured premises; that the policy contains an
considering the manner it was written.[5] As correctly pointed out by respondent insurance commission
endorsement in favor of Arsenio Chua as his mortgage interest may appear to indicate that insured was
in their comment, the decision did not pronounce that it was Arsenio Lopez Chua who has insurable
Arsenio Chua and the complainants; that the premiums due on said fire policy was paid by Arsenio Chua;
interest over the insured property. Perusal of the decision reveals however that it readily absolved
that respondent Travellers is not liable to pay complainants.
respondent insurance company from liability on the basis of the commissioner's conclusion that at the
On May 31, 1977, Tai Tong Chuache & Co. filed a complaint in intervention claiming the proceeds of the time of the occurrence of the peril insured against petitioner as mortgagee had no more insurable
fire Insurance Policy No. F-559 DV, issued by respondent Travellers Multi-Indemnity. interest over the insured property. It was based on the inference that the credit secured by the
mortgaged property was already paid by the Palomos before the said property was gutted down by fire.
Travellers Insurance, in answer to the complaint in intervention, alleged that the Intervenor is not entitled
The foregoing conclusion was arrived at on the basis of the certification issued by the then Court of First
to indemnity under its Fire Insurance Policy for lack of insurable interest before the loss of the insured
Instance of Davao, Branch II that in a certain civil action against the Palomos, Antonio Lopez Chua stands
premises and that the complainants, spouses Pedro and Azucena Palomo, had already paid in full their
as the complainant and not petitioner Tai Tong Chuache & Company.
mortgage indebtedness to the intervenor."[3]
Page 29 of 97
We find the petition to be impressed with merit. It is a well known postulate that the case of a party is The record of the case shows that the petitioner to support its claim for the insurance proceeds offered
constituted by his own affirmative allegations. Under Section 1, Rule 131[6] each party must prove his own as evidence the contract of mortgage (Exh. 1) which has not been cancelled nor released. It has been
affirmative allegations by the amount of evidence required by law which in civil cases as in the present held in a long line of cases that when the creditor is in possession of the document of credit, he need not

case is preponderance of evidence. The party, whether plaintiff or defendant, who asserts the affirmative prove non-payment for it is presumed.[8] The validity of the insurance policy taken by petitioner was not
of the issue has the burden of presenting at the trial such amount of evidence as required by law to assailed by private respondent. Moreover, petitioner's claim that the loan extended to the Palomos has
obtain a favorable judgment.[7] Thus, petitioner who is claiming a right over the insurance must prove its not yet been paid was corroborated by Azucena Palomo who testified that they are still indebted to
case. Likewise, respondent insurance company to avoid liability under the policy by setting up an herein petitioner.[9]
affirmative defense of lack of insurable interest on the part of the petitioner must prove its own
Public respondent argues however, that if the civil case really stemmed from the loan granted to
affirmative allegations.
Azucena Palomo by petitioner the same should have been brought by Tai Tong Chuache or by its
It will be recalled that respondent insurance company did not assail the validity of the insurance policy representative in its own behalf. From the above premise respondent concluded that the obligation
taken out by petitioner over the mortgaged property. Neither did it deny that the said property was secured by the insured property must have been paid.
totally razed by fire within the period covered by the insurance. Respondent, as mentioned earlier
The premise is correct but the conclusion is wrong. Citing Rule 3, Sec. 2[10] respondent pointed out that
advanced an affirmative defense of lack of insurable interest on the part of the petitioner alleging that
the action must be brought in the name of the real party in interest. We agree. However, it should be
before the occurrence of the peril insured against the Palomos had already paid their credit due the
borne in mind that petitioner being a partnership may sue and be sued in its name or by its duly
petitioner. Respondent having admitted the material allegations in the complaint, has the burden of
authorized representative. The fact that Arsenio Lopez Chua is the representative of petitioner is not
proof to show that petitioner has no insurable interest over the insured property at the time the
questioned. Petitioner's declaration that Arsenio Lopez Chua acts as the managing partner of the
contingency took place. Upon that point, there is a failure of proof. Respondent, it will be noted, exerted
partnership was corroborated by respondent insurance company.[11] Thus Chua as the managing partner
no effort to present any evidence to substantiate its claim, while petitioner did. For said respondent's
of the partnership may execute all acts of administration[12] including the right to sue debtors of the
failure, the decision must be adverse to it.
partnership in case of their failure to pay their obligations when it became due and demandable. Or at
However, as adverted to earlier, respondent Insurance Commission absolved respondent insurance the very least, Chua being a partner of petitioner Tai Tong Chuache & Company is an agent of the
company from liability on the basis of the certification issued by the then Court of First Instance of partnership. Being an agent, it is understood that he acted for and in behalf of the firm.[13] Public
Davao, Branch II, that in a certain civil action against the Palomos, Arsenio Lopez Chua stands as the respondent's allegation that the civil case filed by Arsenio Chua was in his capacity as personal creditor of
complainant and not Tai Tong Chuache. From said evidence respondent commission inferred that the spouses Palomo has no basis.

credit extended by herein petitioner to the Palomos secured by the insured property must have been
The respondent insurance company having issued a policy in favor of herein petitioner which policy was
paid. Such is a glaring error which this Court cannot sanction. Respondent Commission's findings are
of legal force and effect at the time of the fire, it is bound by its terms and conditions. Upon its failure to
based upon a mere inference.
prove the allegation of lack of insurable interest on the part of the petitioner, respondent insurance
company is and must be held liable.
Page 30 of 97
IN VIEW OF THE FOREGOING, the decision appealed from is hereby SET ASIDE and ANOTHER judgment On November 11, 1983, Dr. Wilfredo Leuterio, a physician and a housing debtor of DBP applied for

is rendered ordering private respondent Travellers Multi-Indemnity Corporation to pay petitioner the membership in the group life insurance plan. In an application form, Dr. Leuterio answered questions
concerning his health condition as follows:
face value of Insurance Policy No. 599-DV in the amount of P100,000.00. Costs against said private

respondent.
7. Have you ever had, or consulted, a physician for a heart condition, high blood pressure, cancer,
diabetes, lung, kidney or stomach disorder or any other physical impairment?
SO ORDERED

11. Great Pacific Life Assurance Corp V. CA (GRN 113899, 13 October 1999) Answer: No. If so give details ___________.

QUISUMBING, J.: 8. Are you now, to the best of your knowledge, in good health?

This petition for review, under Rule 45 of the Rules of Court, assails the Decision 1 dated May 17, 1993, of Answer: [ x ] Yes [ ] No.4

the Court of Appeals and its Resolution2 dated January 4, 1994 in CA-G.R. CV No. 18341. The appellate
On November 15, 1983, Grepalife issued Certificate No. B-18558, as insurance coverage of Dr. Leuterio, to
court affirmed in toto the judgment of the Misamis Oriental Regional Trial Court, Branch 18, in an
the extent of his DBP mortgage indebtedness amounting to eighty-six thousand, two hundred
insurance claim filed by private respondent against Great Pacific Life Assurance Co. The dispositive
(P86,200.00) pesos.
portion of the trial courts decision reads:

On August 6, 1984, Dr. Leuterio died due to massive cerebral hemorrhage. Consequently, DBP submitted
WHEREFORE, judgment is rendered adjudging the defendant GREAT PACIFIC LIFE ASSURANCE
a death claim to Grepalife. Grepalife denied the claim alleging that Dr. Leuterio was not physically healthy
CORPORATION as insurer under its Group policy No. G-1907, in relation to Certification B-18558 liable
when he applied for an insurance coverage on November 15, 1983. Grepalife insisted that Dr. Leuterio did
and ordered to pay to the DEVELOPMENT BANK OF THE PHILIPPINES as creditor of the insured Dr.
not disclose he had been suffering from hypertension, which caused his death. Allegedly, such non-
Wilfredo Leuterio, the amount of EIGHTY SIX THOUSAND TWO HUNDRED PESOS (P86,200.00);
disclosure constituted concealment that justified the denial of the claim.
dismissing the claims for damages, attorneys fees and litigation expenses in the complaint and
counterclaim, with costs against the defendant and dismissing the complaint in respect to the plaintiffs,
On October 20, 1986, the widow of the late Dr. Leuterio, respondent Medarda V. Leuterio, filed a
other than the widow-beneficiary, for lack of cause of action.3
complaint with the Regional Trial Court of Misamis Oriental, Branch 18, against Grepalife for Specific
Performance with Damages.5 During the trial, Dr. Hernando Mejia, who issued the death certificate, was
The facts, as found by the Court of Appeals, are as follows:
called to testify. Dr. Mejias findings, based partly from the information given by the respondent widow,

A contract of group life insurance was executed between petitioner Great Pacific Life Assurance stated that Dr. Leuterio complained of headaches presumably due to high blood pressure. The inference

Corporation (hereinafter Grepalife) and Development Bank of the Philippines (hereinafter DBP). Grepalife was not conclusive because Dr. Leuterio was not autopsied, hence, other causes were not ruled out.

agreed to insure the lives of eligible housing loan mortgagors of DBP.


On February 22, 1988, the trial court rendered a decision in favor of respondent widow and against
Grepalife. On May 17, 1993, the Court of Appeals sustained the trial courts decision. Hence, the present
petition. Petitioners interposed the following assigned errors:

Page 31 of 97
"1. THE LOWER COURT ERRED IN HOLDING DEFENDANT-APPELLANT LIABLE TO THE DEVELOPMENT Appeals affirmed the trial courts judgment, Grepalife was held liable to pay the proceeds of insurance
BANK OF THE PHILIPPINES (DBP) WHICH IS NOT A PARTY TO THE CASE FOR PAYMENT OF THE contract in favor of DBP, the indispensable party who was not joined in the suit.
PROCEEDS OF A MORTGAGE REDEMPTION INSURANCE ON THE LIFE OF PLAINTIFFS HUSBAND
WILFREDO LEUTERIO ONE OF ITS LOAN BORROWERS, INSTEAD OF DISMISSING THE CASE AGAINST To resolve the issue, we must consider the insurable interest in mortgaged properties and the parties to
DEFENDANT-APPELLANT [Petitioner Grepalife] FOR LACK OF CAUSE OF ACTION. this type of contract. The rationale of a group insurance policy of mortgagors, otherwise known as the
mortgage redemption insurance, is a device for the protection of both the mortgagee and the
2. THE LOWER COURT ERRED IN NOT DISMISSING THE CASE FOR WANT OF JURISDICTION OVER THE mortgagor. On the part of the mortgagee, it has to enter into such form of contract so that in the event
SUBJECT OR NATURE OF THE ACTION AND OVER THE PERSON OF THE DEFENDANT. of the unexpected demise of the mortgagor during the subsistence of the mortgage contract, the
proceeds from such insurance will be applied to the payment of the mortgage debt, thereby relieving the
3. THE LOWER COURT ERRED IN ORDERING DEFENDANT-APPELLANT TO PAY TO DBP THE AMOUNT heirs of the mortgagor from paying the obligation.7 In a similar vein, ample protection is given to the
OF P86,200.00 IN THE ABSENCE OF ANY EVIDENCE TO SHOW HOW MUCH WAS THE ACTUAL mortgagor under such a concept so that in the event of death; the mortgage obligation will be
AMOUNT PAYABLE TO DBP IN ACCORDANCE WITH ITS GROUP INSURANCE CONTRACT WITH extinguished by the application of the insurance proceeds to the mortgage indebtedness.8 Consequently,
DEFENDANT-APPELLANT. where the mortgagor pays the insurance premium under the group insurance policy, making the loss
payable to the mortgagee, the insurance is on the mortgagors interest, and the mortgagor continues to
4. THE LOWER COURT ERRED IN - HOLDING THAT THERE WAS NO CONCEALMENT OF MATERIAL be a party to the contract. In this type of policy insurance, the mortgagee is simply an appointee of the
INFORMATION ON THE PART OF WILFREDO LEUTERIO IN HIS APPLICATION FOR MEMBERSHIP IN THE insurance fund, such loss-payable clause does not make the mortgagee a party to the
GROUP LIFE INSURANCE PLAN BETWEEN DEFENDANT-APPELLANT OF THE INSURANCE CLAIM ARISING contract.9cräläwvirtualibräry
FROM THE DEATH OF WILFREDO LEUTERIO.6
Section 8 of the Insurance Code provides:
Synthesized below are the assigned errors for our resolution:
Unless the policy provides, where a mortgagor of property effects insurance in his own name providing
1. Whether the Court of Appeals erred in holding petitioner liable to DBP as beneficiary in a group life that the loss shall be payable to the mortgagee, or assigns a policy of insurance to a mortgagee, the
insurance contract from a complaint filed by the widow of the decedent/mortgagor? insurance is deemed to be upon the interest of the mortgagor, who does not cease to be a party to the
original contract, and any act of his, prior to the loss, which would otherwise avoid the insurance, will
2. Whether the Court of Appeals erred in not finding that Dr. Leuterio concealed that he had
have the same effect, although the property is in the hands of the mortgagee, but any act which, under
hypertension, which would vitiate the insurance contract?
the contract of insurance, is to be performed by the mortgagor, may be performed by the mortgagee
therein named, with the same effect as if it had been performed by the mortgagor.
3. Whether the Court of Appeals erred in holding Grepalife liable in the amount of eighty six thousand,
two hundred (P86,200.00) pesos without proof of the actual outstanding mortgage payable by the
The insured private respondent did not cede to the mortgagee all his rights or interests in the insurance,
mortgagor to DBP.
the policy stating that: In the event of the debtors death before his indebtedness with the Creditor [DBP]
shall have been fully paid, an amount to pay the outstanding indebtedness shall first be paid to the
Petitioner alleges that the complaint was instituted by the widow of Dr. Leuterio, not the real party in
creditor and the balance of sum assured, if there is any, shall then be paid to the beneficiary/ies
interest, hence the trial court acquired no jurisdiction over the case. It argues that when the Court of
designated by the debtor.10 When DBP submitted the insurance claim against petitioner, the latter denied

Page 32 of 97
payment thereof, interposing the defense of concealment committed by the insured. Thereafter, DBP On the contrary the medical findings were not conclusive because Dr. Mejia did not conduct an autopsy
collected the debt from the mortgagor and took the necessary action of foreclosure on the residential lot on the body of the decedent. As the attending physician, Dr. Mejia stated that he had no knowledge of
of private respondent.11 In Gonzales La O vs. Yek Tong Lin Fire & Marine Ins. Co.12 we held: Dr. Leuterios any previous hospital confinement.16 Dr. Leuterios death certificate stated that hypertension
was only the possible cause of death. The private respondents statement, as to the medical history of her
Insured, being the person with whom the contract was made, is primarily the proper person to bring suit husband, was due to her unreliable recollection of events. Hence, the statement of the physician was
thereon. * * * Subject to some exceptions, insured may thus sue, although the policy is taken wholly or in properly considered by the trial court as hearsay.
part for the benefit of another person named or unnamed, and although it is expressly made payable to
another as his interest may appear or otherwise. * * * Although a policy issued to a mortgagor is taken The question of whether there was concealment was aptly answered by the appellate court, thus:
out for the benefit of the mortgagee and is made payable to him, yet the mortgagor may sue thereon in
his own name, especially where the mortgagees interest is less than the full amount recoverable under The insured, Dr. Leuterio, had answered in his insurance application that he was in good health and that
the policy, * * *. he had not consulted a doctor or any of the enumerated ailments, including hypertension; when he died
the attending physician had certified in the death certificate that the former died of cerebral hemorrhage,
And in volume 33, page 82, of the same work, we read the following: probably secondary to hypertension. From this report, the appellant insurance company refused to pay
the insurance claim. Appellant alleged that the insured had concealed the fact that he had hypertension.
Insured may be regarded as the real party in interest, although he has assigned the policy for the
purpose of collection, or has assigned as collateral security any judgment he may obtain.13 Contrary to appellants allegations, there was no sufficient proof that the insured had suffered from
hypertension. Aside from the statement of the insureds widow who was not even sure if the medicines
And since a policy of insurance upon life or health may pass by transfer, will or succession to any person, taken by Dr. Leuterio were for hypertension, the appellant had not proven nor produced any witness who
whether he has an insurable interest or not, and such person may recover it whatever the insured might could attest to Dr. Leuterios medical history...
have recovered,14 the widow of the decedent Dr. Leuterio may file the suit against the insurer, Grepalife.
xxx
The second assigned error refers to an alleged concealment that the petitioner interposed as its defense
to annul the insurance contract. Petitioner contends that Dr. Leuterio failed to disclose that he had Appellant insurance company had failed to establish that there was concealment made by the insured,
hypertension, which might have caused his death. Concealment exists where the assured had knowledge hence, it cannot refuse payment of the claim.17
of a fact material to the risk, and honesty, good faith, and fair dealing requires that he should
communicate it to the assured, but he designedly and intentionally withholds the The fraudulent intent on the part of the insured must be established to entitle the insurer to rescind the
same.15cräläwvirtualibräry contract.18 Misrepresentation as a defense of the insurer to avoid liability is an affirmative defense and the
duty to establish such defense by satisfactory and convincing evidence rests upon the insurer.19 In the
Petitioner merely relied on the testimony of the attending physician, Dr. Hernando Mejia, as supported case at bar, the petitioner failed to clearly and satisfactorily establish its defense, and is therefore liable to
by the information given by the widow of the decedent. Grepalife asserts that Dr. Mejias technical pay the proceeds of the insurance.
diagnosis of the cause of death of Dr. Leuterio was a duly documented hospital record, and that the
widows declaration that her husband had possible hypertension several years ago should not be And that brings us to the last point in the review of the case at bar. Petitioner claims that there was no
considered as hearsay, but as part of res gestae. evidence as to the amount of Dr. Leuterios outstanding indebtedness to DBP at the time of the

Page 33 of 97
mortgagors death. Hence, for private respondents failure to establish the same, the action for specific QUIASON, J.:
performance should be dismissed. Petitioners claim is without merit. A life insurance policy is a valued
policy.20 Unless the interest of a person insured is susceptible of exact pecuniary measurement, the This is a petition for review for certiorari under Rule 45 of the Revised Rules of Court to reverse and set
measure of indemnity under a policy of insurance upon life or health is the sum fixed in the policy.21 The aside the Decision dated February 21, 1992 of the Court of Appeals in CA-G.R. CV No. 29068, and its
mortgagor paid the premium according to the coverage of his insurance, which states that: Resolution dated April 22, 1992, denying reconsideration thereof.

The policy states that upon receipt of due proof of the Debtors death during the terms of this insurance, We grant the petition.
a death benefit in the amount of P86,200.00 shall be paid.
I
In the event of the debtors death before his indebtedness with the creditor shall have been fully paid, an
amount to pay the outstanding indebtedness shall first be paid to the Creditor and the balance of the On April 15, 1986, Robert John B. Bacani procured a life insurance contract for himself from petitioner. He

Sum Assured, if there is any shall then be paid to the beneficiary/ies designated by the was issued Policy No. 3-903-766-X valued at P100,000.00, with double indemnity in case of accidental

debtor.22(Emphasis omitted) death. The designated beneficiary was his mother, respondent Bernarda Bacani.

However, we noted that the Court of Appeals decision was promulgated on May 17, 1993. In private On June 26, 1987, the insured died in a plane crash. Respondent Bernarda Bacani filed a claim with

respondents memorandum, she states that DBP foreclosed in 1995 their residential lot, in satisfaction of petitioner, seeking the benefits of the insurance policy taken by her son. Petitioner conducted an

mortgagors outstanding loan. Considering this supervening event, the insurance proceeds shall inure to investigation and its findings prompted it to reject the claim.

the benefit of the heirs of the deceased person or his beneficiaries. Equity dictates that DBP should not
In its letter, petitioner informed respondent Bernarda Bacani, that the insured did not disclose material
unjustly enrich itself at the expense of another (Nemo cum alterius detrimenio protest). Hence, it cannot
facts relevant to the issuance of the policy, thus rendering the contract of insurance voidable. A check
collect the insurance proceeds, after it already foreclosed on the mortgage. The proceeds now rightly
representing the total premiums paid in the amount of P10,172.00 was attached to said letter.
belong to Dr. Leuterios heirs represented by his widow, herein private respondent Medarda Leuterio.

Petitioner claimed that the insured gave false statements in his application when he answered the
WHEREFORE, the petition is hereby DENIED. The Decision and Resolution of the Court of Appeals in CA-
following questions:
G.R. CV 18341 is AFFIRMED with MODIFICATION that the petitioner is ORDERED to pay the insurance
proceeds amounting to Eighty-six thousand, two hundred (P86,200.00) pesos to the heirs of the insured,
5. Within the past 5 years have you:
Dr. Wilfredo Leuterio (deceased), upon presentation of proof of prior settlement of mortgagors
indebtedness to Development Bank of the Philippines. Costs against petitioner.
a) consulted any doctor or other health practitioner?

SO ORDERED.
b) submitted to:

12. Sunlife Assurance Company of Canada V. CA (245 SCRA 268, GRN 105135, 22 June
EGG?
1995)
X-rays?

Page 34 of 97
blood tests? Petitioner then moved for a summary judgment and the trial court decided in favor of private
other tests? respondents. The dispositive portion of the decision is reproduced as follows:

c) attended or been admitted to any hospital or other medical WHEREFORE, judgment is hereby rendered in favor of the plaintiffs and against the
facility? defendant, condemning the latter to pay the former the amount of One Hundred
Thousand Pesos (P100,000.00) the face value of insured's Insurance Policy No.
6. Have you ever had or sought advice for: 3903766, and the Accidental Death Benefit in the amount of One Hundred Thousand
Pesos (P100,000.00) and further sum of P5,000.00 in the concept of reasonable
xxx xxx xxx attorney's fees and costs of suit.

b) urine, kidney or bladder disorder? (Rollo, p. 53) Defendant's counterclaim is hereby Dismissed (Rollo, pp. 43-44).

The deceased answered question No. 5(a) in the affirmative but limited his answer to a consultation with In ruling for private respondents, the trial court concluded that the facts concealed by the insured were
a certain Dr. Reinaldo D. Raymundo of the Chinese General Hospital on February 1986, for cough and flu made in good faith and under a belief that they need not be disclosed. Moreover, it held that the health
complications. The other questions were answered in the negative (Rollo, p. 53). history of the insured was immaterial since the insurance policy was "non-medical".

Petitioner discovered that two weeks prior to his application for insurance, the insured was examined and Petitioner appealed to the Court of Appeals, which affirmed the decision of the trial court. The appellate
confined at the Lung Center of the Philippines, where he was diagnosed for renal failure. During his court ruled that petitioner cannot avoid its obligation by claiming concealment because the cause of
confinement, the deceased was subjected to urinalysis, ultra-sonography and hematology tests. death was unrelated to the facts concealed by the insured. It also sustained the finding of the trial court
that matters relating to the health history of the insured were irrelevant since petitioner waived the
On November 17, 1988, respondent Bernarda Bacani and her husband, respondent Rolando Bacani, filed
medical examination prior to the approval and issuance of the insurance policy. Moreover, the appellate
an action for specific performance against petitioner with the Regional Trial Court, Branch 191,
court agreed with the trial court that the policy was "non-medical" (Rollo, pp. 4-5).
Valenzuela, Metro Manila. Petitioner filed its answer with counterclaim and a list of exhibits consisting of
medical records furnished by the Lung Center of the Philippines. Petitioner's motion for reconsideration was denied; hence, this petition.

On January 14, 1990, private respondents filed a "Proposed Stipulation with Prayer for Summary II
Judgment" where they manifested that they "have no evidence to refute the documentary evidence of
concealment/misrepresentation by the decedent of his health condition (Rollo, p. 62). We reverse the decision of the Court of Appeals.

Petitioner filed its Request for Admissions relative to the authenticity and due execution of several The rule that factual findings of the lower court and the appellate court are binding on this Court is not
documents as well as allegations regarding the health of the insured. Private respondents failed to absolute and admits of exceptions, such as when the judgment is based on a misappreciation of the facts
oppose said request or reply thereto, thereby rendering an admission of the matters alleged. (Geronimo v. Court of Appeals, 224 SCRA 494 [1993]).

Page 35 of 97
In weighing the evidence presented, the trial court concluded that indeed there was concealment and The argument, that petitioner's waiver of the medical examination of the insured debunks the materiality
misrepresentation, however, the same was made in "good faith" and the facts concealed or of the facts concealed, is untenable. We reiterate our ruling in Saturnino v. Philippine American Life
misrepresented were irrelevant since the policy was "non-medical". We disagree. Insurance Company, 7 SCRA 316 (1963), that " . . . the waiver of a medical examination [in a non-medical
insurance contract] renders even more material the information required of the applicant concerning
Section 26 of The Insurance Code is explicit in requiring a party to a contract of insurance to previous condition of health and diseases suffered, for such information necessarily constitutes an
communicate to the other, in good faith, all facts within his knowledge which are material to the contract important factor which the insurer takes into consideration in deciding whether to issue the policy or not
and as to which he makes no warranty, and which the other has no means of ascertaining. Said Section ..."
provides:
Moreover, such argument of private respondents would make Section 27 of the Insurance Code, which
A neglect to communicate that which a party knows and ought to communicate, is allows the injured party to rescind a contract of insurance where there is concealment, ineffective
called concealment. (See Vda. de Canilang v. Court of Appeals, supra).

Materiality is to be determined not by the event, but solely by the probable and reasonable influence of Anent the finding that the facts concealed had no bearing to the cause of death of the insured, it is well
the facts upon the party to whom communication is due, in forming his estimate of the disadvantages of settled that the insured need not die of the disease he had failed to disclose to the insurer. It is sufficient
the proposed contract or in making his inquiries (The Insurance Code, Sec. 31). that his non-disclosure misled the insurer in forming his estimates of the risks of the proposed insurance
policy or in making inquiries (Henson v. The Philippine American Life Insurance Co., 56 O.G. No. 48
The terms of the contract are clear. The insured is specifically required to disclose to the insurer matters [1960]).
relating to his health.
We, therefore, rule that petitioner properly exercised its right to rescind the contract of insurance by
The information which the insured failed to disclose were material and relevant to the approval and reason of the concealment employed by the insured. It must be emphasized that rescission was exercised
issuance of the insurance policy. The matters concealed would have definitely affected petitioner's action within the two-year contestability period as recognized in Section 48 of The Insurance Code.
on his application, either by approving it with the corresponding adjustment for a higher premium or
rejecting the same. Moreover, a disclosure may have warranted a medical examination of the insured by WHEREFORE, the petition is GRANTED and the Decision of the Court of Appeals is REVERSED and SET
petitioner in order for it to reasonably assess the risk involved in accepting the application. ASIDE.

In Vda. de Canilang v. Court of Appeals, 223 SCRA 443 (1993), we held that materiality of the information SO ORDERED.
withheld does not depend on the state of mind of the insured. Neither does it depend on the actual or
physical events which ensue. 13. Vda de Canilang V. CA (223 SCRA 443, GRN 92492, 17 June 1993)

Thus, "goad faith" is no defense in concealment. The insured's failure to disclose the fact that he was
hospitalized for two weeks prior to filing his application for insurance, raises grave doubts about FELICIANO, J.:
his bonafides. It appears that such concealment was deliberate on his part.

Page 36 of 97
On 18 June 1982, Jaime Canilang consulted Dr. Wilfredo B. Claudio and was diagnosed as suffering from 1. the ailment of Jaime Canilang was not so serious that, even if it had been disclosed, it would not have
"sinus tachycardia." The doctor prescribed the following fro him: Trazepam, a tranquilizer; and Aptin, a affected Great Pacific's decision to insure him;
beta-blocker drug. Mr. Canilang consulted the same doctor again on 3 August 1982 and this time was
2. Great Pacific had waived its right to inquire into the health condition of the applicant by the issuance
found to have "acute bronchitis."
of the policy despite the lack of answers to "some of the pertinent questions" in the insurance
On next day, 4 August 1982, Jaime Canilang applied for a "non-medical" insurance policy with application;
respondent Great Pacific Life Assurance Company ("Great Pacific") naming his wife, Thelma Canilang, as
3. there was no intentional concealment on the part of the insured Jaime Canilang as he had thought
his beneficiary.1 Jaime Canilang was issued ordinary life insurance Policy No. 345163, with the face value
that he was merely suffering from a minor ailment and simple cold; 10 and
of P19,700, effective as of 9 August 1982.

4. Batas Pambansa Blg. 847 which voids an insurance contract, whether or not concealment was
On 5 August 1983, Jaime Canilang died of "congestive heart failure," "anemia," and "chronic anemia."2
intentionally made, was not applicable to Canilang's case as that law became effective only on 1 June
Petitioner, widow and beneficiary of the insured, filed a claim with Great Pacific which the insurer denied
1985.
on 5 December 1983 upon the ground that the insured had concealed material information from it.

On appeal by Great Pacific, the Court of Appeals reversed and set aside the decision of the Insurance
Petitioner then filed a complaint against Great Pacific with the Insurance Commission for recovery of the
Commissioner and dismissed Thelma Canilang's complaint and Great Pacific's counterclaim. The Court of
insurance proceeds. During the hearing called by the Insurance Commissioner, petitioner testified that
Appealed found that the use of the word "intentionally" by the Insurance Commissioner in defining and
she was not aware of any serious illness suffered by her late husband3 and that, as far as she knew, her
resolving the issue agreed upon by the parties at pre-trial before the Insurance Commissioner was not
husband had died because of a kidney disorder.4 A deposition given by Dr. Wilfredo Claudio was
supported by the evidence; that the issue agreed upon by the parties had been whether the deceased
presented by petitioner. There Dr. Claudio stated that he was the family physician of the deceased Jaime
insured, Jaime Canilang, made a material concealment as the state of his health at the time of the filing
Canilang5 and that he had previously treated him for "sinus tachycardia" and "acute bronchitis."6 Great
of insurance application, justifying respondent's denial of the claim. The Court of Appeals also found that
Pacific for its part presented Dr. Esperanza Quismorio, a physician
the failure of Jaime Canilang to disclose previous medical consultation and treatment constituted
and a medical underwriter working for Great Pacific.7 She testified that the deceased's insurance material information which should have been communicated to Great Pacific to enable the latter to make

application had been approved on the basis of his medical declaration.8 She explained that as a rule, proper inquiries. The Court of Appeals finally held that the Ng Gan Zee case which had involved
medical examinations are required only in cases where the applicant has indicated in his application for misrepresentation was not applicable in respect of the case at bar which involves concealment.
insurance coverage that he has previously undergone medical consultation and hospitalization.9
Petitioner Thelma Canilang is now before this Court on a Petition for Review on Certiorari alleging that:
In a decision dated 5 November 1985, Insurance Commissioner Armando Ansaldo ordered Great Pacific
1. . . . the Honorable Court of Appeals, speaking with due respect, erred in not holding that the issue in
to pay P19,700 plus legal interest and P2,000.00 as attorney's fees after holding that:
the case agreed upon between the parties before the Insurance Commission is whether or not Jaime
Canilang "intentionally" made material concealment in stating his state of health;

Page 37 of 97
2. . . . at any rate, the non-disclosure of certain facts about his previous health conditions does not Policy/TA/Certificate, provided that the first premium is paid and the Policy/TA/Certificate is delivered to,
amount to fraud and private respondent is deemed to have waived inquiry thereto. 11 and accepted by me in person, when I am in actual good health.

The medical declaration which was set out in the application for insurance executed by Jaime Canilang Signed at Manila his 4th day of August, 1992.
read as follows:
Illegible
MEDICAL DECLARATION
——————————
I hereby declare that:
Signature of Applicant. 12
(1) I have not been confined in any hospital, sanitarium or infirmary, nor receive any medical or surgical
We note that in addition to the negative statements made by Mr. Canilang in paragraph 1 and 2 of the
advice/attention within the last five (5) years.
medical declaration, he failed to disclose in the appropriate space, under the caption "Exceptions," that
(2) I have never been treated nor consulted a physician for a heart condition, high blood pressure, he had twice consulted Dr. Wilfredo B. Claudio who had found him to be suffering from "sinus
cancer, diabetes, lung, kidney, stomach disorder, or any other physical impairment. tachycardia" and "acute bronchitis."

(3) I am, to the best of my knowledge, in good health. The relevant statutory provisions as they stood at the time Great Pacific issued the contract of insurance
and at the time Jaime Canilang died, are set out in P.D. No. 1460, also known as the Insurance Code of
EXCEPTIONS:
1978, which went into effect on 11 June 1978. These provisions read as follows:

________________________________________________________________________________
Sec. 26. A neglect to communicate that which a party knows and ought to communicate, is called a
concealment.

xxx xxx xxx

GENERAL DECLARATION Sec. 28. Each party to a contract of insurance must communicate to the other, in good faith, all factors
within his knowledge which are material to the contract and as to which he makes no warranty, and
I hereby declare that all the foregoing answers and statements are complete, true and correct. I hereby
which the other has not the means of ascertaining. (Emphasis supplied)
agree that if there be any fraud or misrepresentation in the above statements material to the risk, the
INSURANCE COMPANY upon discovery within two (2) years from the effective date of insurance shall Under the foregoing provisions, the information concealed must be information which the concealing

have the right to declare such insurance null and void. That the liabilities of the Company under the said party knew and "ought to [have] communicate[d]," that is to say, information which was "material to the

Policy/TA/Certificate shall accrue and begin only from the date of commencement of risk stated in the contract." The test of materiality is contained in Section 31 of the Insurance Code of 1978 which reads:

Page 38 of 97
Sec. 31. Materially is to be determined not by the event, but solely by the probable and reasonable have been made, in assessing the risk involved in making or omitting to make further inquiries and in
influence of the facts upon the party to whom the communication is due, in forming his estimate of the accepting the application for insurance; that "probable and reasonable influence of the facts" concealed
disadvantages of the proposed contract, or in making his inquiries. (Emphasis supplied) must, of course, be determined objectively, by the judge ultimately.

"Sinus tachycardia" is considered present "when the heart rate exceeds 100 beats per minute." 13 The The insurance Great Pacific applied for was a "non-medical" insurance policy. In Saturnino v. Philippine-
symptoms of this condition include pounding in the chest and sometimes faintness and weakness of the American Life Insurance Company, 16 this Court held that:
person affected. The following elaboration was offered by Great Pacific and set out by the Court of
. . . if anything, the waiver of medical examination [in a non-medical insurance contract] renders even
Appeals in its Decision:
more material the information required of the applicant concerning previous condition of health and
Sinus tachycardia is defined as sinus-initiated; heart rate faster than 100 beats per minute. (Harrison' s diseases suffered, for such information necessarily constitutes an important factor which the insurer takes
Principles of Internal Medicine, 8th ed. [1978], p. 1193.) It is, among others, a common reaction to heart into consideration in deciding whether to issue the policy or not . . . . 17 (Emphasis supplied)
disease, including myocardial infarction, and heart failure per se. (Henry J.L. Marriot, M.D.,
The Insurance Commissioner had also ruled that the failure of Great Pacific to convey certain information
Electrocardiography, 6th ed., [1977], p. 127.) The medication prescribed by Dr. Claudio for treatment of
to the insurer was not "intentional" in nature, for the reason that Jaime Canilang believed that he was
Canilang's ailment on June 18, 1982, indicates the condition that said physician was trying to manage.
suffering from minor ailment like a common cold. Section 27 of the Insurance Code of 1978 as it existed
Thus, he prescribed Trazepam, (Philippine Index of Medical Specialties (PIMS), Vol. 14, No. 3, Dec. 1985, p.
from 1974 up to 1985, that is, throughout the time range material for present purposes, provided that:
112) which is anti-anxiety, anti-convulsant, muscle-relaxant; and Aptin, (Idem, p. 36) a cardiac drug, for
palpitations and nervous heart. Such treatment could have been a very material information to the Sec. 27. A concealment entitles the injured party to rescind a contract of insurance.
insurer in determining the action to be take on Canilang's application for life insurance coverage. 14
The preceding statute, Act No. 2427, as it stood from 1914 up to 1974, had provided:
We agree with the Court of Appeals that the information which Jaime Canilang failed to disclose was
Sec. 26. A concealment, whether intentional or unintentional, entitles the injured party to rescind a
material to the ability of Great Pacific to estimate the probable risk he presented as a subject of life
contract of insurance. (Emphasis supplied)
insurance. Had Canilang disclosed his visits to his doctor, the diagnosis made and medicines prescribed
by such doctor, in the insurance application, it may be reasonably assumed that Great Pacific would have Upon the other hand, in 1985, the Insurance Code of 1978 was amended by
made further inquiries and would have probably refused to issue a non-medical insurance policy or, at

the very least, required a higher premium for the same coverage. 15 The materiality of the information B.P. Blg. 874. This subsequent statute modified Section 27 of the Insurance Code of 1978 so as to read as

withheld by Great Pacific did not depend upon the state of mind of Jaime Canilang. A man's state of follows:

mind or subjective belief is not capable of proof in our judicial process, except through proof of external
Sec. 27. A concealment whether intentional or unintentional entitles the injured party to rescind a
acts or failure to act from which inferences as to his subjective belief may be reasonably drawn. Neither
contract of insurance. (Emphasis supplied)
does materiality depend upon the actual or physical events which ensue. Materiality relates rather to the
"probable and reasonable influence of the facts" upon the party to whom the communication should
Page 39 of 97
The unspoken theory of the Insurance Commissioner appears to have been that by deleting the phrase It remains only to note that the Court of Appeals finding that the parties had not agreed in the pretrial
"intentional or unintentional," the Insurance Code of 1978 (prior to its amendment by B.P. Blg. 874) before the Insurance Commission that the relevant issue was whether or not Jaime Canilang had
intended to limit the kinds of concealment which generate a right to rescind on the part of the injured intentionally concealed material information from the insurer, was supported by the evidence of record,

party to "intentional concealments." This argument is not persuasive. As a simple matter of grammar, it i.e., the Pre-trial Order itself dated 17 October 1984 and the Minutes of the Pre-trial Conference dated 15
may be noted that "intentional" and "unintentional" cancel each other out. The net result therefore of the October 1984, which "readily shows that the word "intentional" does not appear in the statement or
phrase "whether intentional or unitentional" is precisely to leave unqualified the term "concealment." definition of the issue in the said Order and Minutes." 18
Thus, Section 27 of the Insurance Code of 1978 is properly read as referring to "any concealment" without
WHEREFORE, the Petition for Review is DENIED for lack of merit and the Decision of the Court of Appeals
regard to whether such concealment is intentional or unintentional. The phrase "whether intentional or
dated 16 October 1989 in C.A.-G.R. SP No. 08696 is hereby AFFIRMED. No pronouncement as to the
unintentional" was in fact superfluous. The deletion of the phrase "whether intentional or unintentional"
costs.
could not have had the effect of imposing an affirmative requirement that a concealment must be
intentional if it is to entitle the injured party to rescind a contract of insurance. The restoration in 1985 by SO ORDERED.
B.P. Blg. 874 of the phrase "whether intentional or unintentional" merely underscored the fact that all
14. Insurance Life Assurance Co. V. Pineda (40 O.G. No 3, p285)
throughout (from 1914 to 1985), the statute did not require proof that concealment must be "intentional"
in order to authorize rescission by the injured party.

PARAS, J.:
In any case, in the case at bar, the nature of the facts not conveyed to the insurer was such that the
Challenged before Us in this petition for review on certiorari are the Orders of the respondent Judge
failure to communicate must have been intentional rather than merely inadvertent. For Jaime Canilang
dated March 19, 1980 and June 10, 1980 granting the prayer in the petition in Sp. Proc. No. 9210 and
could not have been unaware that his heart beat would at times rise to high and alarming levels and that
denying petitioner’s Motion for Reconsideration, respectively.
he had consulted a doctor twice in the two (2) months before applying for non-medical insurance.
Indeed, the last medical consultation took place just the day before the insurance application was filed. In The undisputed facts are as follows:chanrob1es virtual 1aw library

all probability, Jaime Canilang went to visit his doctor precisely because of the discomfort and concern
On January 15, 1968, private respondent procured an ordinary life insurance policy from the petitioner
brought about by his experiencing "sinus tachycardia."
company and designated his wife and children as irrevocable beneficiaries of said policy.

We find it difficult to take seriously the argument that Great Pacific had waived inquiry into the
Under date February 22, 1980 private respondent filed a petition which was docketed as Civil Case No.
concealment by issuing the insurance policy notwithstanding Canilang's failure to set out answers to
9210 of the then Court of First Instance of Rizal to amend the designation of the beneficiaries in his life
some of the questions in the insurance application. Such failure precisely constituted concealment on the
policy from irrevocable to revocable.
part of Canilang. Petitioner's argument, if accepted, would obviously erase Section 27 from the Insurance
Code of 1978. Petitioner, on March 10, 1980 filed an Urgent Motion to Reset Hearing. Also on the same date, petitioner

filed its Comment and/or Opposition to Petition.

Page 40 of 97
When the petition was called for hearing on March 19, 1980, the respondent Judge Gregorio G. Pineda, the beneficiaries is irrevocable (Annex "A" of Petition in Sp. Proc. No. 9210, Annex "C" of the Petition for
presiding Judge of the then Court of First Instance of Rizal, Pasig Branch XXI, denied petitioner’s Urgent Review on Certiorari), to wit:chanrob1es virtual 1aw library
Motion, thus allowing the private respondent to adduce evidence, the consequence of which was the
It is hereby understood and agreed that, notwithstanding the provisions of this policy to the contrary,
issuance of the questioned Order granting the petition.
inasmuch as the designation of the primary/contingent beneficiary/beneficiaries in this Policy has been
Petitioner promptly filed a Motion for Reconsideration but the same was denied in an Order June 10, made without reserving the right to change said beneficiary/beneficiaries, such designation may not be
1980. Hence, this petition raising the following issues for resolution:chanrob1es virtual 1aw library surrendered to the Company, released or assigned; and no right or privilege under the Policy may be
exercised, or agreement made with the Company to any change in or amendment to the Policy, without
I
the consent of the said beneficiary/beneficiaries. (Petitioner’s Memorandum, p. 72, Rollo)

WHETHER OR NOT THE DESIGNATION OF THE IRREVOCABLE BENEFICIARIES COULD BE CHANGED OR


Be it noted that the foregoing is a fact which the private respondent did not bother to disprove.
AMENDED WITHOUT THE CONSENT OF ALL THE IRREVOCABLE BENEFICIARIES.

Inevitably therefore, based on the aforequoted provision of the contract, not to mention the law then
II
applicable, it is only with the consent of all the beneficiaries that any change or amendment in the policy

WHETHER OR NOT THE IRREVOCABLE BENEFICIARIES HEREIN, ONE OF WHOM IS ALREADY DECEASED concerning the irrevocable beneficiaries may be legally and validly effected. Both the law and the policy

WHILE THE OTHERS ARE ALL MINORS, COULD VALIDLY GIVE CONSENT TO THE CHANGE OR do not provide for any other exception, thus, abrogating the contention of the private respondent that

AMENDMENT IN THE DESIGNATION OF THE IRREVOCABLE BENEFICIARIES. said designation can be amended if the Court finds a just, reasonable ground to do so.

Similarly, the alleged acquiescence of the six (6) children beneficiaries of the policy (the beneficiary-wife
predeceased the insured) cannot be considered an effective ratification to the change of the beneficiaries
We are of the opinion that his Honor, the respondent Judge, was in error in issuing the questioned
from irrevocable to revocable. Indubitable is the fact that all the six (6) children named as beneficiaries
Orders.
were minors at the time, ** for which reason, they could not validly give their consent. Neither could they

Needless to say, the applicable law in the instant case is the Insurance Act, otherwise known as Act No. act through their father-insured since their interests are quite divergent from one another. In point is an

2427 as amended, the policy having been procured in 1968. Under the said law, the beneficiary excerpt from the Notes and Cases on Insurance Law by Campos and Campos, 1960, reading —

designated in a life insurance contract cannot be changed without the consent of the beneficiary because
"The insured . . . can do nothing to divest the beneficiary of his rights without his consent. He cannot
he has a vested interest in the policy (Gercio v. Sun Life Ins. Co. of Canada, 48 Phil. 53; Go v. Redfern and
assign his policy, nor even take its cash surrender value without the consent of the beneficiary. Neither
the International Assurance Co., Ltd., 72 Phil. 71).
can the insured’s creditors seize the policy or any right thereunder. The insured may not even add

In this regard, it is worth noting that the Beneficiary Designation Indorsement in the policy which forms another beneficiary because by doing so, he diminishes the amount which the beneficiary may recover

part of Policy Number 0794461 in the name of Rodolfo Cailles Dimayuga states that the designation of and this he cannot do without the beneficiary’s consent."

Page 41 of 97
Therefore, the parent-insured cannot exercise rights and/or privileges pertaining to the insurance WHEREFORE, premises considered, the questioned Orders of the respondent Judge are hereby nullified
contract, for otherwise, the vested rights of the irrevocable beneficiaries would be rendered and set aside.
inconsequential.
SO ORDERED.
Of equal importance is the well-settled rule that the contract between the parties is the law binding on
15. Insular Life Association Co V. Feliciano (74 Phil 468)
both of them and for so many times, this court has consistently issued pronouncements upholding the
validity and effectivity of contracts. Where there is nothing in the contract which is contrary to law, good
morals, good customs, public policy or public order the validity of the contract must be sustained. SYLLABUS

Likewise, contracts which are the private laws of the contracting parties should be fulfilled according to
1. LIFE INSURANCE; VALIDITY OF POLICY CONTAINING FALSE STATEMENTS REGARDING HEALTH OF
the literal sense of their stipulations, if their terms are clear and leave no room for doubt as to the
THE INSURED. — The policies were issued on the basis of the statement subscribed by the applicant to
intention of the contracting parties, for contracts are obligatory, no matter in what form they may be,
the effect that he was and had been in good health, when as a matter of fact he was then suffering from
whenever the essential requisites for their validity are present (Phoenix Assurance Co., Ltd. v. United
advanced pulmonary tuberculosis. Held: Altho the agent and the medical examiner knew that statement
States Lines, 22 SCRA 675, Phil. American General Insurance Co., Inc. v. Mutuc, 61 SCRA 22.)
to be false, no valid contract of insurance was entered into because there was no real meeting of the

In the recent case of Francisco Herrera v. Petrophil Corporation, 146 SCRA 385, this Court ruled that:". . . minds of the parties.

it is settled that the parties may establish such stipulations, clauses, terms, and conditions as they may
2. ID.; ID. — When Evaristo Feliciano, the applicant for insurance, signed the application in blank and
want to include; and as long as such agreements are not contrary to law, good morals, good customs,
authorized the soliciting agent and/or the medical examiner of the Company to write the answers for
public policy or public order, they shall have the force of law between them."
him, he made them his own agents for that purpose, and he was responsible for their acts in that

Undeniably, the contract in the case at bar, contains the indispensable elements for its validity and does connection. If they falsified the answers for him, he could not evade the responsibility for the falsification.

not in any way violate the law, morals, customs, orders, etc. leaving no reason for Us to deny sanction He was not supposed to sign the application in blank. He knew that the answers to the questions therein

thereto. contained would be "the basis of the policy," and for that very reason he was required with his signature
to vouch for the truth thereof.
Finally, the fact that the contract of insurance does not contain a contingency when the change in the
designation of beneficiaries could be validly effected means that it was never within the contemplation of 3. ID.; ID.; CONNIVANCE WITH SOLICITING AGENT AND MEDICAL EXAMINER. — From all the facts and

the parties. The lower court, in gratuitously providing for such contingency, made a new contract for circumstances of the case, we are constrained to conclude that the insured was a coparticipant, and

them, a proceeding which we cannot tolerate. Ergo, We cannot help but conclude that the lower court coresponsible with Agent David and Medical Examiner Valdez, in the fraudulent procurement of the

acted in excess of its authority when it issued the Order dated March 19, 1980 amending the designation policies in question and that by reason thereof said policies are void ab initio.

of the beneficiaries from "irrevocable" to "revocable" over the disapprobation of the petitioner insurance

company.

Page 42 of 97
DECISION securing the Company’s approval of the application so that the policy to be issued thereon might be
credited to said agent in connection with the inter-provincial contest which the Company was then
holding among its soliciting agents to boost the sales of its policies. Agent David bribed Medical

OZAETA, J.: Examiner Valdez with money which the former borrowed from the applicant’s mother by way of
advanced payment on the premium, according to the finding of the Court of Appeals. Said court also
In a four-to-three decision promulgated on September 13, 1941, 1 this Court affirmed the judgment of the
found that before the insured signed the application he, as well as the members of his family, told the
Court of Appeals in favor of the respondents and against the petitioner for the sum of P25,000,
agent and the medical examiner that he had been sick and coughing for some time and that he had
representing the value of two insurance policies issued by the petitioner on the life of Evaristo Feliciano.
gone three times to the Santol Sanatorium and had X-ray pictures of his lungs taken; but that in spite of
A motion to reconsider and set aside said decision has been filed by the petitioner, and both parties have
such information the agent and the medical examiner told them that the applicant was a fit subject for
submitted exhaustive and luminous written arguments in support of their respective contentions.
insurance.

The facts of the case are set forth in the majority and dissenting opinions heretofore handed down by
Each of the policies sued upon contains the following stipulations:
this Court, the salient points of which may be briefly restated as follows:chanrob1es virtual 1aw library
"This policy and the application herefor constitute the entire contract between the parties hereto . . . Only
Evaristo Feliciano, who died on September 29, 1935, was suffering with advanced pulmonary tuberculosis
the President, or the Manager, acting jointly with the Secretary or Assistant Secretary (and then only in
when he signed his application for insurance with the petitioner on October 12, 1934. On that same date
writing signed by them) have power in behalf of the Company to issue permits, or to modify this or any
Doctor Trepp, who had taken X-ray pictures of his lungs, informed the respondent Dr. Serafin D.
contract, or to extend the time for making any premium payment, and the Company shall not be bound
Feliciano, brother of Evaristo, that the latter "was already in a very serious and practically hopeless
by any promise or representation heretofore or hereafter given by any person other than the above-
condition." Nevertheless the question contained in the application — "Have you ever suffered from any
named officials, and by them only in writing and signed conjointly as stated."
ailment or disease of the lungs, pleurisy, pneumonia or asthma?" — appears to have been answered,
"No." And above the signature of the applicant, following the answers to the various questions The application contains, among others, the following statements:

propounded to him, is the following printed statement:


"18. — I [the applicant] hereby declare that all the above statements and answers as well as all those that

I may make to the Company’s Medical Examiner in continuation of this application, to be complete, true
and correct to the best of my knowledge and belief, and I hereby agree as follows:
"I declare on behalf of myself and of any person who shall have or claim any interest in any policy issued
hereunder, that each of the above answers is full, complete and true, and that to the best of my "1. That this declaration, with the answers to be given by me to the Medical Examiner, shall be the basis

knowledge and belief I am a proper subject for life insurance." (Exhibit K.) of the policy and form part of same.

The false answer above referred to, as well as the others, was written by the Company’s soliciting agent "x x x

Romulo M. David, in collusion with the medical examiner Dr. Gregorio Valdez, for the purpose of

Page 43 of 97
"3. That the said policy shall not take effect until the first premium has been paid and the policy has been health." When the applicant signed the application he was "having difficulty in breathing, . . . with a very
delivered to and accepted by me, while I am in good health. high fever." He had gone three times to the Santol Sanatorium and had X-ray pictures taken of his lungs.
He therefore knew that he was not "a proper subject for life insurance." When he accepted the policy, he
"4. That the agent taking this application has no authority to make, modify or discharge contracts, or to
knew that he was not in good health. Nevertheless, he not only accepted the first policy of P20,000 but
waive any of the Company’s rights or requirements.
then and there applied for and later accepted another policy of P5,000.

"5. My acceptance of any policy issued on this application will constitute a ratification by me of any
We cannot bring ourselves to believe that the insured did not take the trouble to read the answers
corrections in or additions to this application made by the Company in the space provided ’For Home
contained in the photostatic copy of the application attached to and made a part of the policy before he
Office Corrections or Additions Only.’ I agree that photographic copy of this application as corrected or
accepted it and paid the premium thereon. He must have noticed that the answers to the questions
added to shall constitute sufficient notice to me of the changes made." (Emphasis added.)
therein asked concerning his clinical history were false, and yet he accepted the first policy and applied

The petitioner insists that upon the facts of the case the policies in question are null and void ab initio for another. In any event, he obligated himself to read the policy when he subscribed to this statement:

and that all that the respondents are entitled to is the refund of the premiums paid thereon. After a "My acceptance of any policy issued on this application will constitute a ratification by me of any

careful re-examination of the facts and the law, we are persuaded that petitioner’s contention is correct. corrections in or additions to this application made by the Company . . ." By accepting the policy he

To the reasons adduced in the dissenting opinion heretofore published, we only desire to add the became charged with knowledge of its contents, whether he actually read it or not. He could not ostrich-

following considerations:chanrob1es virtual 1aw library like hide his head from it in order to avoid his part of the bargain and at the same time claim the benefit
thereof. He knew, or was chargeable with knowledge, from the very terms of the two policies sued upon
When Evaristo Feliciano, the applicant for insurance, signed the application in blank and authorized the
(one of which is printed in English and the other in Spanish) that the soliciting agent and the medical
soliciting agent and/or the medical examiner of the Company to write the answers for him, he made
examiner had no power to bind the Company by any verbal promise or oral representation. The insured,
them his own agents for that purpose, and he was responsible for their acts in that connection. If they
therefore, had no right to rely — and we cannot believe he relied in good faith — upon the oral
falsified the answers for him, he could not evade the responsibility for the falsification. He was not
representation of said agent and medical examiner that he (the applicant) was a fit subject for insurance
supposed to sign the application in blank. He knew that the answers to the questions therein contained
notwithstanding that he had been and was still suffering with advanced pulmonary tuberculosis.
would be "the basis of the policy," and for that very reason he was required with his signature to vouch

for the truth thereof. From all the facts and circumstances of this case, we are constrained to conclude that the insured was a
coparticipant, and coresponsible with Agent David and Medical Examiner Valdez, in the fraudulent
Moreover, from the facts of the case we cannot escape the conclusion that the insured acted in
procurement of the policies in question and that by reason thereof said policies are void ab initio.
connivance with the soliciting agent and the medical examiner of the Company in accepting the policies
in question. Above the signature of the applicant is the printed statement or representation: ". . . I am a Wherefore, the motion for reconsideration is sustained and the judgment of the Court of Appeals is

proper subject for life insurance." In another sheet of the same application and above another signature hereby reversed. Let another judgment be entered in favor of the respondents and against the petitioner

of the applicant was also printed this statement: "That the said policy shall not take effect until the first for the refund of the premiums amounting to P1,389, with legal interest thereon from the date of the

premium has been paid and the policy has been delivered to and accepted by me, while I am in good complaint, and without any finding as to costs.
Page 44 of 97
Moran, Paras, and Bocobo, JJ., concur. am of the opinion that in justice and in equity, the responsibility for the falsifications made by the
insurance agents in the preparation of the insurance application should be laid at the door of the assured
and his beneficiaries.

Separate Opinions
I vote with the majority in granting the motion for reconsideration and in reversing the decision under

YULO, C.J., concurring:chanrob1es virtual 1aw library review.

I can find no quarrel with the legal considerations and conclusions set forth in the original decision HONTIVEROS, J., dissenting:chanrob1es virtual 1aw library

promulgated by this Court. As general rules of law they find full support not only in reason and in logic,
The reasons given in the dissenting opinion in this case, as published in the Official Gazette of October 4,
but also in simple human sense of justice. More so, modern and complicated practices attendant to the
1941 (pp. 2847 to 2855), supplemented by those in the resolution of the majority on the motion for
ever growing trade in life insurance demand the strictest accountability by insurance companies for acts
reconsideration, do not seem to me sufficient to overthrow the decision rendered by the Court of First
of their authorized agents. In this way only may the State afford reasonable protection to the unwary
Instance, confirmed by the Court of Appeals, and sustained by this Supreme Court in its decision of
public from abuse by such organizations as may be found to be of questionable moral standards.
September 18, 1941. The alleged connivance between the insured Evaristo Feliciano, the agent Romulo M.

But a careful consideration of the evidentiary facts as set forth in the decision of the Court of Appeals David, and the medical examiner Dr. Gregorio Valdez not only does not clearly appear of record, but on

leads me to conclude that the ends of justice would not be served by the application to the present case the contrary is denied in the finding of facts of the court a quo and of the Court of Appeals which cannot

of the rules so enunciated. Rather, to serve the ends of justice the case of the respondents should be be reviewed or altered by this Court.

removed from the protection of such rules.


The mere fact that the insured signed at the bottom of the application for insurance when some of its

The subject of the insurance policies under consideration is the life of the assured. It is contended by his lines intended for answers to certain questions were still in blank, answers which according to the

beneficiaries that they took these policies on the basis of a life expectancy of a person gravely stricken evidence and to the findings of the two inferior courts he had grounds to believe will be made in

with tuberculosis. They have consistently made protestations that they had so informed the agents of the accordance with the information which he and his family had given to agent David and to Dr. Valdez,

insurance company. But the policies were issued upon the life of the assured, as a perfectly normal and does not convert these two persons into agents of the insured in a way as to make the latter responsible

healthy person. The error is vital and goes to the very existence of the contract itself. Who is responsible for the acts of the former. That the photostatic copies of said forms which are attached to the policies

for the error? object of this case are almost illegible, is a fact which should be taken into account, together with the
other fact that Evaristo Feliciano does not know English, the language in which those documents are
The direct cause, of course, is the false recitals in the application for insurance. While it is true that it was
written. In support of this dissenting opinion, the following authorities may be cited:
the agents of the insurance company who filled out such application, yet it was the assured who, by
signing the application in blank, made it possible for the said agents to procure the issuance of the "The mere failure of the insured to inform himself of the insertion of false answers in the application

policies on the basis of false information, in order to suit their own purposes. Upon the admitted facts, I which has been filled out by the agent of the insurer does not convict him of lack of good faith." (Vol. 5,
Cooley’s Briefs on Insurance, 2nd Ed., p. 4136, and many cases cited.)
Page 45 of 97
"The insured is not chargeable with such negligence as will render him liable for false answers inserted by "And also where the photostatic copies of the application embodied in the policy are practically illegible,
the agent merely because he signed the application in blank and trusted the agent to fill out by the the insured is not bound to know the contents of the application." (New York Ins. Co. v. Holpem D. C., 57
agent, without reading it." (Id., p. 4136, and many cases cited.) Fed. 2d, 200).

"An illiterate person or one who does not understand the English language (as is the case with Evaristo "According to the great weight of authority, if an agent of the insurer, after obtaining from an applicant
Feliciano) is not guilty of inexcusable negligence in failing to read the application or having it read to him, for insurance a correct and truthful answer to interrogations contained in the application for insurance,
nor can it be said that such person deliberately made a false statement because he did not read over the without knowledge of the applicant fills in false answers, either fraudulently or otherwise, the insurer
application." (81 ALR 865, 866, W. 117 ALR 796.) cannot assert the falsity of such answers as a defense to the liability on the policy and this is generally

without regard to the subject matter of the answers or the nature of the agent’s duties or limitations on
"Nor can it be said that the assured, who has fully, frankly, truthfully, and in good faith answered all the
his authority, at least if not brought to the attention of the applicant. It is equally well settled that if a
required questions, is guilty of negligence in signing, without reading, the application which is thereupon
correct representation is made in a written application, or the insurance agent issuing the policy is
prepared by the agent. He is justified in assuming that the agent, has, with equal good faith, truthfully
appraised of the true facts concerning the matter in question, as for instance the title to the insured
recorded the answers given. He may well say to the Company: ’You accredited this man to me as your
premises, but the agent inserts an incorrect statement in the policy, the insurer cannot rely upon the
representative, and I signed the application thus prepared by him, relying upon the character which you
error in avoidance of its liability." Home Ins. Co. v. Mendenhall, 154 Ill., 452, 45 NE., 1078, 36 LRA., 374;
gave him, when you commissioned him to come to me as your agent. If he acted dishonestly in the
Phoenix Ins. Co. v. Tucker, 92 Ill., 64, 34 Am Rep., 106; Commercial Ins. Co. v. Spanknoble, 52 Ill., 53, 4
matter, you, and not I, must suffer the consequences . . . !’ (Germania Life Ins. Co. v. Lunkeheimer [1931]
Am. Report, 582; Young v. Hartford F. Ins. Co. 45 Iowa, 377, 24 Am. Rep., 754; Welsh v. London Assur. 151
Ind., 538; 26 N. E., 1052.)
Pa., 607, 25 A, 142, 21 Am St. Rep., 726 — (Taken from Am Juris. on Insurance Vol. 29, par. 843).

"In such case the acceptance of the policy, with this application attached, does not require the insured to
"An insured may be justified in signing an application in blank at the request of the insurer’s agent, who
institute an investigation into its provisions, or the conditions upon which it was issued, to ascertain
agrees to fill it in from data furnished by the insured or from an old application. In fact, an insurer cannot
whether the agent has acted in good faith, since, under such circumstances, the insured may rely upon
urge the falsity of representations contained in the policy issued, or in the application, where such
the presumption that he has been honestly dealt with by the insurer." (Otto v. Hartford Ins. Co., 38 Minn.,
representations were inserted therein, either by the company or its agent, after the application was
423).
signed, without the knowledge or consent of the insured, who has made no such representations."

"Besides, the principles that the insured is not bound to know the contents of the application, and may (Couch on Insurance, Vol. 4, par. 842 b.)

rely on the agent’s assurances that his answers have been correctly written will, of course, apply with
I believe that the motion for reconsideration presented in this case should be denied, not only because
special force where the insured is illiterate and unable to read, or is ignorant of the language." (Vol. 5,
of the weighty reasons relied upon in the decision which it attacks, but also because a dangerous
Cooley’s Briefs on Insurance, 2nd Ed. p. 4138, cases cited.)
precedent would otherwise be established, for, with the destruction of the confidence which the public
has hitherto reposed in the duly accredited agents of insurance companies and in their examining
physicians, this branch of the economic life of the people will have to be unfavorably affected.
Page 46 of 97
16. Saturnino V. Philippine American Life Insurance Co. (7 SCRA 316, GRN L-16163, 28 Feb. any physician, undergone any operation or suffered any injury within the preceding five years; and that
1963) she had never been treated for nor did she ever have any illness or disease peculiar to her sex,
particularly of the breast, ovaries, uterus, and menstrual disorders. The application also recites that the
MAKALINTAL, J.: foregoing declarations constituted "a further basis for the issuance of the policy."

Plaintiffs, now appellants, filed this action in the Court of First Instance of Manila to recover the sum of The question at issue is whether or not the insured made such false representations of material facts as
P5,000.00, corresponding to the face value of an insurance policy issued by defendant on the life of to avoid the policy. There can be no dispute that the information given by her in her application for
Estefania A. Saturnino, and the sum of P1,500.00 as attorney's fees. Defendant, now appellee, set up insurance was false, namely, that she had never had cancer or tumors, or consulted any physician or
special defenses in its answer, with a counterclaim for damages allegedly sustained as a result of the undergone any operation within the preceding period of five years. Are the facts then falsely represented
unwarranted presentation of this case. Both the complaint and the counterclaim were dismissed by the material? The Insurance Law (Section 30) provides that "materiality is to be determined not by the event,
trial court; but appellants were declared entitled to the return of the premium already paid; plus interest but solely by the probable and reasonable influence of the facts upon the party to whom the
at 6% up to January 8, 1959, when a check for the corresponding amount — P359.65 — was sent to communication is due, in forming his estimate of the proposed contract, or in making his inquiries." It
them by appellee. seems to be the contention of appellants that the facts subject of the representation were not material in
view of the "non-medical" nature of the insurance applied for, which does away with the usual
The policy sued upon is one for 20-year endowment non-medical insurance. This kind of policy requirement of medical examination before the policy is issued. The contention is without merit. If
dispenses with the medical examination of the applicant usually required in ordinary life policies. anything, the waiver of medical examination renders even more material the information required of the
However, detailed information is called for in the application concerning the applicant's health and applicant concerning previous condition of health and diseases suffered, for such information necessarily
medical history. The written application in this case was submitted by Saturnino to appellee on constitutes an important factor which the insurer takes into consideration in deciding whether to issue
November 16, 1957, witnessed by appellee's agent Edward A. Santos. The policy was issued on the same the policy or not. It is logical to assume that if appellee had been properly apprised of the insured's
day, upon payment of the first year's premium of P339.25. On September 19, 1958 Saturnino died of medical history she would at least have been made to undergo medical examination in order to
pneumonia, secondary to influenza. Appellants here, who are her surviving husband and minor child, determine her insurability.
respectively, demanded payment of the face value of the policy. The claim was rejected and this suit was
subsequently instituted. Appellants argue that due information concerning the insured's previous illness and operation had been
given to appellees agent Edward A. Santos, who filled the application form after it was signed in blank by
It appears that two months prior to the issuance of the policy or on September 9, 1957, Saturnino was Estefania A. Saturnino. This was denied by Santos in his testimony, and the trial court found such
operated on for cancer, involving complete removal of the right breast, including the pectoral muscles testimony to be true. This is a finding of fact which is binding upon us, this appeal having been taken
and the glands found in the right armpit. She stayed in the hospital for a period of eight days, after which upon questions of law alone. We do not deem it necessary, therefore, to consider appellee's additional
she was discharged, although according to the surgeon who operated on her she could not be argument, which was upheld by the trial court, that in signing the application form in blank and leaving it
considered definitely cured, her ailment being of the malignant type. to Edward A. Santos to fill (assuming that to be the truth) the insured in effect made Santos her agent for
that purpose and consequently was responsible for the errors in the entries made by him in that capacity.
Notwithstanding the fact of her operation Estefania A. Saturnino did not make a disclosure thereof in her
application for insurance. On the contrary, she stated therein that she did not have, nor had she ever In the application for insurance signed by the insured in this case, she agreed to submit to a medical
had, among other ailments listed in the application, cancer or other tumors; that she had not consulted examination by a duly appointed examiner of appellee if in the latter's opinion such examination was

Page 47 of 97
necessary as further evidence of insurability. In not asking her to submit to a medical examination, The insurer, relying upon the belief that the assured will disclose every material fact within his
appellants maintain, appellee was guilty of negligence, which precluded it from finding about her actual actual or presumed knowledge, is misled into a belief that the circumstance withheld does not
state of health. No such negligence can be imputed to appellee. It was precisely because the insured had exist, and he is thereby induced to estimate the risk upon a false basis that it does not exist."
given herself a clean bill of health that appellee no longer considered an actual medical checkup
necessary. The judgment appealed from, dismissing the complaint and awarding the return to appellants of the
premium already paid, with interest at 6% up to January 29, 1959, affirmed, with costs against appellants.
Appellants also contend there was no fraudulent concealment of the truth inasmuch as the insured
herself did not know, since her doctor never told her, that the disease for which she had been operated
on was cancer. In the first place the concealment of the fact of the operation itself was fraudulent, as
there could not have been any mistake about it, no matter what the ailment. Secondly, in order to avoid 17. Tan Chay Heng V. West Coast Life Insurance Co. (51 Phil 80, GRN L-27541, 21 Nov.

a policy it is not necessary to show actual fraud on the part of the insured. In the case of Kasprzyk v. 1927)
Metropolitan Insurance Co., 140 N.Y.S. 211, 214, it was held:

Moreover, if it were the law that an insurance company could not depend a policy on the STATEMENT
ground of misrepresentation, unless it could show actual knowledge on the part of the applicant
that the statements were false, then it is plain that it would be impossible for it to protect itself Plaintiff alleges that he is of age and a resident of Bacolod, Occidental Negros; that the defendant is a
and its honest policyholders against fraudulent and improper claims. It would be wholly at the foreign insurance corporation duly organized by the laws of the Philippines to engage in the insurance
mercy of any one who wished to apply for insurance, as it would be impossible to show actual business, its main office of which is in the City of Manila; that in the month of April, 1925, on his

fraud except in the extremest cases. It could not rely on an application as containing application the defendant accepted and approved a life insurance policy of for the sum of P10,000 in
information on which it could act. There would be no incentive to an applicant to tell the truth. which the plaintiff was the sole beneficiary; that the policy was issued upon the payment by the said Tan
Ceang of the first year's premium amounting to P936; that in and by its terms, the defendant agreed to
Wherefore, the parties respectfully pray that the foregoing stipulation of facts be admitted and pay the plaintiff as beneficiary the amount of the policy upon the receipt of the proofs of the death of
approved by this Honorable Court, without prejudice to the parties adducing other evidence to the insured while the policy was in force; that without any premium being due or unpaid, Tan Ceang died
prove their case not covered by this stipulation of facts. 1äwphï1.ñët on May 10, 1925; that in June, 1925, plaintiff submitted the proofs of the death of Tan Ceang with a claim
for the payment of the policy which the defendant refused to pay, for which he prays for a corresponding
In this jurisdiction a concealment, whether intentional or unintentional, entitles the insurer to rescind the judgment, with legal interest from the date of the policy, and costs.
contract of insurance, concealment being defined as "negligence to communicate that which a party
knows and ought to communicate" (Sections 24 & 26, Act No. 2427). In the case of Argente v. West In February, 1926, the defendant filed an answer to the complaint in which it made a general and specific
Coast Life Insurance Co., 51 Phil. 725, 732, this Court said, quoting from Joyce, The Law of Insurance, 2nd denial, and then announced its intention to file an amended answer, alleging special defense, and on
ed., Vol. 3: August 31, 1926, it filed the following:

"The basis of the rule vitiating the contract in cases of concealment is that it misleads or AMENDED ANSWER
deceives the insurer into accepting the risk, or accepting it at the rate of premium agreed upon.

Page 48 of 97
Comes now the defendant, by its undersigned attorneys, and with leave of court amends its answer to 2. That on said date, February 22, 1925, the said Tan Ceang was seriously ill, suffering from pulmonary
plaintiff's complaint herein by making it reads as follows: tuberculosis of about three years' duration, which illness was incurable and was well known to the
plaintiff and his said coconspirators.
I
3. That on or about the same date, February 22, 1925, the said Dr. V. S. Locsin, in his capacity as medical
That it admits paragraph 1 of said complaint. examiner for the defendant insurance company, pursuant to the conspiracy above mentioned, prepared
and falsified the necessary medical certificate, in which it was made to appear, among other things, that
II the said Tan Ceang had never used morphine, cocaine or any other drug; that he was then in good
health and had never consulted any physician; that he had never spit blood; and that there was no sign
That it denies each and every other allegation contained in each and every other paragraph of said
of either present or past disease of his lungs; whereas in truth and in fact, as the plaintiff and his said
complaint.
coconspirators well knew, the said Tan Ceang was addicted to morphine, cocaine, and opium and had
been convicted and imprisoned therefor, and was then, and for about three year prior thereto had been
SPECIAL DEFENSE
suffering from pulmonary tuberculosis.

By way of special defense, defendant alleges:


4. That on or about the same date, to wit, February 22, 1925, the plaintiff and his said coconspirators,
pursuant to the conspiracy above mentioned, cause a confidential report to the defendant insurance
I
company to be signed by one V. Sy Yock Kian, who was an employee of Go Chulian, in which confidential
report, among other things, it was falsely represented to the defendant insurance company that the said
That the insurance policy on the life of Tan Ceang, upon which plaintiff's action is based, was obtained by
Tan Ceang was worth about P40,000, had an annual income of from eight to ten thousand pesos net,
the plaintiff in confabulation with one Go Chulian, of Bacolod, Negros Occidental; Francisco Sanchez of
had the appearance of good health, and never had tuberculosis; that the plaintiff and his said
the same place; and Dr. V. S. Locsin, of La Carlota, Negros Occidental, thru fraud and deceit perpetrated
coconspirators well knew that said representations were false; and that they were made for the purpose
against this defendant in the following manner, to wit:
of deceiving the defendant and inducing it to accept the said application for insurance.

1. That on or about the 22d day of February, 1925, in the municipality of Pulupandan, Occidental Negros,
5. That after the said application for insurance, medical certificate and confidential report had been
the present plaintiff and the said Go Chulian, Francisco Sanchez and Dr., V. S. Locsin, conspiring and
prepared and falsified, as aforesaid, the plaintiff and his said coconspirators caused the same to be
confederating together for the purpose of defrauding and cheating the defendant in the sum of P10,000,
forwarded to the defendant at its office in Manila, the medical certificate thru the said Dr. V. S. Locsin as
caused one Tan Caeng to sign an application for insurance with the defendant in the sum of P10,000, in
medical examiner, and said application for insurance and confidential report thru the said Francisco
which application it was falsely represented to the defendant that the said Tan Ceang was single and was
Sanchez in his capacity as one of the agents of the defendant insurance company in the Province of
a merchant, and that the plaintiff Tan Chai Heng, the beneficiary, was his nephew, whereas in truth and in
Occidental Negros; that the defendant, believing that the representations made in said document were
fact and as the plaintiff and his said coconspirators well knew, the said Tan Ceang was not single but was
true, and relying thereon, provisionally accepted the said application for insurance on the life of Tan
legally married to Marcelina Patalita with whom he had several children; and that he was not a merchant
Ceang in the sum of P10,000 and issued a temporary policy pending the final approval or disapproval of
but was a mere employee of another Chinaman by the name of Tan Quina from whom he received only
said application by defendant's home-office in San Francisco, California, where in case of approval a
a meager salary, and that the present plaintiff was not a nephew of the said Tan Ceang.

Page 49 of 97
permanent policy was to be issued; that such permanent policy was never delivered to the plaintiff Witness: (Sgd.) TAN CHAI HENG
because defendant discovered the fraud before its delivery. TAN CAENG
Signature of Applicant.
6. That the first agreed annual premium on the insurance in question of P936.50 not having been paid
within sixty (60) days after the date of the supposed medical examination of the applicant as required by "Dated at Palupandan on this 10 day of April, 1925." that the statements and representations contained in
the regulations of the defendant insurance company, of which regulations the said Francisco Sanchez as the application for reinstatement above set forth with regard to the health and physical condition of the
agent of the defendant had knowledge, the plaintiff and his said coconspirators in order to secure the said Tan Ceang were false and known to the plaintiff and his said coconspirators to be false; that the said
delivery to them of said temporary policy, and in accordance with said regulations of the defendant temporary policy was delivered by defendant to the insured on April 10, 1925, in the belief that said
company, caused the said Tan Ceang on April 10, 1925 to sign the following document: statements and representations were true and in reliance thereon.

WEST COAST LIFE INSURANCE COMPANY


SAN FRANCISCO, CALIFORNIA
7. That on May 10, 1925, that is to say, two months and a half after the supposed medical examination
HEALTH CERTIFICATE FOR RE-INSTATEMENT above referred to, and exactly one month after the date of the health certificate for reinstatement above
set forth, the said Tan Ceang died in Valladolid, Occidental Negros, of pulmonary tuberculosis, the same
I herewith request the West Coast Life Insurance Company to re-instate Policy No. ............................. issued illness from which suffering at the time it is supposed he was examined by Dr. V. S. Locsin, but that the
by it upon my life, the first unpaid premium on which became due .............................., 19................ plaintiff and his said coconspirators, pursuant to their conspiracy, caused the said Dr. V. S. Locsin to state
falsely in the certificate of death that the said Tan Ceang had died of cerebral hemorrhage.
I certify and state that I am now in good and sound health, that since the date of my examination under
the application on which said policy was written, I have had no injury, sickness, impairment of health or II
symptom thereof, and that since said date I have neither consulted a physician nor made any application
for life insurance that has not been granted in exact kind and amount applied for, except: That the plaintiff Tan Chai Heng, on the dates herein-above mentioned, was, liked V. Sy Yock Kian who
signed the confidential report above mentioned, an employee of the said Go Chulian; that the latter was
NADA the ringleader of a gang of malefactors, who, during, and for some years previous to the dates above
mentioned, were engaged in the illicit enterprise of procuring fraudulent life insurances from the present
(State fully all exceptions to all above statements. If no exceptions insert "NONE.") defendant, similar to the one in question, and which enterprise was capitalized by him by furnishing the
funds with which to pay the premium on said fraudulent insurance; that the said Go Chulian was the one
I agree that, if said policy re-instated, it shall be only on condition of the truth of the above statements
who furnished the money with which to pay the first and only annual premium on the insurance here in
and such re-instatement shall not operate as a waiver on the part of said Company of its right to refuse
question, amounting to P936.50; that the said Go Chulian, on August 28, 1926, was convicted by the
to accept any future overdue premiums or installments thereof.
Court of First Instance of the City of Manila, in criminal case No. 31425 of that court, of the crime of
falsification of private documents in connection with an fraudulent insurance, similar to the present,
committed against this defendant in the month of September, 1924; that in the same case the said
Francisco Sanchez was one of the coaccused of the said Go Chulian but was discharged from the

Page 50 of 97
complaint, because he offered himself and was utilized as a state's witness; that there is another civil 1. In sustaining plaintiff's demurrer to the special defense contained in defendant's amended answer.
action now pending against Go Chulian and Sanchez in the Court of First Instance of Manila (civil case
No. 28680), in which the present defendant is the plaintiff, for the recovery of the amounts of two 2. In holding, in effect, that an insurer cannot avoid a policy which had been procured by fraud unless he
insurance policies aggregating P19,000, fraudulently obtained by the said Go Chulian and Sanchez upon brings an action to rescind it before he is sued thereon.
the lives of one Tan Deco, who was also suffering from and died of tuberculosis, and one Tan Anso, who
was suffering from and died of beriberi. 3. In rejecting all proofs offered by the defendant during the trial for the purpose of defeating plaintiff's
fraudulent claim.
III
4. In not absolving the defendant from plaintiff's complaint.
That by reason of all the facts above set forth, the temporary policy of insurance on the life of Tan Caeng
for the sum of P10,000 upon which the present action is base is null and void. JOHNS, J.:

Wherefore, defendant prays that it be absolved from plaintiff's complaint, with costs against the plaintiff. It will thus be noted that the premium was paid on April 10, 1925, at which time the temporary policy was
issued; that the plaintiff's action was commenced on January 4, 1926; that the original answer of the
To this special defense, the plaintiff, claiming that it was a cross-complaint, filed a general demurrer upon defendant, consisting of a general and specific denial, was filed on February 27, 1926; and that its
the ground that it does not state facts sufficient to constitute a cause of defense. amended answer was filed on August 31, 1926.

After exhaustive arguments and on September 16, 1926, the court rendered the following decision: Based upon those facts the plaintiff vigorously contended in the lower court and now contends in the
court, that section 47 of the Insurance Act should be applied, and that when so applied, defendant is
After considering the demurrer filed by the plaintiff to the special defense contained in the amended barred and estopped to plead and set forth the matters alleged in its special defense. That section is as
answer of the defendant, dated August 31, 1926, without prejudice to writing a more extensive decision, follows:
said demurrer is sustained, and the defendant is given a period of five days within which to amend its
aforesaid answer. y Whenever a right to rescind a contract of insurance is given to the insurer by any provision of this
chapter, such right must be exercised previous to the commencement of an action on the contract.
So ordered.
The defendant contended in the lower court and now contends in this court, that section 47 does not
To which the defendant duly excepted. apply to the new matters alleged in the special defense. If in legal effect defendant's special defense is in
the nature of an act to rescind "a contract of insurance," then such right must be exercised prior to an
As a result of the trial the general issues, the lower court rendered judgment for the plaintiff for P10,000, action enforce the contract. That is the real question involved in this appeal.
with legal interest from January 4, 1926, and costs, to which the defendant duly excepted and filed a
motion for a new trial, which was overruled. On appeal the defendant assigns the following errors: Defendant's original answer was a general and specific denial. In other words, it specifically denied that if
ever issued the policy in question, or that it ever agreed with Tan Ceang in the even of his death to pay
The trial court erred - P10,000 to the plaintiff or any one else. In its amended answer the defendant again makes a general and

Page 51 of 97
specific denial, and alleges the reasons, the specific facts, and the reasons why it never made or entered of insurance, for the simple reason that the minds of the parties never met and never agreed upon the
into the contract alleged in the complaint, and based upon those alleged facts, defendant contends that terms and conditions of the contract. We are clearly of the opinion that, if such matters are known to
it never did enter into any contract of insurance on the life of Tan Caeng. exist by a preponderance of the evidence, they would constitute a valid defense to plaintiff's cause of
action. Upon the question as to whether or not they or are not true, we do not at this time have or
The word "rescind" has a well defined legal meaning, and as applied to contracts, it presupposes the express any opinion, but we are clear that section 47 does not apply to the allegations made in the
existence of a contract to rescind. answer, and that the trial court erred in sustaining the demurrer.

Word & Phrases, volume 7, page 6139, says: The judgment of the lower court is reversed and the case is remanded for such other and further
proceedings as are not inconsistent with this opinion, with costs against the plaintiff. So ordered.
To rescind is to abrogate, annual, avoid, or cancel a contract.
18. Tan V. CA (174 SCRA 403, GRN 48009, 29 June 1989)
The word "rescind," as used in a statement by a party to a contrary as follows. "I hereby terminate and
rescind my said written contract," is synonymous with the word `terminate,' and the rescission therefore GUTIERREZ, JR., J.:
relates only to the unfulfilled part, and not to the entire agreement, making the party rescinding liable on
notes executed pursuant to the contract which matured before the rescission. This is a petition for review on certiorari of the Court of Appeals' decision affirming the decision of the
Insurance Commissioner which dismissed the petitioners' complaint against respondent Philippine
The rescission is the unmaking of a contract, requiring the same concurrence of wills as that which made American Life Insurance Company for the recovery of the proceeds from their late father's policy. The
it, and nothing short of this will suffice. There is a wide difference between the rescission of a contract facts of the case as found by the Court of Appeals are:
and its mere termination or cancellation.
Petitioners appeal from the Decision of the Insurance Commissioner dismissing herein
After a contract has been broken, whether by an inability to perform it, or by rescinding against right or petitioners' complaint against respondent Philippine American Life Insurance Company
otherwise, the party not in fault may sue the other for the damages suffered, or, if the parties can be for the recovery of the proceeds of Policy No. 1082467 in the amount of P 80,000.00.
placed in status quo, he may, should he prefer, return what he has received and recover in a suit value of
what he has paid or done. The latter remedy is termed "rescission." On September 23,1973, Tan Lee Siong, father of herein petitioners, applied for life
insurance in the amount of P 80,000.00 with respondent company. Said application
In the instant case, it will be noted that even in its prayer, the defendant does not seek to have the was approved and Policy No. 1082467 was issued effective November 6,1973, with
alleged insurance contract rescinded. It denies that it ever made any contract of insurance on the life of petitioners the beneficiaries thereof (Exhibit A).
Tan Ceang or that any such a contract ever existed, and that is the question which it seeks to have
litigated by its special defense. In the very nature of things, if the defendant never made or entered into On April 26,1975, Tan Lee Siong died of hepatoma (Exhibit B). Petitioners then filed
the contract in question, there is no contract to rescind, and, hence, section 47 upon which the lower with respondent company their claim for the proceeds of the life insurance policy.
based its decision in sustaining the demurrer does not apply. As stated, an action to rescind a contract is However, in a letter dated September 11, 1975, respondent company denied
founded upon and presupposes the existence of the contract which is sought to be rescinded. If all of the petitioners' claim and rescinded the policy by reason of the alleged misrepresentation
material matters set forth and alleged in the defendant's special plea are true, there was no valid contract and concealment of material facts made by the deceased Tan Lee Siong in his

Page 52 of 97
application for insurance (Exhibit 3). The premiums paid on the policy were thereupon The contention is without merit.
refunded .
The pertinent section in the Insurance Code provides:
Alleging that respondent company's refusal to pay them the proceeds of the policy
was unjustified and unreasonable, petitioners filed on November 27, 1975, a complaint Section 48. Whenever a right to rescind a contract of insurance is given to the insurer
against the former with the Office of the Insurance Commissioner, docketed as I.C. by any provision of this chapter, such right must be exercised previous to the
Case No. 218. commencement of an action on the contract.

After hearing the evidence of both parties, the Insurance Commissioner rendered After a policy of life insurance made payable on the death of the insured shall have
judgment on August 9, 1977, dismissing petitioners' complaint. (Rollo, pp. 91-92) been in force during the lifetime of the insured for a period of two years from the date
of its issue or of its last reinstatement, the insurer cannot prove that the policy is
The Court of Appeals dismissed ' the petitioners' appeal from the Insurance Commissioner's decision for void ab initio or is rescindable by reason of the fraudulent concealment or
lack of merit misrepresentation of the insured or his agent.

Hence, this petition. According to the petitioners, the Insurance Law was amended and the second paragraph of Section 48
added to prevent the insurance company from exercising a right to rescind after the death of the
The petitioners raise the following issues in their assignment of errors, to wit: insured.

A. The conclusion in law of respondent Court that respondent insurer has the right to The so-called "incontestability clause" precludes the insurer from raising the defenses of false
rescind the policy contract when insured is already dead is not in accordance with representations or concealment of material facts insofar as health and previous diseases are concerned if
existing law and applicable jurisprudence. the insurance has been in force for at least two years during the insured's lifetime. The phrase "during the
lifetime" found in Section 48 simply means that the policy is no longer considered in force after the
B. The conclusion in law of respondent Court that respondent insurer may be allowed insured has died. The key phrase in the second paragraph of Section 48 is "for a period of two years."
to avoid the policy on grounds of concealment by the deceased assured, is contrary to
the provisions of the policy contract itself, as well as, of applicable legal provisions and As noted by the Court of Appeals, to wit:
established jurisprudence.
The policy was issued on November 6,1973 and the insured died on April 26,1975. The
C. The inference of respondent Court that respondent insurer was misled in issuing the policy was thus in force for a period of only one year and five months. Considering
policy are manifestly mistaken and contrary to admitted evidence. (Rollo, p. 7) that the insured died before the two-year period had lapsed, respondent company is
not, therefore, barred from proving that the policy is void ab initio by reason of the
The petitioners contend that the respondent company no longer had the right to rescind the contract of insured's fraudulent concealment or misrepresentation. Moreover, respondent
insurance as rescission must allegedly be done during the lifetime of the insured within two years and company rescinded the contract of insurance and refunded the premiums paid on
prior to the commencement of action.

Page 53 of 97
September 11, 1975, previous to the commencement of this action on November We call attention to what this Honorable Court said in Insular Life v. Feliciano, et al., 73 Phil. 201; at page
27,1975. (Rollo, pp. 99-100) 205:

xxx xxx xxx It is of common knowledge that the selling of insurance today is subjected to the
whirlwind pressure of modern salesmanship.
The petitioners contend that there could have been no concealment or misrepresentation by their late
father because Tan Lee Siong did not have to buy insurance. He was only pressured by insistent Insurance companies send detailed instructions to their agents to solicit and procure
salesmen to do so. The petitioners state: applications.

Here then is a case of an assured whose application was submitted because of These agents are to be found all over the length and breadth of the land. They are
repeated visits and solicitations by the insurer's agent. Assured did not knock at the stimulated to more active efforts by contests and by the keen competition offered by
door of the insurer to buy insurance. He was the object of solicitations and visits. the other rival insurance companies.

Assured was a man of means. He could have obtained a bigger insurance, not just P They supply all the information, prepare and answer the applications, submit the
80,000.00. If his purpose were to misrepresent and to conceal his ailments in applications to their companies, conclude the transactions, and otherwise smooth out
anticipation of death during the two-year period, he certainly could have gotten a all difficulties.
bigger insurance. He did not.
The agents in short do what the company set them out to do.
Insurer Philamlife could have presented as witness its Medical Examiner Dr. Urbano
Guinto. It was he who accomplished the application, Part II, medical. Philamlife did not. The Insular Life case was decided some forty years ago when the pressure of insurance
salesmanship was not overwhelming as it is now; when the population of this country
Philamlife could have put to the witness stand its Agent Bienvenido S. Guinto, a relative was less than one-fourth of what it is now; when the insurance companies competing
to Dr. Guinto, Again Philamlife did not. (pp. 138139, Rollo) with one another could be counted by the fingers. (pp. 140-142, Rollo)

xxx xxx xxx xxx xxx xxx

This Honorable Supreme Court has had occasion to denounce the pressure and In the face of all the above, it would be unjust if, having been subjected to the
practice indulged in by agents in selling insurance. At one time or another most of us whirlwind pressure of insurance salesmanship this Court itself has long denounced, the
have been subjected to that pressure, that practice. This court took judicial cognizance assured who dies within the two-year period, should stand charged of fraudulent
of the whirlwind pressure of insurance selling-especially of the agent's practice of concealment and misrepresentation." (p. 142, Rollo)
'supplying the information, preparing and answering the application, submitting the
application to their companies, concluding the transactions and otherwise smoothing The legislative answer to the arguments posed by the petitioners is the "incontestability clause" added by
out all difficulties. the second paragraph of Section 48.

Page 54 of 97
The insurer has two years from the date of issuance of the insurance contract or of its last reinstatement claimed to have been diabetic for five years. (t.s.n., Aug. 23,1976, p. 5; Exhibit 6)
within which to contest the policy, whether or not, the insured still lives within such period. After two Because of the concealment made by the deceased of his consultations and
years, the defenses of concealment or misrepresentation, no matter how patent or well founded, no treatments for hypertension, diabetes and liver disorders, respondent company was
longer lie. Congress felt this was a sufficient answer to the various tactics employed by insurance thus misled into accepting the risk and approving his application as medically standard
companies to avoid liability. The petitioners' interpretation would give rise to the incongruous situation (Exhibit 5- C) and dispensing with further medical investigation and examination
where the beneficiaries of an insured who dies right after taking out and paying for a life insurance (Exhibit 5-A). For as long as no adverse medical history is revealed in the application
policy, would be allowed to collect on the policy even if the insured fraudulently concealed material facts. form, an applicant for insurance is presumed to be healthy and physically fit and no
further medical investigation or examination is conducted by respondent company.
The petitioners argue that no evidence was presented to show that the medical terms were explained in (t.s.n., April 8,1976, pp. 6-8). (Rollo, pp. 96-98)
a layman's language to the insured. They state that the insurer should have presented its two medical
field examiners as witnesses. Moreover, the petitioners allege that the policy intends that the medical There is no strong showing that we should apply the "fine print" or "contract of adhesion" rule in this
examination must be conducted before its issuance otherwise the insurer "waives whatever imperfection case. (Sweet Lines, Inc. v. Teves, 83 SCRA 361 [1978]). The petitioners cite:
by ratification."
It is a matter of common knowledge that large amounts of money are collected from
We agree with the Court of Appeals which ruled: ignorant persons by companies and associations which adopt high sounding titles and
print the amount of benefits they agree to pay in large black-faced type, following
On the other hand, petitioners argue that no evidence was presented by respondent such undertakings by fine print conditions which destroy the substance of the promise.
company to show that the questions appearing in Part II of the application for All provisions, conditions, or exceptions which in any way tend to work a forfeiture of
insurance were asked, explained to and understood by the deceased so as to prove the policy should be construed most strongly against those for whose benefit they are
concealment on his part. The same is not well taken. The deceased, by affixing his inserted, and most favorably toward those against whom they are meant to operate.
signature on the application form, affirmed the correctness of all the entries and (Trinidad v. Orient Protective Assurance Assn., 67 Phil. 184)
answers appearing therein. It is but to be expected that he, a businessman, would not
have affixed his signature on the application form unless he clearly understood its There is no showing that the questions in the application form for insurance regarding the insured's
significance. For, the presumption is that a person intends the ordinary consequence medical history are in smaller print than the rest of the printed form or that they are designed in such a
of his voluntary act and takes ordinary care of his concerns. [Sec. 5(c) and (d), Rule 131, way as to conceal from the applicant their importance. If a warning in bold red letters or a boxed warning
Rules of Court]. similar to that required for cigarette advertisements by the Surgeon General of the United States is
necessary, that is for Congress or the Insurance Commission to provide as protection against high
The evidence for respondent company shows that on September 19,1972, the pressure insurance salesmanship. We are limited in this petition to ascertaining whether or not the
deceased was examined by Dr. Victoriano Lim and was found to be diabetic and respondent Court of Appeals committed reversible error. It is the petitioners' burden to show that the
hypertensive; that by January, 1973, the deceased was complaining of progressive factual findings of the respondent court are not based on substantial evidence or that its conclusions are
weight loss and abdominal pain and was diagnosed to be suffering from hepatoma, contrary to applicable law and jurisprudence. They have failed to discharge that burden.
(t.s.n. August 23, 1976, pp. 8-10; Exhibit 2). Another physician, Dr. Wenceslao Vitug,
testified that the deceased came to see him on December 14, 1973 for consolation and

Page 55 of 97
WHEREFORE, the petition is hereby DENIED for lack of merit. The questioned decision of the Court of On April 20, 1966, Lee See Guat died of lung cancer. Thereafter, the beneficiary of the two policies,
Appeals is AFFIRMED. Vicente E. Tang claimed for their face value in the amount of P100,000 which the insurance company
refused to pay on the ground that the insured was guilty of concealment and misrepresentation at the
SO ORDERED.. time she applied for the two policies. Hence, the filing of Civil Case No. 90062 in the Court of First
Instance of Manila which dismissed the claim because of the concealment practised by the insured in
19. Tang V. CA (90 SCRA 236) violation of the Insurance Law.

ABAD SANTOS, J.: On appeal, the Court of Appeals, affirmed the decision. In its decision, the Court of Appeals stated, inter
alia: "There is no doubt that she deliberately concealed material facts about her physical condition and
This is a petition to review on certiorari of the decision of the Court of Appeals (CA-G.R. No. 55407-R,
history and/or conspired with whoever assisted her in relaying false information to the medical examiner,
June 8, 1978) which affirmed the decision of the Court of First Instance of Manila in Civil Case No. 90062
assuming that the examiner could not communicate directly with her."
wherein the petitioner herein was the plaintiff and Philippine American Life Insurance Co. the herein
respondent was the defendant. The action was for the enforcement of two insurance policies that had The issue in this appeal is the application of Art. 1332 of the Civil Code which stipulates:
been issued by the defendant company under the following circumstances.
Art. 1332. When one of the parties is unable to read, or if the contract is in a language
On September 25, 1965, Lee See Guat, a widow, 61 years old, and an illiterate who spoke only Chinese, not understood by him, and mistake or fraud is alleged, the person enforcing the
applied for an insurance on her life for P60,000 with the respondent Company. The application consisted contract must show that the terms thereof have been fully explained to the former.
of two parts, both in the English language. The second part of her application dealt with her state of
health and because her answers indicated that she was healthy, the Company issued her Policy No. According to the Code Commission: "This rule is especially necessary in the Philippines where
0690397, effective October 23, 1965, with her nephew Vicente E. Tang, herein Petitioner, as her unfortunately there is still a fairly large number of illiterates, and where documents are usually drawn up
beneficiary, in English or Spanish." (Report of the Code Commission, p. 136.) Art. 1332 supplements Art. 24 of the Civil
Code which provides that " In all contractual, property or other relations, when one of the parties is at a
On November 15, 1965, Lee See Guat again applied with the respondent Company for an additional disadvantage on account of his moral dependence, ignorance, indigence, mental weakness, tender age
insurance on her life for P40,000. Considering that her first application had just been approved, no or other handicap, the court must be vigilant for his protection.
further medical examination was made but she was required to accomplish and submit Part I of the
application which reads: "I/WE HEREBY DECLARE AND AGREE that all questions, statements answers It is the position of the petitioner that because Lee See Guat was illiterate and spoke only Chinese, she
contained herein, as well as those made to or to be made to the Medical Examiner in Part II are full, could not be held guilty of concealment of her health history because the applications for insurance were
complete and true and bind all parties in interest under the policy herein applied for; that there shall be in English and the insurer has not proved that the terms thereof had been fully explained to her.
no contract of insurance unless a policy is issued on this application and the fun first premium thereon,
according to the mode of payment specified in answer to question 4D above, actually paid during the It should be noted that under Art. 1332 above quoted, the obligation to show that the terms of the
lifetime and good health of the Proposed Insured." Moreover, her answers in Part II of her previous contract had been fully explained to the party who is unable to read or understand the language of the
application were used in appraising her insurability for the second insurance. On November 28, 1965, contract, when fraud or mistake is alleged, devolves on the party seeking to enforce it. Here the
Policy No. 695632 was issued to Lee See Guat with the same Vicente E. Tang as her beneficiary. insurance company is not seeking to enforce the contracts; on the contrary, it is seeking to avoid their

Page 56 of 97
performance. It is petitioner who is seeking to enforce them even as fraud or mistake is not alleged.
Accordingly, respondent company was under no obligation to prove that the terms of the insurance
contracts were fully explained to the other party. Even if we were to say that the insurer is the one 20. Pacific Timber Export Corp V. CA (112 SCRA 199)
seeking the performance of the contracts by avoiding paying the claim, it has to be noted as above
stated that there has been no imputation of mistake or fraud by the illiterate insured whose personality is
represented by her beneficiary the petitioner herein. In sum, Art. 1332 is inapplicable to the case at bar. DE CASTRO, ** J.:
Considering the findings of both the CFI and Court of Appeals that the insured was guilty of concealment
as to her state of health, we have to affirm. This petition seeks the review of the decision of the Court of Appeals reversing the decision of the Court
of First Instance of Manila in favor of petitioner and against private respondent which ordered the latter
WHEREFORE, the decision of the Court of Appeals is hereby affirmed. No special pronouncement as to to pay the sum of Pll,042.04 with interest at the rate of 12% interest from receipt of notice of loss on April
costs. 15, 1963 up to the complete payment, the sum of P3,000.00 as attorney's fees and the costs 1 thereby
dismissing petitioner s complaint with costs. 2
SO ORDERED.
The findings of the of fact of the Court of Appeals, which are generally binding upon this Court, Except as
Separate Opinions shall be indicated in the discussion of the opinion of this Court the substantial correctness of still
particular finding having been disputed, thereby raising a question of law reviewable by this Court 3 are
ANTONIO, J., concurring: as follows:

I concur. March 19, l963, the plaintiff secured temporary insurance from the defendant for its
exportation of 1,250,000 board feet of Philippine Lauan and Apitong logs to be
In a contract of insurance each party "must communicate to the other, in good faith, all facts within his shipped from the Diapitan. Bay, Quezon Province to Okinawa and Tokyo, Japan. The
knowledge which are material to the contract, and which the other has not the means of ascertaining ... defendant issued on said date Cover Note No. 1010, insuring the said cargo of the
(section 27, Act 2427, as amended. Emphasis supplied). As a general rule, a failure by the insured to plaintiff "Subject to the Terms and Conditions of the WORKMEN'S INSURANCE
disclose conditions affecting the risk, of which he is aware makes the contract voidable at the option of COMPANY, INC. printed Marine Policy form as filed with and approved by the Office
the insurer (45 C.J.S. 153). The reason for this rule is that insurance policies are traditionally contracts of the Insurance Commissioner (Exhibit A).
"ubemae fidei" which means most abundant good faith absolute and perfect candor or openness and
honesty; the absence of any concealment or deception however slight. Here, the Court of Appeals found The regular marine cargo policies were issued by the defendant in favor of the plaintiff
that the insured "deliberately concealed material facts about her physical condition and history and/or on April 2, 1963. The two marine policies bore the numbers 53 HO 1032 and 53 HO
concealed with whoever assisted her in relaying false information to the medical examiner ... " 1033 (Exhibits B and C, respectively). Policy No. 53 H0 1033 (Exhibit B) was for 542
pieces of logs equivalent to 499,950 board feet. Policy No. 53 H0 1033 was for 853
Certainly, petitioner cannot assume inconsistent positions by attempting to enforce the contract of pieces of logs equivalent to 695,548 board feet (Exhibit C). The total cargo insured
insurance for the purpose of collecting the proceeds of the policy and at the same time nullify the under the two marine policies accordingly consisted of 1,395 logs, or the equivalent of
contract by claiming that he executed the same thru fraud or mistake. 1,195.498 bd. ft.

Page 57 of 97
After the issuance of Cover Note No. 1010 (Exhibit A), but before the issuance of the (Sgd.) EMMANUEL S. ATILANO Asst. General Manager.
two marine policies Nos. 53 HO 1032 and 53 HO 1033, some of the logs intended to be
exported were lost during loading operations in the Diapitan Bay. The logs were to be Although dated April 4, 1963, the letter was received in the office of the defendant only
loaded on the 'SS Woodlock' which docked about 500 meters from the shoreline of on April 15, 1963, as shown by the stamp impression appearing on the left bottom
the Diapitan Bay. The logs were taken from the log pond of the plaintiff and from corner of said letter. The plaintiff subsequently submitted a 'Claim Statement
which they were towed in rafts to the vessel. At about 10:00 o'clock a. m. on March 29, demanding payment of the loss under Policies Nos. 53 HO 1032 and 53 HO 1033, in
1963, while the logs were alongside the vessel, bad weather developed resulting in 75 the total amount of P19,286.79 (Exhibit G).
pieces of logs which were rafted together co break loose from each other. 45 pieces of
logs were salvaged, but 30 pieces were verified to have been lost or washed away as a On July 17, 1963, the defendant requested the First Philippine Adjustment Corporation

result of the accident. to inspect the loss and assess the damage. The adjustment company submitted its
'Report on August 23, 1963 (Exhibit H). In said report, the adjuster found that 'the loss
In a letter dated April 4, 1963, the plaintiff informed the defendant about the loss of 'appropriately 32 of 30 pieces of logs is not covered by Policies Nos. 53 HO 1032 and 1033 inasmuch as
pieces of log's during loading of the 'SS Woodlock'. The said letter (Exhibit F) reads as follows: said policies covered the actual number of logs loaded on board the 'SS Woodlock'
However, the loss of 30 pieces of logs is within the 1,250,000 bd. ft. covered by Cover
April 4, 1963 Note 1010 insured for $70,000.00.

Workmen's Insurance Company, Inc. Manila, Philippines On September 14, 1963, the adjustment company submitted a computation of the
defendant's probable liability on the loss sustained by the shipment, in the total
Gentlemen: amount of Pl1,042.04 (Exhibit 4).

This has reference to Insurance Cover Note No. 1010 for shipment of 1,250,000 bd. ft. On January 13, 1964, the defendant wrote the plaintiff denying the latter's claim, on the
Philippine Lauan and Apitong Logs. We would like to inform you that we have received ground they defendant's investigation revealed that the entire shipment of logs
advance preliminary report from our Office in Diapitan, Quezon that we have lost covered by the two marines policies No. 53 110 1032 and 713 HO 1033 were received in
approximately 32 pieces of logs during loading of the SS Woodlock. good order at their point of destination. It was further stated that the said loss may be
considered as covered under Cover Note No. 1010 because the said Note had become
We will send you an accurate report all the details including values as soon as same 'null and void by virtue of the issuance of Marine Policy Nos. 53 HO 1032 and
will be reported to us. 1033'(Exhibit J-1). The denial of the claim by the defendant was brought by the plaintiff
to the attention of the Insurance Commissioner by means of a letter dated March 21,
Thank you for your attention, we wish to remain.
1964 (Exhibit K). In a reply letter dated March 30, 1964, Insurance Commissioner
Francisco Y. Mandanas observed that 'it is only fair and equitable to indemnify the
Very respectfully yours,
insured under Cover Note No. 1010', and advised early settlement of the said marine
loss and salvage claim (Exhibit L).
PACIFIC TIMBER EXPORT CORPORATION

Page 58 of 97
On June 26, 1964, the defendant informed the Insurance Commissioner that, on advice Note, it did not contain, as all Cover Notes do not contain particulars of the shipment that would serve as
of their attorneys, the claim of the plaintiff is being denied on the ground that the basis for the computation of the premiums. As a logical consequence, no separate premiums are
cover note is null and void for lack of valuable consideration (Exhibit M). 4 intended or required to be paid on a Cover Note. This is a fact admitted by an official of respondent
company, Juan Jose Camacho, in charge of issuing cover notes of the respondent company (p. 33, tsn,
Petitioner assigned as errors of the Court of Appeals, the following: September 24, 1965).

I At any rate, it is not disputed that petitioner paid in full all the premiums as called for by the statement
issued by private respondent after the issuance of the two regular marine insurance policies, thereby
THE COURT OF APPEALS ERRED IN HOLDING THAT THE COVER NOTE WAS NULL leaving no account unpaid by petitioner due on the insurance coverage, which must be deemed to
AND VOID FOR LACK OF VALUABLE CONSIDERATION BECAUSE THE COURT include the Cover Note. If the Note is to be treated as a separate policy instead of integrating it to the
DISREGARDED THE PROVEN FACTS THAT PREMIUMS FOR THE COMPREHENSIVE regular policies subsequently issued, the purpose and function of the Cover Note would be set at naught
INSURANCE COVERAGE THAT INCLUDED THE COVER NOTE WAS PAID BY or rendered meaningless, for it is in a real sense a contract, not a mere application for insurance which is
PETITIONER AND THAT INCLUDED THE COVER NOTE WAS PAID BY PETITIONER AND a mere offer. 6
THAT NO SEPARATE PREMIUMS ARE COLLECTED BY PRIVATE RESPONDENT ON ALL
ITS COVER NOTES. It may be true that the marine insurance policies issued were for logs no longer including those which
had been lost during loading operations. This had to be so because the risk insured against is not for loss
II during operations anymore, but for loss during transit, the logs having already been safely placed
aboard. This would make no difference, however, insofar as the liability on the cover note is concerned,
THE COURT OF APPEALS ERRED IN HOLDING THAT PRIVATE RESPONDENT WAS
for the number or volume of logs lost can be determined independently as in fact it had been so
RELEASED FROM LIABILITY UNDER THE COVER NOTE DUE TO UNREASONABLE
ascertained at the instance of private respondent itself when it sent its own adjuster to investigate and
DELAY IN GIVING NOTICE OF LOSS BECAUSE THE COURT DISREGARDED THE
assess the loss, after the issuance of the marine insurance policies.
PROVEN FACT THAT PRIVATE RESPONDENT DID NOT PROMPTLY AND SPECIFICALLY
OBJECT TO THE CLAIM ON THE GROUND OF DELAY IN GIVING NOTICE OF LOSS The adjuster went as far as submitting his report to respondent, as well as its computation of
AND, CONSEQUENTLY, OBJECTIONS ON THAT GROUND ARE WAIVED UNDER respondent's liability on the insurance coverage. This coverage could not have been no other than what
SECTION 84 OF THE INSURANCE ACT. 5 was stipulated in the Cover Note, for no loss or damage had to be assessed on the coverage arising from
the marine insurance policies. For obvious reasons, it was not necessary to ask petitioner to pay premium
1. Petitioner contends that the Cover Note was issued with a consideration when, by express stipulation,
on the Cover Note, for the loss insured against having already occurred, the more practical procedure is
the cover note is made subject to the terms and conditions of the marine policies, and the payment of
simply to deduct the premium from the amount due the petitioner on the Cover Note. The non-payment
premiums is one of the terms of the policies. From this undisputed fact, We uphold petitioner's
of premium on the Cover Note is, therefore, no cause for the petitioner to lose what is due it as if there
submission that the Cover Note was not without consideration for which the respondent court held the
had been payment of premium, for non-payment by it was not chargeable against its fault. Had all the
Cover Note as null and void, and denied recovery therefrom. The fact that no separate premium was
logs been lost during the loading operations, but after the issuance of the Cover Note, liability on the
paid on the Cover Note before the loss insured against occurred, does not militate against the validity of
note would have already arisen even before payment of premium. This is how the cover note as a
petitioner's contention, for no such premium could have been paid, since by the nature of the Cover
"binder" should legally operate otherwise, it would serve no practical purpose in the realm of commerce,

Page 59 of 97
and is supported by the doctrine that where a policy is delivered without requiring payment of the SO ORDERED.
premium, the presumption is that a credit was intended and policy is valid. 7
21. Coquia V. Fieldmen’s Insurance Co. (26 SCRA 178, GRN L-23276, 29 Nov. 1968)
2. The defense of delay as raised by private respondent in resisting the claim cannot be sustained. The
law requires this ground of delay to be promptly and specifically asserted when a claim on the insurance CONCEPCION, C.J.:
agreement is made. The undisputed facts show that instead of invoking the ground of delay in objecting
to petitioner's claim of recovery on the cover note, it took steps clearly indicative that this particular This is an appeal from a decision of the Court of First Instance of Manila, certified to us by the Court of

ground for objection to the claim was never in its mind. The nature of this specific ground for resisting a Appeals, only questions of law being involved therein. Indeed, the pertinent facts have been stipulated

claim places the insurer on duty to inquire when the loss took place, so that it could determine whether and/or, admitted by the parties at the hearing of the case in the trial court, to dispense with the

delay would be a valid ground upon which to object to a claim against it. presentation of evidence therein.

As already stated earlier, private respondent's reaction upon receipt of the notice of loss, which was on It appears that on December 1, 1961, appellant Fieldmen's Insurance Company, Inc. — hereinafter

April 15, 1963, was to set in motion from July 1963 what would be necessary to determine the cause and referred to as the Company — issued, in favor of the Manila Yellow Taxicab Co., Inc. — hereinafter

extent of the loss, with a view to the payment thereof on the insurance agreement. Thus it sent its referred to as the Insured — a common carrier accident insurance policy, covering the period from

adjuster to investigate and assess the loss in July, 1963. The adjuster submitted his report on August 23, December 1, 1961 to December 1, 1962. It was stipulated in said policy that:

1963 and its computation of respondent's liability on September 14, 1963. From April 1963 to July, 1963,
The Company will, subject to the Limits of Liability and under the Terms of this Policy, indemnify
enough time was available for private respondent to determine if petitioner was guilty of delay in
the Insured in the event of accident caused by or arising out of the use of Motor Vehicle against
communicating the loss to respondent company. In the proceedings that took place later in the Office of
all sums which the Insured will become legally liable to pay in respect of: Death or bodily injury
the Insurance Commissioner, private respondent should then have raised this ground of delay to avoid
to any fare-paying passenger including the Driver, Conductor and/or Inspector who is riding in
liability. It did not do so. It must be because it did not find any delay, as this Court fails to find a real and
the Motor Vehicle insured at the time of accident or injury. 1
substantial sign thereof. But even on the assumption that there was delay, this Court is satisfied and
convinced that as expressly provided by law, waiver can successfully be raised against private
While the policy was in force, or on February 10, 1962, a taxicab of the Insured, driven by Carlito Coquia,
respondent. Thus Section 84 of the Insurance Act provides:
met a vehicular accident at Mangaldan, Pangasinan, in consequence of which Carlito died. The Insured
filed therefor a claim for P5,000.00 to which the Company replied with an offer to pay P2,000.00, by way
Section 84.—Delay in the presentation to an insurer of notice or proof of loss is waived
of compromise. The Insured rejected the same and made a counter-offer for P4,000.00, but the
if caused by any act of his or if he omits to take objection promptly and specifically
Company did not accept it. Hence, on September 18, 1962, the Insured and Carlito's parents, namely,
upon that ground.
Melecio Coquia and Maria Espanueva — hereinafter referred to as the Coquias — filed a complaint

From what has been said, We find duly substantiated petitioner's assignments of error. against the Company to collect the proceeds of the aforementioned policy. In its answer, the Company
admitted the existence thereof, but pleaded lack of cause of action on the part of the plaintiffs.
ACCORDINGLY, the appealed decision is set aside and the decision of the Court of First Instance is
reinstated in toto with the affirmance of this Court. No special pronouncement as to costs. After appropriate proceedings, the trial court rendered a decision sentencing the Company to pay to the
plaintiffs the sum of P4,000.00 and the costs. Hence, this appeal by the Company, which contends that

Page 60 of 97
plaintiffs have no cause of action because: 1) the Coquias have no contractual relation with the Company; xxx xxx xxx
and 2) the Insured has not complied with the provisions of the policy concerning arbitration.
7. In the event of death of any person entitled to indemnity under this Policy, the Company will,
As regards the first defense, it should be noted that, although, in general, only parties to a contract may in respect of the liability incurred by such person, indemnify his personal representatives in
bring an action based thereon, this rule is subject to exceptions, one of which is found in the second terms of and subject to the limitations of this Policy, provided, that such representatives shall, as
paragraph of Article 1311 of the Civil Code of the Philippines, reading: though they were the Insured, observe, fulfill and be subject to the Terms of this Policy insofar
as they can apply.
If a contract should contain some stipulation in favor of a third person, he may demand its
fulfillment provided he communicated his acceptance to the obligor before its revocation . A 8. The Company may, at its option, make indemnity payable directly to the claimants or heirs of
mere incidental benefit or interest of a person is not sufficient. The contracting parties must claimants, with or without securing the consent of or prior notification to the Insured, it being
have clearly and deliberately conferred a favor upon a third person.2 the true intention of this Policy to protect, to the extent herein specified and subject always to
the Terms Of this Policy, the liabilities of the Insured towards the passengers of the Motor
This is but the restatement of a well-known principle concerning contracts pour autrui, the enforcement Vehicle and the Public.
of which may be demanded by a third party for whose benefit it was made, although not a party to the
contract, before the stipulation in his favor has been revoked by the contracting parties. Does the policy Pursuant to these stipulations, the Company "will indemnify any authorized Driver who is driving the
in question belong to such class of contracts pour autrui? Motor Vehicle" of the Insured and, in the event of death of said driver, the Company shall, likewise,
"indemnify his personal representatives." In fact, the Company "may, at its option, make indemnity
In this connection, said policy provides, inter alia: payable directly to the claimants or heirs of claimants ... it being the true intention of this Policy to
protect ... the liabilities of the Insured towards the passengers of the Motor Vehicle and the Public" — in
Section I — Liability to Passengers. 1. The Company will, subject to the Limits of Liability and other words, third parties.
under the Terms of this Policy, indemnify the Insured in the event of accident caused by or
arising out of the use of Motor Vehicle against all sums which the Insured will become legally Thus, the policy under consideration is typical of contracts pour autrui, this character being made more
liable to pay in respect of: Death or bodily injury to any fare-paying passenger including the manifest by the fact that the deceased driver paid fifty percent (50%) of the corresponding premiums,
Driver ... who is riding in the Motor Vehicle insured at the time of accident or injury. which were deducted from his weekly commissions. Under these conditions, it is clear that the Coquias
— who, admittedly, are the sole heirs of the deceased — have a direct cause of action against the
Section II — Liability to the Public Company,3 and, since they could have maintained this action by themselves, without the assistance of the
Insured, it goes without saying that they could and did properly join the latter in filing the complaint
xxx xxx xxx
herein.4

3. In terms of and subject to the limitations of and for the purposes of this Section, the
The second defense set up by the Company is based upon Section 17 of the policy reading:
Company will indemnify any authorized Driver who is driving the Motor Vehicle....

If any difference or dispute shall arise with respect to the amount of the Company's liability
Conditions
under this Policy, the same shall be referred to the decision of a single arbitrator to be agreed

Page 61 of 97
upon by both parties or failing such agreement of a single arbitrator, to the decision of two consents to an arbitration; and if the insured demands an arbitration, and the company refuses
arbitrators, one to be appointed in writing by each of the parties within one calendar month to accede to the demand, the insured may maintain a suit on the policy, notwithstanding the
after having been required in writing so to do by either of the parties and in case of language of the twelfth section of the policy, and, where neither party demands an arbitration,
disagreement between the arbitrators, to the decision of an umpire who shall have been both parties thereby waive it.6
appointed in writing by the arbitrators before entering on the reference and the costs of and
incident to the reference shall be dealt with in the Award. And it is hereby expressly stipulated To the same effect was the decision of the Supreme Court of Minnesota in Independent School Dist. No.
and declared that it shall be a condition precedent to any right of action or suit upon this Policy 35, St. Louis County vs. A. Hedenberg & Co., Inc.7 from which we quote:
that the award by such arbitrator, arbitrators or umpire of the amount of the Company's liability
hereunder if disputed shall be first obtained. This rule is not new in our state. In Meyer v. Berlandi, 53 Minn. 59, 54 N.W. 937, decided in 1893,
this court held that the parties to a construction contract, having proceeded throughout the
The record shows, however, that none of the parties to the contract invoked this section, or made any entire course of their dealings with each other in entire disregard of the provision of the
reference to arbitration, during the negotiations preceding the institution of the present case. In fact, contract regarding the mode of determining by arbitration the value of the extras, thereby
counsel for both parties stipulated, in the trial court, that none of them had, at any time during said waived such provision.
negotiations, even suggested the settlement of the issue between them by arbitration, as provided in
said section. Their aforementioned acts or omissions had the effect of a waiver of their respective right to xxx xxx xxx

demand an arbitration. Thus, in Kahnweiler vs. Phenix Ins. Co. of Brooklyn,5 it was held:
The test for determining whether there has been a waiver in a particular case is stated by the

Another well-settled rule for interpretation of all contracts is that the court will lean to that author of an exhaustive annotation in 117 A.L.R. p. 304, as follows: "Any conduct of the parties

interpretation of a contract which will make it reasonable and just. Bish. Cont. Sec. 400. Applying inconsistent with the notion that they treated the arbitration provision as in effect, or any

these rules to the tenth clause of this policy, its proper interpretation seems quite clear. When conduct which might be reasonably construed as showing that they did not intend to avail

there is a difference between the company and the insured as to the amount of the loss the themselves of such provision, may amount to a waiver thereof and estop the party charged with

policy declares: "The same shall then be submitted to competent and impartial arbitrators, one such conduct from claiming its benefits".

to be selected by each party ...". It will be observed that the obligation to procure or demand an
xxx xxx xxx
arbitration is not, by this clause, in terms imposed on either party. It is not said that either the
company or the insured shall take the initiative in setting the arbitration on foot. The company
The decisive facts here are that both parties from the inception of their dispute proceeded in
has no more right to say the insured must do it than the insured has to say the company must
entire disregard of the provisions of the contract relating to arbitration and that neither at any
do it. The contract in this respect is neither unilateral nor self-executing. To procure a reference
stage of such dispute, either before or after commencement of the action, demanded
to arbitrators, the joint and concurrent action of both parties to the contract is indispensable.
arbitration, either by oral or written demand, pleading, or otherwise. Their conduct was as
The right it gives and the obligation it creates to refer the differences between the parties to
effective a rejection of the right to arbitrate as if, in the best Coolidge tradition, they had said,
arbitrators are mutual. One party to the contract cannot bring about an arbitration. Each party is
"We do not choose to arbitrate". As arbitration under the express provisions of article 40 was "at
entitled to demand a reference, but neither can compel it, and neither has the right to insist that
the choice of either party," and was chosen by neither, a waiver by both of the right to
the other shall first demand it, and shall forfeit any right by not doing so. If the company
arbitration followed as a matter of law.
demands it, and the insured refuses to arbitrate, his right of action is suspended until he
Page 62 of 97
WHEREFORE, the decision appealed from should be as it is hereby affirmed in toto, with costs against the As a rule, the nature of the title of the insured over the property which was under the policy is immaterial
herein defendant-appellant, Fieldmen's Insurance Co., Inc. It is so ordered. to the insured’s right to collect on the policy. The insured has a right to collect on the policy, so long as
his relation to the insured property is such that he will be benefited by its existence and injured by its
22. Harvardian Colleges V. Country Bankers Ins. Corp.
loss. The test in determining insurable interest in property is whether one will derive pecuniary benefit or
advantage from its preservation, or will suffer pecuniary loss or damage from its destruction, termination
(NOTE: Did not find any full text online.) or injury by the happening of the event insured against. Harvardian was not only in possession of the
building, but in fact had been using the same for several years with the knowledge and consent of Yap.
Thus, it is reasonable to assume that, had the building not been burned, Harvardian would have been

FACTS: allowed the continued use of the site for its operations. Harvardian is therefore directly affected by the

loss thereof.
Petitioner Yap was president of the Philippine Harvardian College, an educational institution, of which he,
his wife, and his children were all the stockholders. In 1979, a sales agent of respondent Country Bankers WHEREFORE, Harvardian has the right to the proceeds of the fire insurance policy.

proposed that Harvardian insure its primary school building, which Harvardian agreed to.oAfter
inspection, the school building was insured for PhP 500,000.00.oHarvardian contracted itself to pay an
annual premium of PhP 2,500.00.oCounty Bankers therefore issued the fire insurance policy to 23. Development Insurance Corp V. IAC (143 SCRA 62, GRN 71360, 16 July 1986)
Harvardian. In 1980, during the effectivity of the abovementioned insurance policy, the school building
CRUZ, J.:
was totally burned, rendering it a total loss. Harvardian therefore made a claim on Country Bankers
under its insurance policy. Country Bankers denied the claim, contending that plaintiff had no insurable
A fire occurred in the building of the private respondent and it sued for recovery of damages from the
interest over the building, since it had been constructed on the piece of land in the name of the late Yap. petitioner on the basis of an insurance contract between them. The petitioner allegedly failed to answer
Neither party contended that the lot and the building were both in the name of Yap, nor that the policy on time and was declared in default by the trial court. A judgment of default was subsequently rendered
was in the name of Harvardian. on the strength of the evidence submitted ex parte by the private respondent, which was allowed full
recovery of its claimed damages. On learning of this decision, the petitioner moved to lift the order of
ISSUE: default, invoking excusable neglect, and to vacate the judgment by default. Its motion was denied. It then
went to the respondent court, which affirmed the decision of the trial court in toto. The petitioner is now
W/N Harvardian has a right to the proceeds of the fire insurance policy.
before us, hoping presumably that it will fare better here than before the trial court and the Intermediate
Appellate Court. We shall see.
HELD:

On the question of default, the record argues mightily against it. It is indisputable that summons was
Yes, it does.
served on it, through its senior vice-president, on June 19,1980. On July 14, 1980, ten days after the
expiration of the original 15-day period to answer (excluding July 4), its counsel filed an ex parte motion

Page 63 of 97
for an extension of five days within which to file its answer. On July 18, 1980, the last day of the requested No less preposterous is the petitioner's claim that the elevators were insured after the occurrence of the
extension-which at the time had not yet been granted-the same counsel filed a second motion for fire, a case of shutting the barn door after the horse had escaped, so to speak.4 This pretense merits
another 5-day extension, fourteen days after the expiry of the original period to file its answer. The trial scant attention. Equally undeserving of serious consideration is its submission that the elevators were not
court nevertheless gave it five days from July 14, 1980, or until July 19, 1980, within which to file its answer. damaged by the fire, against the report of The arson investigators of the INP5 and, indeed, its own
But it did not. It did so only on July 26, 1980, after the expiry of the original and extended periods, or expressed admission in its answer6 where it affirmed that the fire "damaged or destroyed a portion of the
twenty-one days after the July 5, deadline. As a consequence, the trial court, on motion of the private 7th floor of the insured building and more particularly a Hitachi elevator control panel." 7
respondent filed on July 28, 1980, declared the petitioner in default. This was done almost one month
later, on August 25, 1980. Even so, the petitioner made no move at all for two months thereafter. It was There is no reason to disturb the factual findings of the lower court, as affirmed by the Intermediate
only on October 27, 1980, more than one month after the judgment of default was rendered by the trial Appellate Court, that the heat and moisture caused by the fire damaged although they did not actually
court on September 26, 1980, that it filed a motion to lift the order of default and vacate the judgment by burn the elevators. Neither is this Court justified in reversing their determination, also factual, of the value
default.1 of the loss sustained by the private respondent in the amount of P508,867.00.

The pattern of inexcusable neglect, if not deliberate delay, is all too clear. The petitioner has slumbered The only remaining question to be settled is the amount of the indemnity due to the private respondent
on its right and awakened too late. While it is true that in Trajano v. Cruz,2 which it cites, this Court under its insurance contract with the petitioner. This will require an examination of this contract, Policy
declared "that judgments by default are generally looked upon with disfavor," the default judgment in No. RY/F-082, as renewed, by virtue of which the petitioner insured the private respondent's building
that case was set aside precisely because there was excusable neglect, Summons in that case was served against fire for P2,500,000.00. 8
through "an employee in petitioners' office and not the person in-charge," whereas in the present case
summons was served on the vice-president of the petitioner who however refused to accept it. The petitioner argues that since at the time of the fire the building insured was worth P5,800,000.00, the

Furthermore, as Justice Guerrero noted, there was no evidence showing that the petitioners in Trajano private respondent should be considered its own insurer for the difference between that amount and the

intended to unduly delay the case. face value of the policy and should share pro rata in the loss sustained. Accordingly, the private
respondent is entitled to an indemnity of only P67,629.31, the rest of the loss to be shouldered by it
Besides, the petitioners in Trajano had a valid defense against the complaint filed against them, and this alone. In support of this contention, the petitioner cites Condition 17 of the policy, which provides:
justified a relaxation of the procedural rules to allow full hearing on the substantive issues raised. In the
instant case, by contrast, the petitioner must just the same fail on the merits even if the default orders If the property hereby insured shall, at the breaking out of any fire, be collectively of

were to be lifted. As the respondent Court observed, "Nothing would be gained by having the order of greater value than the sum insured thereon then the insured shall be considered as

default set aside considering the appellant has no valid defense in its favor." 3 being his own insurer for the difference, and shall bear a ratable proportion of the loss
accordingly. Every item, if more than one, of the policy shall be separately subject to
The petitioner's claim that the insurance covered only the building and not the elevators is absurd, to say this condition.
the least. This Court has little patience with puerile arguments that affront common sense, let alone basic
legal principles with which even law students are familiar. The circumstance that the building insured is However, there is no evidence on record that the building was worth P5,800,000.00 at the time of the

seven stories high and so had to be provided with elevators-a legal requirement known to the petitioner loss; only the petitioner says so and it does not back up its self-serving estimate with any independent

as an insurance company-makes its contention all the more ridiculous. corroboration. On the contrary, the building was insured at P2,500,000.00, and this must be considered,
by agreement of the insurer and the insured, the actual value of the property insured on the day the fire

Page 64 of 97
occurred. This valuation becomes even more believable if it is remembered that at the time the building 24. Harding V. Commercial Union Ass. Co. (38 Phil 464, GRN L-12707, 10 August 1918)
was burned it was still under construction and not yet completed. FISHER, J.:

This was an action by plaintiffs to recover from defendant the sum of P3,000 and interest, alleged to be
The Court notes that Policy RY/F-082 is an open policy and is subject to the express condition that:
due under the ierms of a policy of insurance. The trial court gave plaintiffs judgment for the amount

Open Policy demanded, with interest and costs, and from that decision the defendant appeals.

'The court below stated the issues made by the pleadings in this case, and its findings of fact, as follows:
This is an open policy as defined in Section 57 of the Insurance Act. In the event of
loss, whether total or partial, it is understood that the amount of the loss shall be "It is alleged by plaintiffs and admitted by defendant that plaintiffs are husband and wife and residents of

subject to appraisal and the liability of the company, if established, shall be limited to the city of Manila; that the defendant is a foreign corporation organized and existing under and by virtue

the actual loss, subject to the applicable terms, conditions, warranties and clauses of of the laws of Great Britain and duly registered in the Philippine Islands, and Smith, Bell & Co. (limited), a

this Policy, and in no case shall exceed the amount of the policy. corporation organized and existing under the laws of the Philippine Islands, with its principal domicile in
the city of Manila, is the agent in the Philippine Islands of said defendant.
As defined in the aforestated provision, which is now Section 60 of the Insurance Code, "an open policy is
"The plaintiffs alleged that on February 16, 1916, the plaintiff Mrs. Henry E. Harding was the owner of a
one in which the value of the thing insured is not agreed upon but is left to be ascertained in case of loss.
Studebaker automobile, registered number 2063, in the city of Manila; that on said date, in consideration
" This means that the actual loss, as determined, will represent the total indemnity due the insured from
of the payment to the defendant of the premium of P150, by said plaintiff, Mrs. Henry E. Harding, with
the insurer except only that the total indemnity shall not exceed the face value of the policy.
the consent of her husband, the defendant by its duly authorized agent, Smith, Bell & Company (limited),
made its policy of insurance in writing upon said automobile in the sum of P3,000 and that the value of
The actual loss has been ascertained in this case and, to repeat, this Court will respect such factual
said automobile was set forth in said policy (Exhibit A) to be P3,000; that on March 24, 1916, said
determination in the absence of proof that it was arrived at arbitrarily. There is no such showing. Hence,
automobile was totally destroyed by fire; that the loss thereby to plaintiffs was the sum of P3,000; that
applying the open policy clause as expressly agreed upon by the parties in their contract, we hold that
thereafter, within the period mentioned in the said policy of insurance, the plaintiff, Mrs. Henry E.
the private respondent is entitled to the payment of indemnity under the said contract in the total
Harding, furnished the defendant the proofs of her said loss and interest, and otherwise performed all
amount of P508,867.00.
the conditions of said policy on her part, and that the defendant has not paid said loss nor any part .
thereof, although due demand was made upon defendant therefor.
The refusal of its vice-president to receive the private respondent's complaint, as reported in the sheriff's
return, was the first indication of the petitioner's intention to prolong this case and postpone the "The defendant, by its answer, admitted the allegations of the residence and status of the parties and
discharge of its obligation to the private respondent under this agreement. That intention was revealed denied all the other allegation of the said complaint, and for a separate and affirmative defense alleged
further in its subsequent acts-or inaction-which indeed enabled it to avoid payment for more than five (1) that on February 17, 1916, at the city of Manila, P. I., the defendant upon request of plaintiff, Mrs. Henry
years from the filing of the claim against it in 1980. The petitioner has temporized long enough to avoid E. Harding, issued to the said plaintiff the policy of insurance on an automobile alleged by the said
its legitimate responsibility; the delay must and does end now. plaintiff to be her property; that the said request for the issuance of said policy of insurance was made by
means of a proposal in writing signed and delivered by said plaintiff to the defendant, guaranteeing the
WHEREFORE, the appealed decision is affirmed in full, with costs against the petitioner. SO ORDERED. truth of the statements contained therein which said proposal is referred to in the said policy of insurance
and made a part thereof; (2) that certain of the statements and representations contained in said

Page 65 of 97
proposal and warranted by said plaintiff to be true, to wit: (a) the price paid by the proposer for the said policy the said automobile was described as of the 'present value' of P3,000, and the said defendant
automobile; (b) the value of said automobile at the time of the execution and delivery of the said charged the said plaintiff Mrs. Henry E. Harding as premium on said policy the sum of P150, or 5 per cent
proposal and (c) the ownership of said automobile, were false and known to be false by the said plaintiff of the then estimated value of P3,000 (Exhibit A).
at the time of signing and delivering the said proposal and were made for the purpose of misleading and
"The 'Schedule' in said policy of insurance describes the automobile here in question, and provides in
deceiving the defendant, and inducing the defendant to issue the said policy of insurance; (3) that the
part as follows:
defendant, relying upon the warranties, statements, and representations contained in the said proposal
and believing the same to be true, issued the said policy of insurance. " 'Now it is hereby agreed as follows:

"The defendant prays that judgment be entered declaring the said policy of insurance to be null and " 'That during the period above set forth and during any period for which the company may agree to
void, and that plaintiffs take nothing by this action; and for such further relief as to the court may seem renew this policy the company will subject to the exception and conditions contained herein or endorsed
just and equitable. hereon indemnify the insured against loss of or damage to any motor car described in the schedule
hereto' (including accessories) by whatever cause such loss or damage may be occasioned and will
"The evidence in this case shows that some time in the year 1913 Levy Hermanos, the Manila agents for
further indemnify the insured up to the value of the car or P3,000 whichever is the greater against any
the Studebaker automobile, sold the automobile No. 2063 to John Ganson for P3,200 (testimony of Mr.
claim at common law made by any person (not being a person in the said motor car nor in the insured's
Diehl) ; that under date of October 14, 1914, John Canson sold the said automobile to Henry Harding for
service) for loss of life or for accidental bodily injury or damage to property caused by the said motor car
the sum of P1,500 (Exhibit 2); that under date of November 19, 1914, the said Henry Harding sold the said
including law costs payable in connection with such claim when incurred with the consent of the
automobile No. 2063 to J. Brannigan, of Los Banos, Province of Laguna, P. I., for the sum of P2,000
company.'
(Exhibit 3) ; that under date of December 20, 1915, J. C. Graham of Los Banos, Province of Laguna, P. I.,
sold the said automobile No. 2063 to Henry Harding of the city of Manila for the sum of P2,800 (Exhibit 4 "The evidence further shows that on March 24, 1916, the said automobile was totally destroyed by. fire,
and testimony of J. C. Graham); that on or about January 1, 1916, the said Henry Harding gave the said and that the iron and steel portions of said automobile which did not burn were taken into the
automobile to his wife, Mrs. Henry E. Harding, one of the plaintiffs, as a present; that said automobile was possession of the defendant by and through its agent Smith, Bell & Company (limited), and sold by it for
repaired and repainted at the Luneta Garage at a cost of some P900 (testimony of Mr. Server); that while a small sum, which had never been tendered to the plaintiff prior to the trial of this case, but in open
the said automobile was at the Lufleta Garage, the said Luneta Garage, acting as agent for Smith, Bell & court during the trial the sum of P10 as the proceeds of such sale was tendered to plaintiff and refused."
Company (limited), solicited of the plaintiff Mrs. Harding the insurance of said automobile by the
Upon the facts so found, which we hold are supported by the evidence, the trial judge deciced that there
defendant Company (testimony of Mrs. Harding and Mr. Server); that a proposal was filled out by the
was no proof of fraud on the part of plaintiff in her statement of the value of the automobile, or with
said agent and signed by the plaintiff Mrs. Henry E. Harding, and in said proposal under the heading
respect to its ownership; that she had an insurable interest therein; and that defendant, having agreed to
'Price paid by proposer' is the amount of '3,500' and under another heading "Present value" is the
the estimated value, P3,000 and haying, insured thei automobile for that amount, upon the basis of
amount of '3,000' (Exhibit 1).
which the premium was paid, is bound by it and must pay the loss in accordance with the stipulated
"The evidence tends to show that after the said proposal was made a representative of the Manila agent insured value. The assignments of error made on behalf of appellant put in issue the correctness of those
of defendant went to the Luneta Garage and examined said automobile No. 2063 and Mr. Server, the conclusions of law, and some others of minor importance relating to the exclusion of evidence. Disposing
General Manager of the Luneta Garage, an experienced automobile mechanic, testified that at the time of the minor objections first, as we have reached the conclusion that the trial court was right in holding
this automobile was insured it was worth about P3,000, and the defendant, by and through its said agent that the defendant is bound by the estimated value of the automobile upon which the policy was issued,
Smith, Bell & Company (limited), thereafter issued a policy of insurance upon said proposal, in which and that the plaintiff was not guilty of fraud in regard thereto, the exclusion of the testimony of the

Page 66 of 97
witness Diehl is without importance. It merely tended to show the alleged actual value of the automobile, it has not been shown by the evidence that the statement was false on the contrary we believe that it
and in the view we take of the case such evidence was irrelevant. shows that the automobile had in fact cost more than the amount mentioned. The court below found,
and the evidence shows, that the automobile was bought by plaintiff's husband a few weeks before the
Appellant contends that Mrs. Harding was not the owner of the automobile at the time of the issuance of
issuance of the policy in question for the sum of P2,800, and that between that time and the issuance of
the policy, and, therefore, had no insurable interest in it. The court below found that the automobile was
the policy some P900 was spent upon it in repairs and repainting. The witness Server, an expert
given to plaintiff by her husband shortly, before the issuance of the policy here in question. Appellant
automobile mechanic, testified that the automobile was practically as good as new at the time the
does not dispute the correctness of this finding, but contends that the gift was void, citing article 1334 of
insurance was effected. The form of proposal upon which the policy was issued does not call for a
the Civil Code which provides that "All gifts between spouses during the marriage shall be void.
statement regarding the value of the automobile at the time of its acquisition by the applicant for the
Moderate gifts which the spouses bestow on each other on festive days of the family are not included in
insurance, but merely a statement of its cost. The amount stated was less than the actual outlay which
this rule."
the automobile represented to Mr. Harding, including repairs, when the insurance policy was issued. It is
We are of the opinion that this contention is without merit. In the case of Cook vs. McMicking (27 Phil. true that the printed form calls for a statement of the "price paid by the proposer," but we are of the
Rep., 10), this court said: opinion that it would be unfair to hold the policy void simply because the outlay represented by the
automobile was made by the plaintiff's husband and not by his wife, to whom he had given the
"It is claimed by the appellants that the so-called transfer from plaintiff's husband to her was completely
automobile. It cannot be assumed that defendant should not have issued the policy unless it were strictly
void under article 1458 of the Civil Code and that, therefore, the property still remains the property of
true that the price representing the cost of the machine had been paid by the insured and by no other
Edward Cook and subject to levy under execution against him.
person that it would in no event insure an automobile acquired by gift, inheritance, exchange, or any
In our opinion the position taken by appellants is untenable. They are not in a position to challenge the other title not requiring the owner to make a specific cash outlay for its acquisition.
validity of the transfer, if it may be called such. They bore absolutely no relation to the parties to the
Furthermore, the court below found and the evidence shows, without dispute, that the proposal upon
transfer at the time it occurred and had no rights or interests inchoate, present, remote,or otherwise, in
which the policy in question was issued was made out by defendant's agent, by whom the insurance was
the property in question at the time the transfer occurred, Although certain transfers from husband to
solicited, and that appellee simply signed the same. It also appears that an examiner employed by the
wife or from wife to husband are prohibited in the article referred to, such prohibition can be taken
defendant made an inspection of the automobile before the acceptance of the, risk, and that the sum of
advantage of only by persons who bear such a relation to the parties making the transfer or to the
P3,000 was fixed as the "present value" of the automobile after this examination. The trial court found
property itself that such transfer interferes with their rights or interests. Unless such a relationship
that Mrs. Harding, in fixing the value of the automobile at P3,000, acted upon information given her by
appears the transfer cannot be attacked."
her husband and by Mr. Server, the manager of the Luneta Garage. The Luneta Garage, it will be
Even assuming that defendant might have invoked article 1334 as a defense, the burden would be upon remembered, was the agent of the defendant corporation in the solicitation of the insurance. Mrs.
it to show that the gift in question does not fall within the exception therein established. We cannot say, Harding did not state of her own knowledge that the automobile originally cost P3,000, or that its value
as a matter of law, that the gift of an automobile by a husband to his wife is not a moderate one. at the time of the insurance was P3,000. She merely repeated the information which had been given her
Whether it is or is not would depend upon the circumstances of the parties, as to which nothing is by her husband, and at the same time disclosed to defendant's agent the source of her information.
disclosed by the record. There is no evidence to sustain the contention that this communication was made in bad faith. It appears
that the statements in the proposal as to the price paid for the automobile and as to its value were
Defendant contends that the statement regarding the cost of the automobile was a warranty, that the
written by Mr. Quimby who solicited the insurance on behalf of defendant, in his capacity as an
statement was false, and that, therefore, the policy never attached to the risk. We are of the opinion that
employee of the Luneta Garage, and wrote out the proposal for Mrs. Harding to sign. Under these

Page 67 of 97
circumstances, we do not think that the facts stated in the proposal can be held as a warranty of the Schoolenberger, 44 Pa., 259; Beal vs. Ins. Co., 16 Wis., 241; Davenport vs. Ins. Co., 17 Iowa, 276.) An
insured, even if it should have been shown that they were incorrect in the absence of proof of willful insurance company, establishing a local agency, must be held responsible to the parties with whom they
misstatement. Under such circumstance, the proposal is to be regarded as the act of the insurer and not transact business, for the acts and declarations of the agent, within the scope of his employment, as if
of the insured. This question was considered in the case of the Union Insurance Company vs. Wilkinson they proceeded from the principal. (Sav. Bk. vs. Ins. Co., 31 Conn., 517; Hortwite vs. Ins. Co., 40 Mo., 557;
(13 Wall., 222; 20 L. ed., 617), in which the Supreme Court of the United States said: Ayres vs. Ins. Co., 17 Iowa, 176; Howard Ins. Co. vs. Bruner, 23 Pa., 50.)

"This question has been decided differently by courts of the highest respectability in cases precisely "In the fifth edition of American Leading Cases, 917, after a full consideration of the authorities, it is said:
analogous to the present. It is not to be denied that the application, logically considered, is the work of
" 'By the interested or officious zeal of the agents employed by the insurance companies in the wish to
the assured, and if left to himself or to such assistance as he might select, the person so selected would
outbid each other and procure customers, they not unfrequently mislead the insured, by a false or
be his agent, and he alone would be responsible. On the other hand, it is well known, so well that no
erroneous statement of what the application should contain; or, taking the preparation of it into their
court would be justified in shutting its eyes to it, that insurance companies organized under the laws of
own hands, procure his signature by an assurance that it is properly drawn, and will meet the
one State, and having in that State their principal business office, send these agents all over the land,
requirements of the policy. The better opinion seems to be that, when this course is pursued, the
with directions to solicit and procure applications for policies furnishing them with printed arguments in
description of the risk should, though nominally proceeding from the insured, be regarded as the act of
favor of the value and necessity of life insurance, and of the special advantages of the corporation which
the insurers.' (Rowley vs. Empire Ins. Co., 36 N. Y., 550.)
the agent represents. They pay these agents large commissions on the premiums thus obtained, and the
policies are delivered at their hands to the assured. The agents are stimulated by letters and instructions "The modern decisions fully sustain this proposition, and they seem to us founded on reason and justice,
to activity in procuring contracts, and the party who is in this manner induced to take out a policy, rarely and meet our entire approval. This principle does not admit oral testimony to vary or contradict that
sees or knows anything about the company or its officers by whom it is issued, but looks to and relies which is in writing, but it goes upon the idea that the writing offered in evidence was not the instrument
upon the agent who has persuaded him to effect insurance as the full and complete representative of the of the party whose name is signed to it; that it was procured under such circumstances by the other side
company, in all that is said or done in making the contract. Has he not a right to so regard him? It is quite as estops that side from using it or relying on its contents; not that it may be contradicted by oral
true that the reports of judicial decisions are filled with the efforts of these companies, by their counsel, testimony, but that it may be shown by such testimony that it cannot be lawfully used against the party
to establish the doctrine that they can do all this and yet limit their responsibility for the acts of these whose name is signed to it." (See also Am. Life Ins. Co. vs. Mahone, 21 Wallace, 152.)
agents to the simple receipt of the premium and delivery of the policy, the argument being that, as to all
The defendant, upon the information given by plaintiff, and after an inspection of the automobile by its
other acts of the agent, he is the agent of the assured. This proposition is not without support in some of
examiner, having agreed that it was worth P3,000, is bound by this valuation in the absence of fraud on
the earlier decision on the subject; and, at a time when insurance companies waited for parties to come
the part of the insured. All statements of value are, of necessity, to a large extent matters of opinion, and
to them to seek assurance, or to forward applications on their own motion, the doctrine had a
it would be outrageous to hold that the validity of all valued policies must depend upon the absolute
reasonable foundation to rest upon. But to apply such a doctrine, in its full force, to the system of selling
correctness of such estimated value. As was said by the Supreme Court of the United States in the case
policies through agents, which we have described, would be a snare and a delusion, leading, as it has
of the First National Bank vs. Hartford Fire Insurance Co. (5 Otto, 673; 24 L. ed., 563), at p. 565 of the
done in numerous instances, to the grossest frauds, of which the insurance corporations receive the
Lawyer's Edition:
benefits, and the parties supposing themselves insured are the victims. The tendency of the modern
decisions in this country is steadily in the opposite direction. The powers of the agent are, prima facie, "The ordinary test of the value of property is the price it will commend in the market if offered for sale.
coextensive with the business intrusted to his care, and will not be narrowed by limitations not But that test cannot, in the very nature of the case, be applied at the time application is made for
communicated to the person with whom he deals. (Bebee vs. Ins. Co., 25 Conn., 51; Lycoming Ins. Co. vs. insurance. Men may honestly differ about the value of property, or as to what it will bring in the market;

Page 68 of 97
and such differences are often very marked among those whose special business it is to buy and sell provisions of Section 61-A of the Insurance Act (No. 2427) as amended by Act 4101. The year for
property of all kinds. The assured could do no more than estimate such value; and that, it seems, was all instituting action in court must be reckoned, therefore, from the time of appellee’s refusal to comply with
that he was required to do in this case. His duty was to deal fairly with the Company in making such
its bond. In so far, therefore, as condition eight of the bond requires action to be filed within one year
estimate. The special finding shows that he discharged that duty and observed good faith. We shall not
from the filing of the claim for loss, such stipulation contradicts the public policy expressed in Section 61-
presume that the Company, after requiring the assured in his application to give the 'estimated value,'
A of the Philippine Insurance Act. Condition eight of the bond, therefore, is null and void, and the
and then to covenant that he had stated all material facts in regard to such value, so far as known to him,
and after carrying that covenant, by express words, into the written contract, intended to abandon the appellant is not bound to comply with its provisions. Action must be brought within the statutory period

theory upon which it sought the contract, and make the absolute correctness of such estimated value a of limitation for written contracts (N.C.C., Article 1144).
condition precedent to any insurance whatever. The application, with its covenant and stipulations,
DECISION
having been made a part of the policy, that presumption cannot be indulged without imputing to the
.Company a purpose; by studied intricacy or an ingenious framing of the policy, to entrap the assured
REYES, J.B.L., J.:
into incurring obligations which, perhaps, he had no thought of assuming."

Section 163 of the Insurance Law (Act No. 2427) provides that "the effect of a valuation in a policy of fire Appeal, on points of law, against a decision of the Court of First Instance of Manila, in its Case No. 43372,

insurance is the same as in a policy of marine insurance." upholding a motion to dismiss.

By the terms of section 149 of the Act cited, the valuation in a policy of marine insurance is conclusive if At issue is the question whether or not the provision of a fidelity bond that no action shall be had or
the insured had an insurable interest and was not guilty of fraud.
maintained thereon unless commenced within one year from the making of a claim for the loss upon
We are, therefore, of the opinion and hold that plaintiff was the owner of the automobile in question and which the action is based, is valid or void, in view of Section 61-A of the Insurance Act invalidating
had an insurable interest therein; that there was no fraud on her part in procuring the insurance; that the stipulations limiting the time for commencing an action thereon to less than one year from the time the
valuation of the automobile, for the purposes of the insurance, is binding upon the defendant
cause of action accrues.
corporation, and that the judgment of the court below is, therefore, correct and must be affirmed, with
interest, the costs of this appeal to be paid by the appellant. So ordered. Material to this decision are the following facts:chanrob1es virtual 1aw library

According to the allegations of the complaint, in order to guarantee the Asingan Farmers’ Cooperative
25. ACCFA V. Alpha Insurance & Surety Inc. (24 SCRA 151, GR No. L-24566, 29 July 1968) Marketing Association, Inc. (FACOMA) against loss on account of "personal dishonesty, amounting to
larceny or estafa" of its Secretary-Treasurer, Ricardo A. Ladines, the appellee, Alpha Insurance & Surety
SYLLABUS
Company had issued, on 14 February 1958, its bond, No. P-FID-15-58, for the sum of Five Thousand

Pesos (P5,000.00) with said Ricardo Ladines as principal and the appellee as solidary surety. On the same
1. COMMERCIAL LAW; INSURANCE; FIDELITY BOND; STIPULATION SHORTENING PERIOD OF
date, the Asingan FACOMA assigned its rights to the appellant, Agricultural Credit Cooperative and
INSTITUTING ACTION AGAINST BOND. — A fidelity bond is, in effect, in the nature of a contract of
Financing Administration (ACCFA for short), with approval of the principal and the surety.
insurance against loss from misconduct and is governed by the same principles of interpretation.
Consequently, the condition of the bond limiting the period for bringing action thereon, is subject to the
Page 69 of 97
During the effectivity of the bond, Ricardo Ladines converted and misappropriated, to his personal the Insurance Act (No. 2427), as amended by Act 4101 of the pre-Commonwealth Philippine Legislature,
benefit some P11,513.22 of the FACOMA funds, of which P6,307.33 belonged to the ACCFA. Upon prescribing that —
discovery of the loss, ACCFA immediately notified in writing the surety company on 10 October 1958, and
"SEC. 61-A. — A condition, stipulation or agreement in any policy of insurance, limiting the time for
presented the proof of loss within the period fixed in the bond; but despite repeated demands the surety
commencing an action thereunder to a period of less than one year from the time when the cause of
company refused and failed to pay. Whereupon, ACCFA filed suit against appellee on 30 May 1960.
action accrues is void."
Defendant Alpha Insurance & Surety Co., Inc., (now appellee) moved to dismiss the complaint for failure
Since a "cause of action" requires, as essential elements, not only a legal right of the plaintiff and a
to state a cause of action, giving as reason that (1) the same was filed more than one year after plaintiff
correlative obligation of the defendant but also "an act or omission of the defendant in violation of said
made claim for loss, contrary to the eight condition of the bond, providing as follows:
legal right" (Maao Sugar Central v. Barrios, 79 Phil. 666), the cause of action does not accrue until the
"EIGHT. LIMITATION OF ACTION party obligated refuses, expressly or impliedly, to comply with its duty (in this case, to pay the amount of
the bond). The year for instituting action in court must be reckoned, therefore, from the time of
‘No action, suit or proceeding shall be had or maintained upon this Bond unless the same be
appellee’s refusal to comply with its bond; it can not be counted from the creditor’s filing of the claim of
commenced within one year from the time of making claim for the loss upon which such action, suit or
loss, for that does not import that the surety company will refuse to pay. In so far, therefore, as condition
proceeding is based, in accordance with the fourth section hereof.’"
eight of the bond requires action to be filed within one year from the filing of the claim for loss, such

(2) the complaint failed to show that plaintiff had filed civil or criminal action against Ladines, as required stipulation contradicts the public policy expressed in Section 61-A of the Philippine Insurance Act.

by conditions 4 and 11 of the bond; and (3) that Ladines was a necessary and indispensable party but had Condition eight of the bond, therefore, is null and void, and the appellant is not bound to comply with its

not been joined as such. provisions.

At first, the Court of First Instance denied dismissal; but, upon reconsideration, the court reversed its In Eagle Star Insurance Co. v. Chia Yu, 96 Phil. 696, 701, this Court ruled:

original stand, and dismissed the complaint on the ground that the action was filed beyond the
"It may perhaps be suggested that the policy clause relied on by the insurer for defeating plaintiff’s
contractual limitation period (Record on Appeal, pages 56-59).
action should be given the construction that would harmonize it with section 61-A of the Insurance Act

Hence, this appeal. by taking it to mean that the time given the insured for bringing his suit is twelve months after the cause
of action accrues. But the question then would be: When did the cause of action accrue? On that
We find the appeal meritorious.
question we agree with the court below that plaintiff’s cause of action did not accrue until his claim was

A fidelity bond is, in effect, in the nature of a contract of insurance against loss from misconduct, and is finally rejected by the insurance company. This is because, before such final rejection, there was no real

governed by the same principles of interpretation: Mechanics Savings Bank & Trust Co. v. Guarantee necessity for bringing suit. As the policy provides that the insured should file his claim, first, with the

Company, 68 Fed. 459; Pao Chan Wei v. Nemorosa, 103 Phil. 57. Consequently, the condition of the bond carrier and then with the insurer, he had a right to wait for his claim to be finally decided before going to

in question, limiting the period for bringing action thereon, is subject to the provisions of Section 61-A of court. The law does not encourage unnecessary litigation."

Page 70 of 97
The discouraging of unnecessary litigation must be deemed a rule of public policy, considering the house, wins the wagers. The prize is the recompense to be given by the insurer to make good the loss

unrelieved congestion in the courts. the insured has sustained.

As a consequence of the foregoing, condition eight of the Alpha bond is null and void, and action may It would be a pity then if, having lost his house, the insured were also to lose the payment he expects to

be brought within the statutory period of limitation for written contracts (New Civil Code, Article 1144). recover for such loss. Sometimes it is his fault that he cannot collect, as where there is a defect imputable
to him in the insurance contract. Conversely, the reason may be an unjust refusal of the insurer to
The case of Ang v. Pulton Fire Insurance Co., 2 SCRA 945 (31 July 1961), relied upon by the Court a quo, is
acknowledge a just obligation, as has happened many times.
no authority against the views herein expressed, since the effect of Section 61-A of the Insurance law on
the terms of the policy or contract was not there considered. In the instant case the private respondent has been sustained by the Insurance Commission in her claim
for compensation for her burned property. The petitioner is now before us to dispute the decision, 1 on
The condition of previous conviction (paragraph b, clause 4, of the contract) having been deleted by
the ground that there was no valid insurance contract at the time of the loss.
express agreement, and the surety having assumed solidary liability, the other grounds of the motion to
dismiss are equally untenable. A creditor may proceed against any one of the solidary debtors, or some The chronology of the relevant antecedent facts is as follows:
or all of them simultaneously (Article 1216, New Civil Code).
On June 7, 1981, the petitioner (hereinafter called (MICO) issued to the private respondent, Coronacion
WHEREFORE, the appealed order granting the motion to dismiss is reversed and set aside, and the Pinca, Fire Insurance Policy No. F-001-17212 on her property for the amount of P14,000.00 effective July
records are remanded to the Court of First Instance, with instructions to require defendant to answer and 22, 1981, until July 22, 1982. 2

thereafter proceed in conformity with the law and the Rules of Court. Costs against appellee.
On October 15,1981, MICO allegedly cancelled the policy for non-payment, of the premium and sent the

SO ORDERED. corresponding notice to Pinca. 3

26. Malayan Insurance Co. V. Cruz Arnaldo (154 SCRA 672, GRN L-67835, 12 Oct. 1987) On December 24, 1981, payment of the premium for Pinca was received by DomingoAdora, agent of
MICO. 4
CRUZ, J.:
On January 15, 1982, Adora remitted this payment to MICO,together with other payments. 5
When a person's house is razed, the fire usually burns down the efforts of a lifetime and forecloses hope
for the suddenly somber future. The vanished abode becomes a charred and painful memory. Where On January 18, 1982, Pinca's property was completely burned. 6

once stood a home, there is now, in the sighing wisps of smoke, only a gray desolation. The dying
embers leave ashes in the heart. On February 5, 1982, Pinca's payment was returned by MICO to Adora on the ground that her policy had
been cancelled earlier. But Adora refused to accept it. 7
For peace of mind and as a hedge against possible loss, many people now secure fire insurance. This is
an aleatory contract. By such insurance, the insured in effect wagers that his house will be burned, with
the insurer assuring him against the loss, for a fee. If the house does burn, the insured, while losing his

Page 71 of 97
In due time, Pinca made the requisite demands for payment, which MICO rejected. She then went to the Between the two dates, the court chooses to believe June 13, 1982, not only because the numbers "6-13-
Insurance Commission. It is because she was ultimately sustained by the public respondent that the 82" appear on both annexes but also because it is the date authenticated by the administrative division
petitioner has come to us for relief. of the Insurance Commission. Annex "B" is at worst self-serving; at best, it might only indicate that it was
received on June 18, 1982, by the legal department of MICO, after it had been received earlier by some
From the procedural viewpoint alone, the petition must be rejected. It is stillborn. other of its personnel on June 13, 1982. Whatever the reason for the delay in transmitting it to the legal
department need not detain us here.
The records show that notice of the decision of the public respondent dated April 5, 1982, was received
by MICO on April 10, 1982. 8 On April 25, 1982, it filed a motion for reconsideration, which was denied on Under Section 416 of the Insurance Code, the period for appeal is thirty days from notice of the decision
June 4, 1982. Notice of this denial was received by MICO on June 13, 1982, as evidenced by Annex "1"
9
of the Insurance Commission. The petitioner filed its motion for reconsideration on April 25, 1981, or
duly authenticated by the Insurance Commission. 10 The instant petition was filed with this Court on July fifteen days such notice, and the reglementary period began to run again after June 13, 1981, date of its
2, 1982. 11 receipt of notice of the denial of the said motion for reconsideration. As the herein petition was filed on
July 2, 1981, or nineteen days later, there is no question that it is tardy by four days.
The position of the petition is that the petition is governed by Section 416 0f the Insurance Code giving it
thirty days wthin which to appeal by certiorari to this Court. Alternatively, it also invokes Rule 45 of the Counted from June 13, the fifteen-day period prescribed under Rule 45, assuming it is applicable, would
Rules of Court. For their part, the public and private respondents insist that the applicable law is B.P. 129, end on June 28, 1982, or also four days from July 2, when the petition was filed.
which they say governs not only courts of justice but also quasi-judicial bodies like the Insurance
Commission. The period for appeal under this law is also fifteen days, as under Rule 45. If it was filed under B.P. 129, then, considering that the motion for reconsideration was filed on the
fifteenth day after MICO received notice of the decision, only one more day would have remained for it
The pivotal date is the date the notice of the denial of the motion for reconsideration was received by to appeal, to wit, June 14, 1982. That would make the petition eighteen days late by July 2.
MICO.
Indeed, even if the applicable law were still R.A. 5434, governing appeals from administrative bodies, the
MICO avers this was June 18, 1982, and offers in evidence its Annex "B," 12 which is a copy of the Order of petition would still be tardy. The law provides for a fixed period of ten days from notice of the denial of a
June 14, 1982, with a signed rubber-stamped notation on the upper left-hand corner that it was received seasonable motion for reconsideration within which to appeal from the decision. Accordingly, that ten-
on June 18, 1982, by its legal department. It does not indicate from whom. At the bottom, significantly, day period, counted from June 13, 1982, would have ended on June 23, 1982, making the petition filed on
there is another signature under which are the ciphers "6-13-82," for which no explanation has been July 2, 1982, nine days late.
given.
Whichever law is applicable, therefore, the petition can and should be dismissed for late filing.
Against this document, the private respodent points in her Annex "1," 13 the authenticated copy of the
same Order with a rubber-stamped notation at the bottom thereof indicating that it was received for the On the merits, it must also fail. MICO's arguments that there was no payment of premium and that the
Malayan Insurance Co., Inc. by J. Gotladera on "6-13-82." The signature may or may not habe been policy had been cancelled before the occurence of the loss are not acceptable. Its contention that the
written by the same person who signed at the bottom of the petitioner's Annex "B." claim was allowed without proof of loss is also untenable.

The petitioner relies heavily on Section 77 of the Insurance Code providing that:

Page 72 of 97
SEC. 77. An insurer is entitled to payment of the premium as soon as the thing is SEC. 306. xxx xxx xxx
exposed to the peril insured against. Notwithstanding any agreement to the contrary,
no policy or contract of insurance issued by an insurance company is valid and binding Any insurance company which delivers to an insurance agant or insurance broker a
unless and until the premium thereof has been paid, except in the case of a life or an policy or contract of insurance shall be demmed to have authorized such agent or
industrial life policy whenever the grace period provision applies. broker to receive on its behalf payment of any premium which is due on such policy or
contract of insurance at the time of its issuance or delivery or which becomes due
The above provision is not applicable because payment of the premium was in fact eventually made in thereon.
this case. Notably, the premium invoice issued to Pinca at the time of the delivery of the policy on June 7,
1981 was stamped "Payment Received" of the amoung of P930.60 on "12-24-81" by Domingo And it is a well-known principle under the law of agency that:
Adora. 14 This is important because it suggests an understanding between MICO and the insured that
such payment could be made later, as agent Adora had assured Pinca. In any event, it is not denied that Payment to an agent having authority to receive or collect payment is equivalent to

this payment was actually made by Pinca to Adora, who remitted the same to MICO. payment to the principal himself; such payment is complete when the money delivered
is into the agent's hands and is a discharge of the indebtedness owing to the
The payment was made on December 24, 1981, and the fire occured on January 18, 1982. One wonders: principal. 15
suppose the payment had been made and accepted in, say, August 1981, would the commencement date
of the policy have been changed to the date of the payment, or would the payment have retroacted to There is the petitioner's argument, however, that Adora was not authorized to accept the premium

July 22, 1981? If MICO accepted the payment in December 1981 and the insured property had not been payment because six months had elapsed since the issuance by the policy itself. It is argued that this

burned, would that policy not have expired just the same on July 22, 1982, pursuant to its original terms, prohibition was binding upon Pinca, who made the payment to Adora at her own riskl as she was bound

and not on December 24, 1982? to first check his authority to receive it. 16

It would seem from MICO's own theory, that the policy would have become effective only upon MICO is taking an inconsistent stand. While contending that acceptance of the premium payment was

payment, if accepted and so would have been valid only from December 24, 1981m but only up to July prohibited by the policy, it at the same time insists that the policy never came into force because the

22, 1981, according to the original terms. In others words, the policy would have run for only eight premium had not been paid. One surely, cannot have his cake and eat it too.

months although the premium paid was for one whole year.
We do not share MICO's view that there was no existing insurance at the time of the loss sustained by

It is not disputed that the preium was actually paid by Pinca to Adora on December 24, 1981, who Pinca because her policy never became effective for non-payment of premium. Payment was in fact

received it on behalf of MICO, to which it was remitted on January 15, 1982. What is questioned is the made, rendering the policy operative as of June 22, 1981, and removing it from the provisions of Article

validity of Pinca's payment and of Adora's authority to receive it. 77, Thereafter, the policy could be cancelled on any of the supervening grounds enumerated in Article 64
(except "nonpayment of premium") provided the cancellation was made in accordance therewith and
MICO's acknowledgment of Adora as its agent defeats its contention that he was not authorized to with Article 65.
receive the premium payment on its behalf. It is clearly provided in Section 306 of the Insurance Code
that: Section 64 reads as follows:

Page 73 of 97
SEC. 64. No policy of insurance other than life shall be cancelled by the insurer except (3) The notice must be (a) in writing, (b) mailed, or delivered to the named insured, (c) at the address
upon prior notice thereof to the insured, and no notice of cancellation shall be shown in the policy; 19
effective unless it is based on the occurrence, after the effective date of the policy, of
one or more of the following: (4) It must state (a) which of the grounds mentioned in Section 64 is relied upon and (b) that upon
written request of the insured, the insurer will furnish the facts on which the cancellation is based. 20
(a) non-payment of premium;
MICO's claims it cancelled the policy in question on October 15, 1981, for non-payment of premium. To
(b) conviction of a crime arising out of acts increasing the hazard insured against; support this assertion, it presented one of its employees, who testified that "the original of the
endorsement and credit memo" — presumably meaning the alleged cancellation — "were sent the
(c) discovery of fraud or material misrepresentation; assured by mail through our mailing section" 21However, there is no proof that the notice, assuming it
complied with the other requisites mentioned above, was actually mailed to and received by Pinca. All
(d) discovery of willful, or reckless acts or commissions increasing the hazard insured MICO's offers to show that the cancellation was communicated to the insured is its employee's testimony
against; that the said cancellation was sent "by mail through our mailing section." without more. The petitioner
then says that its "stand is enervated (sic) by the legal presumption of regularity and due performance of
(e) physical changes in the property insured which result in the property becoming
duty." 22 (not realizing perhaps that "enervated" means "debilitated" not "strengthened").
uninsurable;or

On the other hand, there is the flat denial of Pinca, who says she never received the claimed cancellation
(f) a determination by the Commissioner that the continuation of the policy would
and who, of course, did not have to prove such denial Considering the strict language of Section 64 that
violate or would place the insurer in violation of this Code.
no insurance policy shall be cancelled except upon prior notice, it behooved MICO's to make sure that
the cancellation was actually sent to and received by the insured. The presumption cited is unavailing
As for the method of cancellation, Section 65 provides as follows:
against the positive duty enjoined by Section 64 upon MICO and the flat denial made by the private
respondent that she had received notice of the claimed cancellation.
SEC. 65. All notices of cancellation mentioned in the preceding section shall be in
writing, mailed or delivered to the named insured at the address shown in the policy,
It stands to reason that if Pinca had really received the said notice, she would not have made payment on
and shall state (a) which of the grounds set forth in section sixty-four is relied upon
the original policy on December 24, 1981. Instead, she would have asked for a new insurance, effective on
and (b) that, upon written request of the named insured, the insurer will furnish the
that date and until one year later, and so taken advantage of the extended period. The Court finds that if
facts on which the cancellation is based.
she did pay on that date, it was because she honestly believed that the policy issued on June 7, 1981, was
still in effect and she was willing to make her payment retroact to July 22, 1981, its stipulated
A valid cancellation must, therefore, require concurrence of the following conditions:
commencement date. After all, agent Adora was very accomodating and had earlier told her "to call him
up any time" she was ready with her payment on the policy earlier issued. She was obviously only
(1) There must be prior notice of cancellation to the insured; 17
reciprocating in kind when she paid her premium for the period beginning July 22, 1981, and not
December 24, 1981.
(2) The notice must be based on the occurrence, after the effective date of the policy, of one or more of
the grounds mentioned;18
Page 74 of 97
MICO's suggests that Pinca knew the policy had already been cancelled and that when she paid the The last point raised by the petitioner should not pose much difficulty. The valuation fixed in fire
premium on December 24, 1981, her purpose was "to renew it." As this could not be done by the agent insurance policy is conclusive in case of total loss in the absence of fraud, 24
which is not shown here.
alone under the terms of the original policy, the renewal thereof did not legally bind MICO. which had Loss and its amount may be determined on the basis of such proof as may be offered by the insured,
not ratified it. To support this argument, MICO's cites the following exchange: which need not be of such persuasiveness as is required in judicial proceedings. 25 If, as in this case, the
insured files notice and preliminary proof of loss and the insurer fails to specify to the former all the
Q: Now, Madam Witness, on December 25th you made the alleged defects thereof and without unnecessary delay, all objections to notice and proof of loss are deemed
payment. Now, my question is that, did it not come to your mind waived under Section 90 of the Insurance Code.
that after the lapse of six (6) months, your policy was cancelled?
The certification 26 issued by the Integrated National Police, Lao-ang, Samar, as to the extent of Pinca's
A: I have thought of that but the agent told me to call him up at loss should be considered sufficient. Notably,MICO submitted no evidence to the contrary nor did it even
anytime. question the extent of the loss in its answer before the Insurance Commission. It is also worth observing
that Pinca's property was not the only building bumed in the fire that razed the commercial district of
Q: So if you thought that your policy was already intended to revive Lao-ang, Samar, on January 18, 1982. 27
cancelled policy?
There is nothing in the Insurance Code that makes the participation of an adjuster in the assessment of
A: Misleading, Your Honor. the loss imperative or indespensable, as MICO suggests. Section 325, which it cites, simply speaks of the
licensing and duties of adjusters.
Hearing Officer: The testimony of witness is that, she thought of
that. We see in this cases an obvious design to evade or at least delay the discharge of a just obligation
through efforts bordering on bad faith if not plain duplicity, We note that the motion for reconsideration
Q: I will revise the question. Now, Mrs. Witness, you stated that you
was filed on the fifteenth day from notice of the decision of the Insurance Commission and that there
thought the policy was cancelled. Now, when you made the
was a feeble attempt to show that the notice of denial of the said motion was not received on June 13,
payment of December 24, 1981, your intention was to revive the
1982, to further hinder the proceedings and justify the filing of the petition with this Court fourteen days
policy if it was already cancelled?
after June 18, 1982. We also look askance at the alleged cancellation, of which the insured and MICO's
agent himself had no knowledge, and the curious fact that although Pinca's payment was remitted to
A: Yes, to renew it. 23
MICO's by its agent on January 15, 1982, MICO sought to return it to Adora only on February 5, 1982,
after it presumably had learned of the occurrence of the loss insured against on January 18, 1982. These
A close study of the above transcript will show that Pinca meant to renew the policy if it had really been
circumstances make the motives of the petitioner highly suspect, to say the least, and cast serious doubts
already cancelled but not if it was stffl effective. It was all conditional. As it has not been shown that there
upon its candor and bona fides.
was a valid cancellation of the policy, there was consequently no need to renew it but to pay the
premium thereon. Payment was thus legally made on the original transaction and it could be, and was,
WHEREFORE, the petition is DENIED. The decision of the Insurance Commission dated April 10, 1981, and
validly received on behalf of the insurer by its agent Adora. Adora. incidentally, had not been informed of
its Order of June 4, 1981, are AFFIRMED in full, with costs against the petitioner. This decision is
the cancellation either and saw no reason not to accept the said payment.
immediately executory.

Page 75 of 97
SO ORDERED.
b) Defendant had put plaintiff and its alleged broker on notice of non-renewal earlier; and
27. UCPB General Insurance Co., Inc. V. Masagna Telemart Inc. (GRN 137172, 4 April 2001)
c) The properties covered by the said policies were burned in a fire that took place last June 13, 1992, or
before tender of premium payment."
DAVIDE, JR., C.J.:

(Record, p. 5)
In our decision of 15 June 1999 in this case, we reversed and set aside the assailed decision 1 of the Court
of Appeals, which affirmed with modification the judgment of the trial court (a) allowing Respondent to
Hence Masagana filed this case.
consign the sum of P225,753.95 as full payment of the premiums for the renewal of the five insurance
policies on Respondent’s properties; (b) declaring the replacement-renewal policies effective and binding
The Court of Appeals disagreed with Petitioner’s stand that Respondent’s tender of payment of the
from 22 May 1992 until 22 May 1993; and (c) ordering Petitioner to pay Respondent P18,645,000.00 as
premiums on 13 July 1992 did not result in the renewal of the policies, having been made beyond the
indemnity for the burned properties covered by the renewal-replacement policies. The modification
effective date of renewal as provided under Policy Condition No. 26, which states:
consisted in the (1) deletion of the trial court’s declaration that three of the policies were in force from
August 1991 to August 1992; and (2) reduction of the award of the attorney’s fees from 25% to 10% of the
26. Renewal Clause. — Unless the company at least forty five days in advance of the end of the policy
total amount due the Respondent.
period mails or delivers to the assured at the address shown in the policy notice of its intention not to
renew the policy or to condition its renewal upon reduction of limits or elimination of coverages, the
The material operative facts upon which the appealed judgment was based are summarized by the Court
assured shall be entitled to renew the policy upon payment of the premium due on the effective date of
of Appeals in its assailed decision as follows:chanrob1es virtual 1aw library
renewal.

Plaintiff [herein Respondent] obtained from defendant [herein Petitioner] five (5) insurance policies
Both the Court of Appeals and the trial court found that sufficient proof exists that Respondent, which
(Exhibits "A" to "E", Record, pp. 158-175) on its properties [in Pasay City and Manila] . . . .
had procured insurance coverage from Petitioner for a number of years, had been granted a 60 to 90-
day credit term for the renewal of the policies. Such a practice had existed up to the time the claims were
All five (5) policies reflect on their face the effectivity term: "from 4:00 P.M. of 22 May 1991 to 4:00 P.M. of
filed. Thus:
22 May 1992." On June 13, 1992, plaintiffs properties located at 2410-2432 and 2442-2450 Taft Avenue,
Fire Insurance Policy No. 34658 covering May 22, 1990 to May 22, 1991 was issued on May 7, 1990 but
Pasay City were razed by fire. On July 13, 1992, plaintiff tendered, and defendant accepted, five (5)
premium was paid more than 90 days later on August 31, 1990 under O.R. No. 4771 (Exhs. "T" and "T-1").
Equitable Bank Manager’s Checks in the total amount of P225,753.45 as renewal premium payments for
Fire Insurance Policy No. 34660 for Insurance Risk Coverage from May 22, 1990 to May 22, 1991 was
which Official Receipt Direct Premium No. 62926 (Exhibit "Q", Record, p. 191) was issued by defendant.
issued by UCPB on May 4, 1990 but premium was collected by UCPB only on July 13, 1990 or more than
On July 14, 1992, Masagana made its formal demand for indemnification for the burned insured
60 days later under O.R. No. 46487 (Exhs. "V" and "V-1"). And so were as other policies: Fire Insurance
properties. On the same day, defendant returned the five (5) manager’s checks stating in its letter (Exhibit
Policy No. 34657 covering risks from May 22, 1990 to May 22, 1991 was issued on May 7, 1990 but
"R" / "8", Record, p. 192) that it was rejecting Masagana’s claim on the following grounds:
premium therefor was paid only on July 19, 1990 under O.R. No. 46583 (Exhs. "W" and "W-1"). Fire
Insurance Policy No. 34661 covering risks from May 22, 1990 to May 22, 1991 was issued on May 3, 1990
"a) Said policies expired last May 22, 1992 and were not renewed for another term;

Page 76 of 97
but premium was paid only on July 19, 1990 under O.R. No. 46582 (Exhs. "X" and "X-1"). Fire Insurance In our decision of 15 June 1999, we defined the main issue to be "whether the fire insurance policies
Policy No. 34688 for insurance coverage from May 22, 1990 to May 22, 1991 was issued on May 7, 1990 issued by petitioner to the respondent covering the period from May 22, 1991 to May 22, 1992 . . . had
but premium was paid only on July 19, 1990 under O.R. No. 46585 (Exhs. "Y" and "Y-1"). Fire Insurance been extended or renewed by an implied credit arrangement though actual payment of premium was
Policy No. 29126 to cover insurance risks from May 22, 1989 to May 22, 1990 was issued on May 22, 1989 tendered on a later date and after the occurrence of the (fire) risk insured against." We resolved this issue
but premium therefor was collected only on July 25, 1990[sic] under O.R. No. 40799 (Exhs. "AA" and "AA- in the negative in view of Section 77 of the Insurance Code and our decisions in Valenzuela v. Court of
1"). Fire Insurance Policy No. HO/F-26408 covering risks from January 12, 1989 to January 12, 1990 was Appeals; 2 South Sea Surety and Insurance Co., Inc. v. Court of Appeals; 3 and Tibay v. Court of Appeals.
issued to Intratrade Phils. (Masagana’s sister company) dated December 10, 1988 but premium therefor 4 Accordingly, we reversed and set aside the decision of the Court of Appeals.
was paid only on February 15, 1989 under O.R. No. 38075 (Exhs. "BB" and "BB-1"). Fire Insurance Policy
No. 29128 was issued on May 22, 1989 but premium was paid only on July 25, 1989 under O.R. No. 40800 Respondent seasonably filed a motion for the reconsideration of the adverse verdict. It alleges in the
for insurance coverage from May 22, 1989 to May 22, 1990 (Exhs. "CC" and "CC-1"). Fire Insurance Policy motion that we had made in the decision our own findings of facts, which are not in accord with those of
No. 29127 was issued on May 22, 1989 but premium was paid only on July 17, 1989 under O.R. No. 40682 the trial court and the Court of Appeals. The courts below correctly found that no notice of non-renewal
for insurance risk coverage from May 22, 1989 to May 22, 1990 (Exhs. "DD" and "DD-1"). Fire Insurance was made within 45 days before 22 May 1992, or before the expiration date of the fire insurance policies.
Policy No. HO/F-29362 was issued on June 15, 1989 but premium was paid only on February 13, 1990 Thus, the policies in question were renewed by operation of law and were effective and valid on 30 June
under O.R. No. 39233 for insurance coverage from May 22, 1989 to May 22, 1990 (Exhs. "EE" and "EE-1"). 1992 when the fire occurred, since the premiums were paid within the 60- to 90-day credit term.
Fire Insurance Policy No. 26303 was issued on November 22, 1988 but premium therefor was collected
only on March 15, 1989 under O.R. NO. 38573 for insurance risks coverage from December 15, 1988 to Respondent likewise disagrees with our ruling that parties may neither agree expressly or impliedly on
December 15, 1989 (Exhs. "FF" and "FF-1"). the extension of credit or time to pay the premium nor consider a policy binding before actual payment.
It urges the Court to take judicial notice of the fact that despite the express provision of Section 77 of the
Moreover, according to the Court of Appeals the following circumstances constitute preponderant proof Insurance Code, extension of credit terms in premium payment has been the prevalent practice in the
that no timely notice of non-renewal was made by Petitioner: insurance industry. Most insurance companies, including Petitioner, extend credit terms because Section
77 of the Insurance Code is not a prohibitive injunction but is merely designed for the protection of the
(1) Defendant-appellant received the confirmation (Exhibit "11", Record, p. 350) from Ultramar parties to an insurance contract. The Code itself, in Section 78, authorizes the validity of a policy
Reinsurance Brokers that plaintiff’s reinsurance facility had been confirmed up to 67.5% only on April 15, notwithstanding non-payment of premiums.
1992 as indicated on Exhibit "11." Apparently, the notice of non-renewal (Exhibit "7," Record, p. 320) was
sent not earlier than said date, or within 45 days from the expiry dates of the policies as provided under Respondent also asserts that the principle of estoppel applies to Petitioner. Despite its awareness of
Policy Condition No. 26; (2) Defendant insurer unconditionally accepted, and issued an official receipt for, Section 77 Petitioner persuaded and induced Respondent to believe that payment of premium on the
the premium payment on July 1[3], 1992 which indicates defendant’s willingness to assume the risk 60- to 90-day credit term was perfectly alright; in fact it accepted payments within 60 to 90 days after the
despite only a 67.5% reinsurance cover[age]; and (3) Defendant insurer appointed Esteban Adjusters and due dates. By extending credit and habitually accepting payments 60 to 90 days from the effective dates
Valuers to investigate plaintiff’s claim as shown by the letter dated July 17, 1992 (Exhibit "11", Record, p. of the policies, it has implicitly agreed to modify the tenor of the insurance policy and in effect waived
254). the provision therein that it would pay only for the loss or damage in case the same occurred after
payment of the premium.

Page 77 of 97
Petitioner filed an opposition to the Respondent’s motion for reconsideration. It argues that both the trial Section 77 of the Insurance Code of 1978 provides:
court and the Court of Appeals overlooked the fact that on 6 April 1992 Petitioner sent by ordinary mail
to Respondent a notice of non-renewal and sent by personal delivery a copy thereof to Respondent’s SECTION 77. An insurer is entitled to payment of the premium as soon as the thing insured is exposed to
broker, Zuellig. Both courts likewise ignored the fact that Respondent was fully aware of the notice of the peril insured against. Notwithstanding any agreement to the contrary, no policy or contract of
non-renewal. A reading of Section 66 of the Insurance Code readily shows that in order for an insured to insurance issued by an insurance company is valid and binding unless and until the premium thereof has
be entitled to a renewal of a non-life policy, payment of the premium due on the effective date of been paid, except in the case of a life or an industrial life policy whenever the grace period provision
renewal should first be made. Respondent’s argument that Section 77 is not a prohibitive provision finds applies.
no authoritative support.
This Section is a reproduction of Section 77 of P.D. No. 612 (The Insurance Code) promulgated on 18
Upon a meticulous review of the records and reevaluation of the issues raised in the motion for December 1974. In turn, this Section has its source in Section 72 of Act No. 2427 otherwise known as the
reconsideration and the pleadings filed thereafter by the parties, we resolved to grant the motion for Insurance Act as amended by R.A. No. 3540, approved on 21 June 1963, which read:
reconsideration. The following facts, as found by the trial court and the Court of Appeals, are indeed duly
established:chanrob1es virtual 1aw library SECTION 72. An insurer is entitled to payment of premium as soon as the thing insured is exposed to the
peril insured against, unless there is clear agreement to grant the insured credit extension of the
1. For years, Petitioner had been issuing fire policies to the Respondent, and these policies were annually premium due. No policy issued by an insurance company is valid and binding unless and until the
renewed. premium thereof has been paid. (Emphasis supplied)

2. Petitioner had been granting Respondent a 60- to 90-day credit term within which to pay the It can be seen at once that Section 77 does not restate the portion of Section 72 expressly permitting an
premiums on the renewed policies. agreement to extend the period to pay the premium. But are there exceptions to Section 77?

3. There was no valid notice of non-renewal of the policies in question, as there is no proof at all that the The answer is in the affirmative.
notice sent by ordinary mail was received by Respondent, and the copy thereof allegedly sent to Zuellig
was ever transmitted to Respondent. The first exception is provided by Section 77 itself, and that is, in case of a life or industrial life policy
whenever the grace period provision applies.
4. The premiums for the policies in question in the aggregate amount of P225,753.95 were paid by
Respondent within the 60- to 90-day credit term and were duly accepted and received by Petitioner’s The second is that covered by Section 78 of the Insurance Code, which provides:
cashier.
SECTION 78. Any acknowledgment in a policy or contract of insurance of the receipt of premium is
The instant case has to rise or fall on the core issue of whether Section 77 of the Insurance Code of 1978 conclusive evidence of its payment, so far as to make the policy binding, notwithstanding any stipulation
(P.D. No. 1460) must be strictly applied to Petitioner’s advantage despite its practice of granting a 60- to therein that it shall not be binding until premium is actually paid.
90-day credit term for the payment of premiums.

Page 78 of 97
A third exception was laid down in Makati Tuscany Condominium Corporation v. Court of Appeals, 5 the payment of the premium and loss occurs before the expiration of the term, recovery on the policy
wherein we ruled that Section 77 may not apply if the parties have agreed to the payment in installments should be allowed even though the premium is paid after the loss but within the credit term.
of the premium and partial payment has been made at the time of loss. We said therein, thus:
Moreover, there is nothing in Section 77 which prohibits the parties in an insurance contract to provide a
We hold that the subject policies are valid even if the premiums were paid on installments. The records credit term within which to pay the premiums. That agreement is not against the law, morals, good
clearly show that the petitioners and private respondent intended subject insurance policies to be customs, public order or public policy. The agreement binds the parties. Article 1306 of the Civil Code
binding and effective notwithstanding the staggered payment of the premiums. The initial insurance provides:
contract entered into in 1982 was renewed in 1983, then in 1984. In those three years, the insurer
accepted all the installment payments. Such acceptance of payments speaks loudly of the insurer’s ARTICLE 1306. The contracting parties may establish such stipulations clauses, terms and conditions as
intention to honor the policies it issued to petitioner. Certainly, basic principles of equity and fairness they may deem convenient, provided they are not contrary to law, morals, good customs, public order,
would not allow the insurer to continue collecting and accepting the premiums, although paid on or public policy.
installments, and later deny liability on the lame excuse that the premiums were not prepaid in full.
Finally in the instant case, it would be unjust and inequitable if recovery on the policy would not be
Not only that. In Tuscany, we also quoted with approval the following pronouncement of the Court of permitted against Petitioner, which had consistently granted a 60- to 90-day credit term for the payment
Appeals in its Resolution denying the motion for reconsideration of its decision: of premiums despite its full awareness of Section 77. Estoppel bars it from taking refuge under said
Section, since Respondent relied in good faith on such practice. Estoppel then is the fifth exception to
While the import of Section 77 is that prepayment of premiums is strictly required as a condition to the Section 77.
validity of the contract, We are not prepared to rule that the request to make installment payments duly
approved by the insurer would prevent the entire contract of insurance from going into effect despite WHEREFORE, the Decision in this case of 15 June 1999 is RECONSIDERED and SET ASIDE, and a new one
payment and acceptance of the initial premium or first installment. Section 78 of the Insurance Code in is hereby entered DENYING the instant petition for failure of Petitioner to sufficiently show that a
effect allows waiver by the insurer of the condition of prepayment by making an acknowledgment in the reversible error was committed by the Court of Appeals in its challenged decision, which is hereby
insurance policy of receipt of premium as conclusive evidence of payment so far as to make the policy AFFIRMED in toto.
binding despite the fact that premium is actually unpaid. Section 77 merely precludes the parties from
stipulating that the policy is valid even if premiums are not paid, but does not expressly prohibit an No pronouncement as to cost.
agreement granting credit extension, and such an agreement is not contrary to morals, good customs,
public order or public policy (De Leon, The Insurance Code, p. 175). So is an understanding to allow SO ORDERED.
insured to pay premiums in installments not so prescribed. At the very least, both parties should be
deemed in estoppel to question the arrangement they have voluntarily accepted. .
Separate Opinions
By the approval of the aforequoted findings and conclusion of the Court of Appeals, Tuscany has
provided a fourth exception to Section 77, namely, that the insurer may grant credit extension for the VITUG, J.:
payment of the premium. This simply means that if the insurer has granted the insured a credit term for

Page 79 of 97
An essential characteristic of an insurance is its being synallagmatic, a highly reciprocal contract where been paid, except in the case of a life or an industrial life policy whenever the grace period provision
the rights and obligations of the parties correlate and mutually correspond. The insurer assumes the risk applies."
of loss which an insured might suffer in consideration of premium payments under a risk-distributing
device. Such assumption of risk is a component of a general scheme to distribute actual losses among a This provision amended Section 72 of the then Insurance Act by deleting the phrase, "unless there is a
group of persons, bearing similar risks, who make ratable contributions to a fund from which the losses clear agreement to grant the insured credit extension of the premium due," and adding at the beginning
incurred due to exposures to the peril insured against are assured and compensated. of the second sentence the phrase," [n]otwithstanding any agreement to the contrary." Commenting on
the new provision, Dean Hernando B. Perez states:
It is generally recognized that the business of insurance is one imbued with public interest. 1 For the
general good and mutual protection of all the parties, it is aptly subjected to regulation and control by "Under the former rule, whenever the insured was granted credit extension of the premium due or given
the State by virtue of an exercise of its police power. 2 The State may regulate in various respects the a period of time to pay the premium on the policy issued, such policy was binding although premiums
relations between the insurer and the insured, including the internal affairs of an insurance company, had not been paid (Section 72, Insurance Act; 6 Couch 2d. 67). This rule was changed when the present
without being violative of due process. 3 provision eliminated the portion concerning credit agreement, and added the phrase ‘notwithstanding
any agreement to the contrary’ which precludes the parties from stipulating that the policy is valid even if
A requirement imposed by way of State regulation upon insurers is the maintenance of an adequate premiums are not paid. Hence, under the present law, the policy is not valid and binding unless and until
legal reserve in favor of those claiming under their policies. 4 The law generally mandates that insurance the premium is paid (Arce v. Capital Insurance & Surety Co., Inc., 117 SCRA 63). If the insurer wants to
companies should retain an amount sufficient to guarantee the security of its policyholders in the remote favor the insured by making the policy binding notwithstanding the non-payment of premium, a mere
future, as well as the present, and to cover any contingencies that may arise or may be fairly anticipated. credit agreement would not be sufficient. The remedy would be for the insurer to acknowledge in the
The integrity of this legal reserve is threatened and undermined if a credit arrangement on the payment policy that premiums were paid although they were not, in which case the policy becomes binding
of premium were to be sanctioned. Calculations and estimations of liabilities under the risk insured because such acknowledgment is a conclusive evidence of payment of premium (Section 78). Thus, the
against are predicated on the basis of the payment of premiums, the vital element that establishes the Supreme Court took note that under the present law, Section 77 of the Insurance Code of 1978 has
juridical relation between the insured and the insurer. By legislative fiat, any agreement to the contrary deleted the clause ‘unless there is a clear agreement to grant the insured credit extension of the
notwithstanding, the payment of premium is a condition precedent to, and essential for, the premium due’ (Velasco v. Apostol, 173 SCRA 228)." 6
efficaciousness of the insurance contract, except (a) in case of life or industrial life insurance where a
grace period applies, or (b) in case of a written acknowledgment by the insurer of the receipt of By weight of authority, estoppel cannot create a contract of insurance, 7 neither can it be successfully
premium, such as by a deposit receipt, the written acknowledgment being conclusive evidence of the invoked to create a primary liability, 8 nor can it give validity to what the law so proscribes as a matter of
premium payment so far as to make the policy binding. 5 public policy. 9 So essential is the premium payment to the creation of the vinculum juris between the
insured and the insurer that it would be doubtful to have that payment validly excused even for a
Section 77 of the Insurance Code provides: fortuitous event. 10

"SECTION 77. An insurer is entitled to payment of the premium as soon as the thing insured is exposed The law, however, neither requires for the establishment of the juridical tie, nor measures the strength of
to the peril insured against. Notwithstanding any agreement to the contrary, no policy or contract of such tie by, any specific amount of premium payment. A part payment of the premium, if accepted by
insurance issued by an insurance company is valid and binding unless and until the premium thereof has the insurer, can thus perfect the contract and bring the parties into an obligatory relation. 11 Such a

Page 80 of 97
payment puts the contract into full binding force, not merely pro tanto, thereby entitling and obligating terms of the policy to give immediate written notice of loss. This must be complied with in the utmost
the parties by their agreement. Hence, in case of loss, full recovery less the unpaid portion of the good faith.
premium (by the operative act of legal compensation), can be had by the insured and, correlatively, if no
loss occurs the insurer can demand the payment of the unpaid balance of the premium. 12 Another badge of fraud is that respondent deviated from its previous practice of coursing its premium
payments through its brokers. This time, respondent Masagana went directly to petitioner and paid
In the instant case, no juridical tie appears to have been established under any of the situations through its cashier with manager’s checks. Naturally, the cashier routinely accepted the premium
hereinabove discussed.chanrob1es virtua1 1aw 1ibrary payment because he had no written notice of the occurrence of the fire. Such fact was concealed by the
insured and not revealed to petitioner at the time of payment.
WHEREFORE, I vote to deny the motion for reconsideration.
Indeed, if as contended by respondent, there was a clear agreement regarding the grant of a credit
Melo, J., concurs. extension, respondent would have given immediate written notice of the fire that razed the property. This
clearly showed respondent’s attempt to deceive petitioner into believing that the subject property still
PARDO, J., dissenting: existed and the risk insured against had not happened.

The majority resolved to grant respondent’s motion for reconsideration of the Court’s decision Second: The claim for insurance benefits must fall as well because the failure to give timely written notice
promulgated on June 15, 1999. By this somersault, petitioner must now pay respondent’s claim for of the fire was a material misrepresentation affecting the risk insured against.
insurance proceeds amounting to P18,645,000.00, exclusive of interests, plus 25% of the amount due as
attorney’s fees, P25,000.00 as litigation expenses, and costs of suit, covering its Pasay City property razed Section 1 of the policy provides:
by fire. What an undeserved largess! Indeed, an unjust enrichment at the expense of petitioner; even the
award of attorney’s fees is bloated to 25% of the amount due. "All benefits under the policy shall be forfeited if the claim be in any respect fraudulent, or if any false
declaration be made or used in support thereof, or if any false declaration be made or used in support
We cannot give our concurrence. We beg to dissent. We find respondent’s claim to be fraudulent: thereof, or if any fraudulent means or devices are used by the insured or any one acting on his behalf to
obtain any benefit under the policy." 2
First: Respondent Masagana surreptitiously tried to pay the overdue premiums before giving written
notice to petitioner of the occurrence of the fire that razed the subject property. This failure to give In the factual milieu, the purported practice of giving 60 to 90-day credit extension for payment of
notice of the fire immediately upon its occurrence blatantly showed the fraudulent character of its claim. premiums was a disputed fact. But it is a given fact that the written notice of loss was not immediately
The fire totally destroyed the property on June 13, 1992; the written notice of loss was given only more given. It was given only the day after the attempt to pay the delayed premiums.
than a month later, on July 14, 1992, the day after respondent surreptitiously paid the overdue premiums.
Respondent very well knew that the policy was not renewed on time. Hence, the surreptitious attempt to At any rate, the purported credit was a mere verbal understanding of the respondent Masagana of an
pay overdue premiums. Such act revealed a reprehensible disregard of the principle that insurance is a agreement between the insurance company (petitioner) and the insurance brokers of respondent
contract uberrima fides, the most abundant good faith. 1 Respondent is required by law and by express Masagana. The president of respondent Masagana admitted that the insurance policy did not contain
any proviso pertaining to the grant of credit within which to pay the premiums. Respondent Masagana

Page 81 of 97
merely deduced that a credit agreement existed based on previous years’ practice that they had of "Q: You also identified several receipts; here; official receipts issued by UCPB General Insurance
delayed payments accepted by the insurer as reflected on the face of the receipts issued by UCPB Company, Inc., which has been previously marked as Exhibits "F", "G", "H", "I", and "J" for the plaintiff; is
evidencing the payment of premiums.chanrob1es virtua1 1aw 1ibrary that correct?
"Q: You also claim that you have 60 to 90 days credit arrangement with UCPB; is that correct?,
A: Yes.
A: Yes, ma’am.
Q: And, you would agree with me that the dates indicated in these particular Official Receipts (O. R.),
Q: I’m showing to you the policy which had previously been marked in evidence as Exhibit "A", "B", "C", merely indicated the dates when UCPB General Insurance Company issued these receipts? Do you admit
"D", & "E" ‘ for the plaintiff and likewise, marked as exhibits "1", "2", "3", "4", & "5" for the defendant. that, Mr. Witness?
Could you show us, Mr. witness where in these policies does it show that you are actually given 60 to 90
days credit arrangement with UCPB? A: That was written in the receipts.

A: Well, it’s verbal with your company, and Ansons Insurance Brokerage. It is not written. Q: But, you would also agree that this did not necessarily show the dates when you actually forwarded
the checks to your broker, Anson Insurance Agency, for payment to UCPB General Insurance Co. Inc.,
Q: It is not written in the policy? isn’t it?

A: Yes. A: The actual support of this would be the cash voucher of the company, Masagana Telamart Inc., the
date when they picked up the check from the company.
Q: You merely have verbal agreement with Ansons Insurance Brokerage?
Q: And are these cash voucher with you?
A: Yes; as shown in our mode of payment; in our vouchers and the receipts issued by the insurance
company." 3 A: I don’t know if it is in the folder or in our folder, now.

It must be stressed that a verbal understanding of respondent Masagana cannot amend an insurance Q: So, you are not certain, whether or not you actually delivered the checks covered by these Official
policy. In insurance practice, amendments or even corrections to a policy are done by written Receipts to UCPB General Insurance, on the dates indicated?
endorsements or tickets appended to the policy.
A: I would suppose it is few days earlier, when they picked up the payment in our office." 4
However, the date on the face of the receipts does not refer to the date of actual remittance by
respondent Masagana to UCPB of the premium payments, but merely to the date of remittance to UCPB Hence, what has been established was the grant of credit to the insurance brokers, not to the assured.
of the premium payments by the insurance brokers of respondent Masagana. The insurance company recognized the payment to the insurance brokers as payment to itself, though
the actual remittance of the premium payments to the principal might be made later. Once payment of

Page 82 of 97
premiums is made to the insurance broker, the assured would be covered by a valid and binding Hence, because of respondent’s failure to pay the premiums prior to the occurrence of the fire insured
insurance policy, provided the loss occurred after payment to the broker has been made. against, no valid and binding insurance policy was created to cover the loss and destruction of the
property. The fire took place on June 13, 1992, twenty-two (22) days after the expiration of the policy of
Assuming arguendo that the 60 to 90 day-credit-term has been agreed between the parties, respondent fire insurance. The tender of payment of premiums was made only thirty (30) days after the occurrence of
could not still invoke estoppel to back up its claim. "Estoppel is unavailing in this case," 5 thus spoke the the fire, or on July 13, 1992. Respondent Masagana did not give immediate notice to petitioner of the fire
Supreme Court through the pen of Justice Hilario G. Davide, Jr., now Chief Justice. Mutatis mutandi, he as it occurred as required in the insurance policy. Respondent Masagana tried to tender payment of the
may well be speaking of this case. He added that" [E]stoppel cannot give validity to an act that is premiums overdue surreptitiously before giving notice of the occurrence of the fire. More importantly,
prohibited by law or against public policy." 6 The actual payment of premiums is a condition precedent the parties themselves expressly stipulated that the insurance policy would not be binding on the insurer
to the validity of an insurance contract other than life insurance policy. 7 Any agreement to the contrary unless the premiums thereon had been paid in full. Section 2 of the policy provides:
is void as against the law and public policy. Section 77 of the Insurance Code provides:
"2. This policy including any renewal and/or endorsement thereon is not in force until the premium has
"An insurer is entitled to payment of the premium as soon as the thing insured is exposed to the peril been fully paid and duly receipted by the Company in the manner provided therein.
insured against. Notwithstanding any agreement to the contrary, no policy or contract of insurance
issued by an insurance company is valid and binding unless and until the premium thereof has been "Any supplementary agreement seeking to amend this condition prepared by agent, broker or company
paid, except in the case of a life or an industrial life policy whenever the grace period provision applies." official, shall be deemed invalid and of no effect.
[Emphasis supplied]
"No payment in respect of any premium shall be deemed to be payment to the Company unless a
An incisive reading of the afore-cited provision would show that the emphasis was on the conclusiveness printed form of receipt for the same signed by an Official or duly appointed Agent of the Company shall
of the acknowledgment in the policy of the receipt of premium, notwithstanding the absence of actual have been given to the Insured, except when such printed receipt is not available at the time of payment
payment of premium, because of estoppel. Under the doctrine of estoppel, an admission or and the company or its representative accepts the premium in which case a temporary receipt other than
representation is rendered conclusive upon the person making it, and cannot be denied or disproved as the printed form may be issued in lieu thereof. "Except only on those specific cases where corresponding
against the person relying thereon. "A party may not go back on his own acts and representations to the rules and regulations which now we are or may hereafter be in force provide for the payment of the
prejudice of the other party who relied upon them." 8 stipulated premiums in periodic installments at fixed percentages, it is hereby declared, agreed and
warranted that this policy shall be deemed effective valid and binding upon the Company when the
This is the only case of estoppel which the law considers a valid exception to the mandatory requirement premiums thereof have actually been paid in full and duly acknowledged in a receipt signed by any
of pre-payment of premium. The law recognized that the contracting parties, in entering a contract of authorized official or representative/agent of the Company in such manner as provided herein." 9
insurance, are free to enter into stipulations and make personal undertakings so long as they are not [Emphasis supplied]
contrary to law or public policy. However, the law is clear in providing that the acknowledgment must be
contained in the policy or contract of insurance. Anything short of it would not fall under the exception Thus, the insurance policy, including any renewal thereof or any endorsements thereon shall not come in
so provided in Section 78. force until the premiums have been fully paid and duly received by the insurance Company. No payment
in respect of any premiums shall be deemed to be payment to the Insurance Company unless a printed

Page 83 of 97
form of receipt for the same signed by an Official or duly appointed Agent of the Company shall be
given to the insured. There is no dispute that like in any other contract, the parties to a contract of insurance enjoy the
freedom to stipulate on the terms and conditions that will govern their agreement so long as they are
The case of Tibay v. Court of Appeals 10 is in point. The issue raised therein was: "May a fire insurance not contrary to law, morals, good customs, public order or public policy. However, the agreement
policy be valid, binding and enforceable upon mere partial payment of premium?" In the said case, containing such terms and conditions must be clear and definite.
Fortune Life and General Insurance Co., Inc. issued Fire Insurance Policy No. 136171 in favor of Violeta R.
Tibay and/or Nicolas Roraldo, on a two-storey residential building located at 5855 Zobel Street, Makati In the case at bar, there was no clear and definite agreement between petitioner and respondent on the
City, together with all the personal effects therein, The insurance was for P600,000.00, covering the grant of a credit extension; neither was there partial payment of premiums for petitioner to invoke the
period from 23 January 1987 to 23 January 1988. On 23 January 1987, of the total premium of P2,983.50, exceptional doctrine in Tuscany.
Violeta Tibay only paid P600.00, thus leaving a substantial balance unpaid. On March 8, 1987, the insured
building was completely destroyed by fire. Two days later, or on 10 March 1987, Violeta Tibay paid the Hence, the circumstances in the above cited case are totally different from the case at bar, and
balance of the premium. On the same day, she filed with Fortune a claim for the proceeds of the fire consequently, not applicable herein.
insurance policy.
Insurance is an aleatory contract whereby one undertakes for a consideration to indemnify another
In denying the claim of insurance, the Court ruled that "by express agreement of the parties, no vinculum against loss, damage or liability arising from an unknown or contingent event. 14 The consideration is the
juris or bond of law was to be established until full payment was effected prior to the occurrence of the premium, which must be paid at the time and in the manner specified in the policy, and if not so paid,
risk insured against. 11 As expressly stipulated in the contract, full payment must be made before the risk the policy will lapse and be forfeited by its own terms. 15
occurs for the policy to be considered effective and in force. "No vinculum juris whereby the insurer
bound itself to indemnify the assured according to law ever resulted from the fractional payment of With regard to the contention that the absence of notice of non-renewal of the policy resulted to the
premium." 12 automatic renewal of the insurance policy, we find the contention untenable. As above discussed, the law
provides that only upon payment of the insurance premium will the insurance policy bind the insurer to
The majority cited the case of Makati Tuscany Condominium Corp. v. Court of Appeals 13 to support the the peril insured against and hold it liable under the policy in case of loss.
contention that the insurance policies subject of the instant case were valid and effective. However, the
factual situation in that case was different from the case at bar. Even in the absence of notice of non-renewal, the assured would be bound by the law that a non life
insurance policy takes effect only on the date payment of the premium was made.
In Tuscany, the Court held that the insurance policies were valid and binding because there was partial
payment of the premiums and a clear understanding between the parties that they had intended the Verily, it is elemental law that the payment of premium is a mandatory requisite to make the policy of
insurance policies to be binding and effective notwithstanding the staggered payment of the premiums. insurance effective. If the premium is not paid in the manner prescribed in the policy as intended by the
On the basis of equity and fairness, the Court ruled that there was a perfected contract of insurance parties, the policy is void and ineffective. 16
upon the partial payment of the premiums, notwithstanding the provisions of Section 77 to the contrary.
The Court would not allow the insurer to continue collecting and accepting the premiums, although paid
on installments, and later deny liability on the lame excuse that the premiums were not prepaid in full.

Page 84 of 97
Basically a contract of indemnity, an insurance contract is the law between the parties. Its terms and On April 19, 1961, or before the expiration of the one-year term, plaintiff notified defendant, through its
conditions constitute the measure of the insurer’s liability and compliance therewith is a condition Indorsement No. F-6963/61, of the cancellation of the Policy allegedly upon request of defendant. 1 The
precedent to the insured’s right to recovery from the insurer. 17 latter has denied having made such a request. In said Indorsement, plaintiff credited defendant with the
amount of P3,110.25 for the unexpired period of 94 days, and claimed the balance of P7,483.11
IN VIEW WHEREOF, I vote to DENY the respondent’s motion for reconsideration, for lack of merit. representing ,learned premium from July 21, 1960 to 18th April 1961 or, say 271 days." On July 6, 1961,
plaintiff demanded in writing for the payment of said amount. 2 Defendant, through counsel, disclaimed
any liability in its reply- letter of August 15, 1961, contending, in essence, that it need not pay premium
"because the Insurer did not stand liable for any indemnity during the period the premiums were not
paid." 3
28. Phil Phoenix Surety & Ins Co V. Woodworks Inc. (GRN L-25317, 6 Aug. 1979)

On January 30, 1962, plaintiff commenced action in the Court of First Instance of Manila, Branch IV (Civil
MELENCIO-HERRERA, J.:
Case No. 49468), to recover the amount of P7,483.11 as "earned premium." Defendant controverted
basically on the theory that its failure "to pay the premium after the issuance of the policy put an end to
This case was certified to this Tribunal by the Court of Appeals in its Resolution of October 4, 1965 on a
the insurance contract and rendered the policy unenforceable." 4
pure question of law and "because the issues raised are practically the same as those in CA-G.R. No.
32017-R" between the same parties, which case had been forwarded to us on April 1, 1964. The latter
On September 13, 1962, judgment was rendered in plaintiff's favor "ordering defendant to pay plaintiff
case, "Philippine Phoenix Surety & Insurance Inc. vs. Woodworks, Inc.," docketed in this Court as L-22684,
the sum of P7,483.11, with interest thereon at the rate of 6%, per annum from January 30, 1962, until the
was decided on August 31, 1967 and has been reported in 20 SCRA 1270.
principal shall have been fully paid, plus the sum of P700.00 as attorney's fees of the plaintiff, and the
costs of the suit." From this adverse Decision, defendant appealed to the Court of Appeals which, as
Specifically, this action is for recovery of unpaid premium on a fire insurance policy issued by plaintiff,
heretofore stated, certified the case to us on a question of law.
Philippine Phoenix Surety & Insurance Company, in favor of defendant Woodworks, Inc.

The errors assigned read:


The following are the established facts:

1. The lower court erred in sustaining that Fire Insurance Policy, Exhibit A, was a binding contract even if
On July 21, 1960, upon defendant's application, plaintiff issued in its favor Fire Insurance Policy No. 9749
the premium stated in the policy has not been paid.
for P500,000.00 whereby plaintiff insured defendant's building, machinery and equipment for a term of
one year from July 21, 1960 to July 21, 1961 against loss by fire. The premium and other charges including
2. That the lower court erred in sustaining that the premium in Insurance Policy, Exhibit B, became an
the margin fee surcharge of P590.76 and the documentary stamps in the amount of P156.60 affixed on
obligation which was demandable even after the period in the Policy has expired.
the Policy, amounted to P10,593.36.

3. The lower court erred in not deciding that a premium not paid is not a debt enforceable by action of
It is undisputed that defendant did not pay the premium stipulated in the Policy when it was issued nor
the insurer.
at any time thereafter.

We find the appeal meritorious.

Page 85 of 97
Insurance is "a contract whereby one undertakes for a consideration to indemnify another against loss, Clearly, the Policy provides for pre-payment of premium. Accordingly; "when the policy is tendered the
damage or liability arising from an unknown or contingent event." 5 The consideration is the "premium". insured must pay the premium unless credit is given or there is a waiver, or some agreement obviating
"The premium must be paid at the time and in the way and manner specified in the policy and, if not so the necessity for prepayment." 7 To constitute an extension of credit there must be a clear and express
paid, the policy will lapse and be forfeited by its own terms." 6 agreement therefor." 8

The provisions on premium in the subject Policy read: From the Policy provisions, we fail to find any clear agreement that a credit extension was accorded
defendant. And even if it were to be presumed that plaintiff had extended credit from the circumstances
THIS POLICY OF INSURANCE WITNESSETH, THAT in consideration of - MESSRS. WOODWORKS, INC. - of the unconditional delivery of the Policy without prepayment of the premium, yet it is obvious that
hereinafter called the Insured, paying to the PHILIPPINE PHOENIX SURETY AND INSURANCE, INC., defendant had not accepted the insurer's offer to extend credit, which is essential for the validity of such
hereinafter called the Company, the sum of - PESOS NINE THOUSAND EIGHT HUNDRED FORTY SIX agreement.
ONLY - the Premium for the first period hereinafter mentioned. ...
An acceptance of an offer to allow credit, if one was made, is as essential to make a valid agreement for
xxx xxx xxx credit, to change a conditional delivery of an insurance policy to an unconditional delivery, as it is to
make any other contract. Such an acceptance could not be merely a mental act or state of mind, but
THE COMPANY HEREBY AGREES with the Insured ... that if the Property above described, or any part would require a promise to pay made known in some manner to defendant. 9
thereof, shall be destroyed or damaged by Fire or Lightning after payment of Premium, at any time
between 4:00 o'clock in the afternoon of the TWENTY FIRST day of JULY One Thousand Nine Hundred In this respect, the instant case differs from that involving the same parties entitled Philippine Phoenix
and SIXTY and 4:00 o'clock in the afternoon of the TWENTY FIRST day of JULY One Thousand Nine Surety & Insurance Inc. vs. Woodworks, Inc., 10 where recovery of the balance of the unpaid premium was
Hundred and SIXTY ONE. ... (Emphasis supplied) allowed inasmuch as in that case "there was not only a perfected contract of insurance but a partially
performed one as far as the payment of the agreed premium was concerned." This is not the situation
Paragraph "2" of the Policy further contained the following condition: obtaining here where no partial payment of premiums has been made whatsoever.

2. No payment in respect of any premium shall be deemed to be payment to the Company unless a Since the premium had not been paid, the policy must be deemed to have lapsed.
printed form of receipt for the same signed by an Official or duly-appointed Agent of the Company shall
have been given to the Insured. The non-payment of premiums does not merely suspend but put, an end to an insurance contract, since
the time of the payment is peculiarly of the essence of the contract. 11
Paragraph "10" of the Policy also provided:
... the rule is that under policy provisions that upon the failure to make a payment of a premium or
10. This insurance may be terminated at any time at the request of the Insured, in which case the assessment at the time provided for, the policy shall become void or forfeited, or the obligation of the
Company will retain the customary short period rate for the time the policy has been in force. This insurer shall cease, or words to like effect, because the contract so prescribes and because such a
insurance may also at any time be terminated at the option of the Company, on notice to that effect stipulation is a material and essential part of the contract. This is true, for instance, in the case of life,
being given to the Insured, in which case the Company shall be liable to repay on demand a ratable health and accident, fire and hail insurance policies. 12
proportion of the premium for the unexpired term from the date of the cancelment.

Page 86 of 97
In fact, if the peril insured against had occurred, plaintiff, as insurer, would have had a valid defense unless and until the premium thereof has been paid, except in the case of a life or an
against recovery under the Policy it had issued. Explicit in the Policy itself is plaintiff's agreement to industrial life policy whenever the grace period provision applies.
indemnify defendant for loss by fire only "after payment of premium," supra. Compliance by the insured
with the terms of the contract is a condition precedent to the right of recovery. Sometime in early 1982, private respondent American Home Assurance Co. (AHAC), represented by
American International Underwriters (Phils.), Inc., issued in favor of petitioner Makati Tuscany
The burden is on an insured to keep a policy in force by the payment of premiums, rather than on the Condominium Corporation (TUSCANY) Insurance Policy No. AH-CPP-9210452 on the latter's building
insurer to exert every effort to prevent the insured from allowing a policy to elapse through a failure to and premises, for a period beginning 1 March 1982 and ending 1 March 1983, with a total premium of
make premium payments. The continuance of the insurer's obligation is conditional upon the payment of P466,103.05. The premium was paid on installments on 12 March 1982, 20 May 1982, 21 June 1982 and 16
premiums, so that no recovery can be had upon a lapsed policy, the contractual relation between the November 1982, all of which were accepted by private respondent.
parties having ceased. 13

On 10 February 1983, private respondent issued to petitioner Insurance Policy No. AH-CPP-9210596,
Moreover, "an insurer cannot treat a contract as valid for the purpose of collecting premiums and invalid which replaced and renewed the previous policy, for a term covering 1 March 1983 to 1 March 1984. The
for the purpose of indemnity." 14 premium in the amount of P466,103.05 was again paid on installments on 13 April 1983, 13 July 1983, 3
August 1983, 9 September 1983, and 21 November 1983. All payments were likewise accepted by private
The foregoing findings are buttressed by section 77 of the Insurance Code (Presidential Decree No. 612, respondent.
promulgated on December 18, 1974), which now provides that no contract of insurance issued by an
insurance company is valid and binding unless and until the premium thereof has been paid, On 20 January 1984, the policy was again renewed and private respondent issued to petitioner Insurance
notwithstanding any agreement to the contrary. Policy No. AH-CPP-9210651 for the period 1 March 1984 to 1 March 1985. On this renewed policy,
petitioner made two installment payments, both accepted by private respondent, the first on 6 February
WHEREFORE, the judgment appealed from is reversed, and plaintiff's complaint hereby dismissed. 1984 for P52,000.00 and the second, on 6 June 1984 for P100,000.00. Thereafter, petitioner refused to pay
the balance of the premium.
29. Makati Tuscany Condominium Corp V CA (GRN 95546, 6 Nov. 1992)
Consequently, private respondent filed an action to recover the unpaid balance of P314,103.05 for
BELLOSILLO, J.: Insurance Policy No. AH-CPP-9210651.

This case involves a purely legal question: whether payment by installment of the premiums due on an In its answer with counterclaim, petitioner admitted the issuance of Insurance Policy No. AH-CPP-
insurance policy invalidates the contract of insurance, in view of Sec. 77 of P.D. 612, otherwise known as 9210651. It explained that it discontinued the payment of premiums because the policy did not contain a
the Insurance Code, as amended, which provides: credit clause in its favor and the receipts for the installment payments covering the policy for 1984-85, as
well as the two (2) previous policies, stated the following reservations:
Sec. 77. An insurer is entitled to the payment of the premium as soon as the thing is
exposed to the peril insured against. Notwithstanding any agreement to the contrary, 2. Acceptance of this payment shall not waive any of the company rights to deny
no policy or contract of insurance issued by an insurance company is valid and binding liability on any claim under the policy arising before such payments or after the
expiration of the credit clause of the policy; and

Page 87 of 97
3. Subject to no loss prior to premium payment. If there be any loss such is not installments, and there is no pretense that the parties never envisioned to make the
covered. insurance contract binding between them. It was renewed for two succeeding years,
the second and third policies being a renewal/replacement for the previous one. And
Petitioner further claimed that the policy was never binding and valid, and no risk attached to the policy. the insured never informed the insurer that it was terminating the policy because the
It then pleaded a counterclaim for P152,000.00 for the premiums already paid for 1984-85, and in its terms were unacceptable.
answer with amended counterclaim, sought the refund of P924,206.10 representing the premium
payments for 1982-85. While it may be true that under Section 77 of the Insurance Code, the parties may not
agree to make the insurance contract valid and binding without payment of premiums,
After some incidents, petitioner and private respondent moved for summary judgment. there is nothing in said section which suggests that the parties may not agree to allow
payment of the premiums in installment, or to consider the contract as valid and
On 8 October 1987, the trial court dismissed the complaint and the counterclaim upon the following binding upon payment of the first premium. Otherwise, we would allow the insurer to
findings: renege on its liability under the contract, had a loss incurred (sic) before completion of
payment of the entire premium, despite its voluntary acceptance of partial payments, a
While it is true that the receipts issued to the defendant contained the aforementioned
result eschewed by a basic considerations of fairness and equity.
reservations, it is equally true that payment of the premiums of the three
aforementioned policies (being sought to be refunded) were made during the lifetime To our mind, the insurance contract became valid and binding upon payment of the
or term of said policies, hence, it could not be said, inspite of the reservations, that no first premium, and the plaintiff could not have denied liability on the ground that
risk attached under the policies. Consequently, defendant's counterclaim for refund is payment was not made in full, for the reason that it agreed to accept installment
not justified. payment. . . . 3

As regards the unpaid premiums on Insurance Policy No. AH-CPP-9210651, in view of Petitioner now asserts that its payment by installment of the premiums for the insurance policies for 1982,
the reservation in the receipts ordinarily issued by the plaintiff on premium payments 1983 and 1984 invalidated said policies because of the provisions of Sec. 77 of the Insurance Code, as
the only plausible conclusion is that plaintiff has no right to demand their payment amended, and by the conditions stipulated by the insurer in its receipts, disclaiming liability for loss for
after the lapse of the term of said policy on March 1, 1985. Therefore, the defendant occurring before payment of premiums.
was justified in refusing to pay the same. 1
It argues that where the premiums is not actually paid in full, the policy would only be effective if there is
Both parties appealed from the judgment of the trial court. Thereafter, the Court of Appeals rendered a an acknowledgment in the policy of the receipt of premium pursuant to Sec. 78 of the Insurance Code.
decision 2modifying that of the trial court by ordering herein petitioner to pay the balance of the The absence of an express acknowledgment in the policies of such receipt of the corresponding
premiums due on Policy No. AH-CPP-921-651, or P314,103.05 plus legal interest until fully paid, and premium payments, and petitioner's failure to pay said premiums on or before the effective dates of said
affirming the denial of the counterclaim. The appellate court thus explained — policies rendered them invalid. Petitioner thus concludes that there cannot be a perfected contract of
insurance upon mere partial payment of the premiums because under Sec. 77 of the Insurance Code, no
The obligation to pay premiums when due is ordinarily as indivisible obligation to pay
contract of insurance is valid and binding unless the premium thereof has been paid, notwithstanding
the entire premium. Here, the parties herein agreed to make the premiums payable in

Page 88 of 97
any agreement to the contrary. As a consequence, petitioner seeks a refund of all premium payments The reliance by petitioner on Arce vs. Capital Surety and Insurance
made on the alleged invalid insurance policies. Co. 5 is unavailing because the facts therein are substantially different from those in the case at bar.
In Arce, no payment was made by the insured at all despite the grace period given. In the case before Us,
We hold that the subject policies are valid even if the premiums were paid on installments. The records petitioner paid the initial installment and thereafter made staggered payments resulting in full payment
clearly show that petitioner and private respondent intended subject insurance policies to be binding and of the 1982 and 1983 insurance policies. For the 1984 policy, petitioner paid two (2) installments although
effective notwithstanding the staggered payment of the premiums. The initial insurance contract entered it refused to pay the balance.
into in 1982 was renewed in 1983, then in 1984. In those three (3) years, the insurer accepted all the
installment payments. Such acceptance of payments speaks loudly of the insurer's intention to honor the It appearing from the peculiar circumstances that the parties actually intended to make three (3)
policies it issued to petitioner. Certainly, basic principles of equity and fairness would not allow the insurance contracts valid, effective and binding, petitioner may not be allowed to renege on its obligation
insurer to continue collecting and accepting the premiums, although paid on installments, and later deny to pay the balance of the premium after the expiration of the whole term of the third policy (No. AH-
liability on the lame excuse that the premiums were not prepared in full. CPP-9210651) in March 1985. Moreover, as correctly observed by the appellate court, where the risk is
entire and the contract is indivisible, the insured is not entitled to a refund of the premiums paid if the
We therefore sustain the Court of Appeals. We quote with approval the well-reasoned findings and insurer was exposed to the risk insured for any period, however brief or momentary.
conclusion of the appellate court contained in its Resolution denying the motion to reconsider its
Decision — WHEREFORE, finding no reversible error in the judgment appealed from, the same is AFFIRMED. Costs
against petitioner.
While the import of Section 77 is that prepayment of premiums is strictly required as a
condition to the validity of the contract, We are not prepared to rule that the request SO ORDERED.
to make installment payments duly approved by the insurer, would prevent the entire
contract of insurance from going into effect despite payment and acceptance of the 30. Artex Development Co. v. Wellington Ins. Co. (GRN L-29508, 27 June 1983)
initial premium or first installment. Section 78 of the Insurance Code in effect allows
waiver by the insurer of the condition of prepayment by making an acknowledgment TEEHANKEE, J.:

in the insurance policy of receipt of premium as conclusive evidence of payment so far


In this appeal from the decision of the court of first instance of Rizal at Caloocan city, the Court reiterates
as to make the policy binding despite the fact that premium is actually unpaid. Section
the establish doctrine that a third party not privy to a contract that contains no stipulations pour autrui in
77 merely precludes the parties from stipulating that the policy is valid even if
its favor may not sue enforcement of the contract.
premiums are not paid, but does not expressly prohibit an agreement granting credit
extension, and such an agreement is not contrary to morals, good customs, public
Hence, in this case where the lower court ordered defendant insurer to pay plaintiff-insured the balance
order or public policy (De Leon, the Insurance Code, at p. 175). So is an understanding
of the insured property loss of P3,624,683.43 and its ascertained business interruption loss of
to allow insured to pay premiums in installments not so proscribed. At the very least,
P1,748,460.00 with interest and attorney's fees, the Court affirms the correctness of the lower court's
both parties should be deemed in estoppel to question the arrangement they have
ruling that it is no defense for the insurer as against insured that the insurer had obtained reinsurance
voluntarily accepted. 4
from other companies to cover its liability.

Page 89 of 97
Defendant-appellant's lone assignment of error that lower court should have ruled instead "that plaintiff- MANIFESTATION
appellant cause of action (as insured) should have been directed against the reinsurers and not against
defendant-appellant" is manifestly untenable since there is no privity of contract between the insured Plaintiff-appellee, through counsel, respectfully manifests that, in view of the Deeds of
and the reinsurers. Plaintiff-appellee insured can only move for enforcement of its insurance contract Discharge dated 10 April 1969 and Collateral Agreement dated 10 April 1969, hereto
with its insurer, the defendant-appellant. attached as Annexes "A" and "B", the only remaining liability subject of litigation shall
be that proportion of the loss reinsured with or through Alexander and Alexander, Inc.
Unless there is a specific grant in, or assignment of, reinsurance contract in favor of the insured or a of New York, U.S.A., namely, P397,813.00 — the rest having been paid and settled per
manifest intention of the contracting parties to the insurance contrary to grant such benefit or favor to the said deeds Annexes "A" and "B".
the insured, not being privy to the reinsurance contract, has no cause of action against the reinsurer. It is
expressly provided in section 91 the Insurance Act1 that "(T)he original insured has no interest in a Quezon City for Manila, 10 April 1969.
contract of insurance."
(Signed) NORBERTO J. QUISUMBING Counsel for Plaintiff-Appellant P.O. Box No. 226,
The lower court's judgment of April 2, 1968 was rendered the basis of the parties' stipulation of facts and Manila.
there is dispute as to the property and business interruption loss of the insured as thus determined nor
as to the partial payment made by defendant-insurer that have greatly reduced the amount still due and CONFORME:

owing under the judgment under appeal.


ARTEX DEVELOPMENT CO., INC.

Briefly, the trial court found that from the evidence and stipulation of facts presented, it appears that the
By:
defendant, Wellington Insurance Co., Inc. insured for P24,346,509.00 the buildings, stocks and machinery
(Signed) DOMINGO G. CASTILLO
of plaintiff Artex Development Co., Inc., against loss or damage by fire or lighting (Exh. A) upon payment
President3
by plaintiff of the corresponding premiums; that on August 2, 1963, said properties were insured for an
additional sum of P833,034.00 (Exh. A-1) that on May 12, 1963 defendant insured plaintiff against business
The amended documents recited further that:
interruption (use and occupancy) for P5,200,000.00 (Exh. B); that on September 22, 1963, the buildings,
stocks and machineries of plaintiff's spinning department were burned; that notice of the loss and
1. Artex hereby acknowledges receipt of the sum of P3,600,000.00 in Philippine
damage was given the defendant, and the loss was referred to the H. H. Bayne Adjustment Co. and the
currency paid by Minet on behalf itself and Willington and Minet & Co. in full and final
Allied Adjustment Co.; that as per report of the adjusters, the total property loss of the plaintiff was the
settlement of all any claims Artex may have against Willington, Minet and Minet Co. in
sum of P10,106,554.40 and the total business interruption loss was P3,000,000.00; that defendant has
respect of the losses resulting from the said fire of 22nd September 1963 the Policies
paid to the plaintiff the sum of P6,481,870.07 of the property loss suffered by plaintiff and P1,864,134.08
of Insurance and the Contracts Reinsurance specified in the said Deeds of Discharge
on its business interruption loss, leaving a balance of P3,624,683.43 and P1,748,460.00, respectively."2
and discharge Willington, Minet and Minet & Co. jointly and severally from all actions,
proceedings, claims, demands, costs and expenses in respect thereof including the said
On May 29, 1969, counsel for plaintiff-appellee filed a manifestation dated April 10, 1969, bearing the
judgment obtained in the Court of First Instance of Rizal and additionally Artex waives
conformity of plaintiff itself under the signature of its president, Domingo G. Castillo, as follows:

Page 90 of 97
in favor of Minet and Minet & Co. Artex's right of recourse against them under Article Article 1311 of our Civil Code expresses the universal rule that "Contracts take effect only between the
1177 of the Civil Code of the Philippines.4 parties, their assigns and heirs" (with the heir being "not liable beyond the value of the property he
received from the decedent,") and provides for the exception of stipulations pour autrui or in favor of a
Upon the parties' joint motion dated May 22, 1969 for temporary suspension of the proceedings by third person not a party to the contract, in this wise:
virtue of such payment, the Court per its resolution of June 30, 1969 resolve to suspend the proceedings
until July 30, 1969.5 The Court also noted defendant-appellant's manifestation dated June 18, 1969, to the If a contract should contain some stipulation in favor of a third person, he may
effect that "the statement in plaintiff-appellee' Manifestation that the only remaining amount of its demand its fulfillment provided he communicated his acceptance to the obligor before
claimant subject of litigation is the proportion of the loss reinsured wit Alexander and Alexander, Inc. of its revocation. A mere incidental benefit or interest of a person is not sufficient. The
New York, U.S.A. in the amount of P397,813.00 because the reinsurers of defendant-appellant made contracting parties must have clearly and deliberately conferred favor upon a third
additional partial payments, is true and correct but without prejudice to the legal question presented in person. (Art. 1311, Civil Code, second paragraph)
defendant-appellant's brief."6
The Court has a since the early case of Uy Tam vs. Leonard8 that the "intent of the contracting parties to
Thereafter, plaintiff-appellee filed on August 8, 1969 its brief, and prayed for affirmance of the appealed benefit third party by means of such stipulations pour autrui must clearly expressed, and hence, a clause
judgment with modification, as follows: in a contractor's executed solely in favor of the City of Manila and condition pay for all labor and
materials cannot be construed stipulation pour autrui available to materialmen who supplied certain
In the light of the foregoing discussion, the lower court did not commit any error in its materials to the contractor for use in the performance of the latter's contract with the city.
appealed decision, which must accordingly be sustained and affirmed. It is however
respectfully prayed that the same be modified as to the amount of liability adjudged In Bonifacio Bros, Inc. vs. Mora9 the Court reiterated same established doctrine, holding that the clause in
against defendant appellant in favor of plaintiff-appellee, in accordance with their a motor vehicle insurance policy authorizing the owner of damaged vehicle to contract for its repair does
Collateral Agreement executed by them on April 10, 1969 (Annex "B", of manifestation not mean that the repairman may collect the cost of the repair directly the insurer, there being no clause
of the same date, filed in this Court on 29 May 1969), which should now be fixed at "from which we can infer that there is an obligation on the part of the insurance company to pay the cost
P397,813.00, plus of course 12% interest per annum thereof for late payment until 10 of repairs directly to them,' and that the mortgagee of the car (expressly named in the insure policy as
April 1969, attorney's fees of 15% of the recovery, expenses of litigation and costs of beneficiary of any loss payable thereunder) had better right than the repairman to the insurance
suit, already adjudged by the lower court, no writ of execution to issue however on any proceeds.
adjudged liability until after three (3) years from 10 April 1969, pursuant to the same
'Collateral Agreement of the parties. Plaintiff-insured, not being a party or privy to defendant insurer's reinsurance contracts, therefore, could
not directly demand enforcement of such insurance contracts. Defendant-appellant's contention that the
On the sole issue of law raised by defendant-appellant in its brief, the Court finds, as above indicated, insured should be deemed have agreed to look solely to the reinsurers for indemnity case of loss, since it
that no single clause in the reinsurance contracts has been cited by defendant-insurer that would justify was evident that with its mere P500,000. paid-up capital stock, it had to secure reinsurance coverage the
its claim that they contained a stipulation pour autrui in favor of plaintiff-insured, and whereby "plaintiff- over P24-million fire insurance coverage of the policy issued by it to plaintiff-insured, is manifestly
appellee is deemed to have agreed to look solely to the reinsurers for indemnity in case of loss."7 untenable.

Page 91 of 97
Assuming that plaintiff-insured could avail of the reinsurance contracts and directly sue the reinsurers for sufficient to form a belief as to the truth veracity of Plaintiff Appellee's imputation to Defendant-
payment of the loss, still such assumption would not in any way affect or cancel out defendant-insurer's Appellant that the latter had filed the above-entitled a only for delay." 11
direct contractual liability to plaintiff-insured under the insurance policy to indemnify plaintiff for the
property losses. Plaintiff's right as insured to sue defendant as insurer directly and solely would thereby Since the claim at bar of plaintiff against defendant merely for the balance of a proven undisputed claim
not be affected or curtailed in any way, without prejudice to defendant in turn filing a third party (as amount) — long tried and decided as per the trial court judgment of April 2, 1968 before the
complaint or separate suit against its reinsurers: Thus, in Naga Development Corp. vs. Court of liquidation order issued only last year on September 18, 1972 — the Court has herein resolved and
Appeals 10 the Court held that the contractor remain liable to the supplier for materials delivered, disposed of the sole issue of law raised in the appeal. Plaintiff's judgment claim as now judiciary
notwithstanding arrangements made on its GSIS loan for the GSIS to issue treasury warrants on account determined will have to be satisfied in compliance with requirements of the Insurance Act governing
of such loan, directly in favor of the supplier, since "such an arrangement obviously cannot destroy or distribution assets, priorities of payments of proven claims, etc., insurance companies under liquidation
modify the direct legal responsibility of the (contractor) to the (supplier) to pay for what the latter gave and with prior authorization of the court in the liquidation proceeding pending in the Manila court of first
and rendered to the former." instance.

On April 4, 1973, plaintiff-appellee filed a manifestation informing the Court that in Republic of the ACCORDINGLY, as prayed for by plaintiff-appellee in brief, the judgment of the lower court is affirmed,
Philippines vs. Wellington Insurance Co., Inc., docketed as Civil Case No. 88046 of the court of first with the modification that the remaining liability of defendant appellant to plaintiff-appellee in
instance of Manila, an order was issued on September 18, 1972 for the 'Liquidation of said insurance accordance with the "collateral agreement" of April 10, 1969 is fixed at P397,813.00 with twelve (12%)
company, herein defendant-appellant; that the Insurance Commissioner was designated receiver and as percent interest per annum until 10 April 1969, attorney's fees of fifteen (15%) percent of the recovery,
such issued on November 4, 1972 an order for the filing of claims against said defendant; that and cost of suit.
accordingly plaintiff filed its verified statement of claim wherein it asked the Insurance Commissioner "to
move to dismiss the above-entitled appeal as filed only for delay." 31. Avon Insurance PLC v CA (278 SCRA 312, GRN 97642, 29 Aug. 1997)

Requested by the Court to file their comments, defendant through counsel admitted the fact of
TORRES, JR., J.:
liquidation proceedings but denied any dilatory motive in its appeal, stating that "although it does not
raise any issue of fact in (this) appeal, yet the question of law raised (herein) is of first impression in this
Just how far can our courts assert jurisdiction over the persons of foreign entities being charged with
jurisdiction" and of "utmost importance" to insurance companies taking out reinsurance policies.
contractual liabilities by residents of the Philippines?

The Insurance Commissioner, in her manifestation of May 18, 1973, confirmed the fact of her taking over
"title to all property, contracts, rights of action and all of the records of the (defendant) insurance Appealing from the Court of Appeals’ October 11, 1990 Decision 1 in CA-G.R. No. 22005, petitioners claim

company" as liquidator pursuant to section 175-B of the Insurance claiming the sole right-now to officially that the trial court’s jurisdiction does not extend to them, since they are foreign reinsurance companies

represent and act for defendant company and asserting "exclusive jurisdiction determine this claim" even that are not doing business in the Philippines. Having entered into reinsurance contracts abroad,

as against this Court according to her, should be deemed to have "ceased to jurisdiction over the subject petitioners are beyond the jurisdictional ambit of our courts and cannot be served summons through

of this pending action," but at the same time not moving to dismiss the appeal, as suggest plaintiff, and extraterritorial service, as under Section 17, Rule 14 of the Rules of Court, nor through the Insurance

instead manifesting that "the Insurance Commissioner is absolutely without any knowledge information Commissioner, under Section 14. Private respondent Yupangco Cotton Mills contends on the other hand

Page 92 of 97
that petitioners are within our courts’ cognitive powers, having submitted voluntarily to their jurisdiction As fate would have it, on December 16, 1979 and May 2, 1981, within the respective effectivity periods of
by filing motions to dismiss 2 the private respondent’s suit below. Policies 20719 and 25896, the properties therein insured were razed by fire, thereby giving rise to the
obligation of the insurer to indemnify the Yupangco Cotton Mills. Partial payments were made by
The antecedent facts, as found by the appellate court, are as follows: Worldwide Surety and Insurance and some of the reinsurance companies.

"Respondent Yupangco Cotton Mills filed a complaint against several foreign reinsurance companies On May 2, 1983, Worldwide Surety and Insurance, in a Deed of Assignment, acknowledged a remaining
(among which are petitioners) to collect their alleged percentage liability under contract treaties between balance of P19,444,447.75 still due Yupangco Cotton Mills, and assigned to the latter all reinsurance
the foreign insurance companies and the international insurance broker C.J. Boatright, acting as agent for proceeds still collectible from all the foreign reinsurance companies. Thus, in its interest as assignee and
respondent Worldwide Surety and Insurance Company. Inasmuch as petitioners are not engaged in original insured, Yupangco Cotton Mills instituted this collection suit against the petitioners.
business in the Philippines with no offices, places of business or agents in the Philippines, the reinsurance
treaties having been entered abroad, service of summons upon motion of respondent Yupangco, was Service of summons upon the petitioners was made by notification to the Insurance Commissioner,
made upon petitioners through the Office of the Insurance Commissioner. Petitioners, by counsel on pursuant to Section 14, Rule 14 of the Rules of Court. 5
special appearance, seasonably filed motions to dismiss disputing the jurisdiction of respondent Court
and the extra-territorial service of summons. Respondent Yupangco filed its opposition to the motions to In a Petition for Certiorari filed with the Court of Appeals, petitioners submitted that respondent Court
dismiss, petitioners filed their reply, and respondent Yupangco filed its rejoinder. In an Order dated April has no jurisdiction over them, being all foreign corporations not doing business in the Philippines with no
30, 1990, respondent Court denied the motions to dismiss and directed petitioners to file their answer. office, place of business or agents in the Philippines. The remedy of Certiorari was resorted to by the
On May 29, 1990, petitioners filed their notice of appeal. In an order dated June 4, 1990, respondent petitioners on the premise that if petitioners had filed an answer to the complaint as ordered by the
court denied due course to the appeal." 3 respondent court, they would risk abandoning the issue of jurisdiction. Moreover, extra-territorial service
of summons on petitioners is null and void because the complaint for collection is not one affecting
To this day, trial on the merits of the collection suit has not proceeded as in the present petition, plaintiff’s status and not relating to property within the Philippines.
petitioners continue vigorously to dispute the trial court’s assumption of jurisdiction over them.
The Court of Appeals found the petition devoid of merit, stating that:chanrob1es virtual 1aw library
It will be remembered that in the plaintiff’s complaint, 4 it was contended that on July 6, 1979 and on
October 1, 1980, Yupangco Cotton Mills engaged to secure with Worldwide Security and Insurance Co. 1. Petitioners were properly served with summons and whatever defect, if any, in the service of summons
Inc., several of its properties for the periods July 6, 1979 to July 6, 1980 as under Policy No. 20719 for a were cured by their voluntary appearance in court, via motion to dismiss.
coverage of P100,000,000.00 and from October 1, 1980 to October 1, 1981, under Policy No. 25896, also
for P100,000,000.00. Both contracts were covered by reinsurance treaties between Worldwide Surety and 2. Even assuming that petitioners have not yet voluntarily appeared as co-defendants in the case below
Insurance and several foreign reinsurance companies, including the petitioners. The reinsurance even after having filed the motions to dismiss adverted to, still the situation does not deserve dismissal of
arrangements had been made through international broker C.J. Boatwright and Co. Ltd., acting as agent the complaint as far as they are concerned, since as held by this Court in Lingner Fisher GMBH v. IAC, 125
of Worldwide Surety and Insurance. SCRA 523;

Page 93 of 97
"A case should not be dismissed simply because an original summons was wrongfully served. It should For its part, private respondent Yupangco counter-submits:
be difficult to conceive for example, that when a defendant personally appears before a court
complaining that he had not been validly summoned, that the case filed against him should be "1. Foreign corporations, such as petitioners, not doing business in the Philippines, can be sued in
dismissed. An alias summons can be actually served on said defendant." Philippine Courts, not withstanding petitioners’ claim to the contrary.

3. Being reinsurers of respondent Worldwide Surety and Insurance of the risk which the latter assumed 2. While the complaint before the Honorable Trial Court is for a sum of money, not affecting status or
when it issued the fire insurance policies in dispute in favor of respondent Yupangco, petitioners cannot relating to property, petitioners (then defendants) can submit themselves voluntarily to the jurisdiction of
now validly argue that they do not do business in this country. At the very least, petitioners must be Philippine Courts, even if there is no extra-judicial (sic) service of summons upon them.
deemed to have engaged in business in the Philippines no matter how isolated or singular such business
might be, even on the assumption that among the local domestic insurance corporations of this country, 3. The voluntary appearance of the petitioners (then defendants) before the Honorable Trial Court
it is only in favor of Worldwide Surety and Insurance that they have ever reinsured any risk arising from amounted, in effect, to voluntary submission to its jurisdiction over their persons." 7
any reinsurance within the territory.
In the decisions of the courts below, there is much left to speculation and conjecture as to whether or not
4. The issue of whether or not petitioners are doing business in the country is a matter best referred to a the petitioners were determined to be "doing business in the Philippines" or not.
trial on the merits of the case, and so should be addressed there.
To qualify the petitioners’ business of reinsurance within the Philippine forum, resort must be made to
Maintaining its submission that they are beyond the jurisdiction of Philippine Courts, petitioners are now the established principles in determining what is meant by "doing business in the Philippines." In
before us, stating: Communication Materials and Design, Inc. et. al. v. Court of Appeals, 8 it was observed that:

"Petitioners, being foreign corporations, as found by the trial court, not doing business in the Philippines "There is no exact rule or governing principle as to what constitutes doing or engaging in or transacting
with no office, place of business or agents in the Philippines, are not subject to the jurisdiction of business. Indeed, such case must be judged in the light of its peculiar circumstances, upon its peculiar
Philippine courts. facts and upon the language of the statute applicable. The true test, however, seems to be whether the
foreign corporation is continuing the body or substance of the business or enterprise for which it was
The complaint for sum of money being a personal action not affecting status or relating to property, organized.
extraterritorial service of summons on petitioners — all not doing business in the Philippines — is null
and void. Article 44 of the Omnibus Investments Code of 1987 defines the phrase to include:chanrob1es virtual 1aw
library
The appearance of counsel for petitioners being explicitly ‘by special appearance without waiving
objections to the jurisdiction over their persons or the subject matter’ and the motions to dismiss having ‘soliciting orders, purchases, service contracts, opening offices, whether called ‘liaison’ offices or
excluded non-jurisdictional grounds, there is no voluntary submission to the jurisdiction of the trial branches; appointing representatives or distributors who are domiciled in the Philippines or who in any
court." 6 calendar year stay in the Philippines for a period or periods totaling one hundred eighty (180) days or
more; participating in the management, supervision or control of any domestic business firm, entity or

Page 94 of 97
corporation in the Philippines, and any other act or acts that imply a continuity or commercial dealings or remotely connected with the Philippines. Moreover, there is authority to the effect that a reinsurance
arrangements and contemplate to that extent the performance of acts or works, or the exercise of some company is not doing business in a certain state merely because the property or lives which are insured
of the functions normally incident to, and in progressive prosecution of, commercial gain or of the by the original insurer company are located in that state. 12 The reason for this is that a contract of
purpose and object of the business organization.’" reinsurance is generally a separate and distinct arrangement from the original contract of insurance,
whose contracted risk is insured in the reinsurance agreement. 13 Hence, the original insured has
The term ordinarily implies a continuity of commercial dealings and arrangements, and contemplates, to generally no interest in the contract of reinsurance. 14
that extent, the performance of acts or works or the exercise of the functions normally incident to and in
progressive prosecution of the purpose and object of its organization. 9 A foreign corporation, is one which owes its existence to the laws of another state, 15 and generally, has
no legal existence within the state in which it is foreign. In Marshall Wells Co. v. Elser, 16 it was held that
A single act or transaction made in the Philippines, however, could qualify a foreign corporation to be corporations have no legal status beyond the bounds of the sovereignty by which they are created.
doing business in the Philippines, if such singular act is not merely incidental or casual, but indicates the Nevertheless, it is widely accepted that foreign corporations are, by reason of state comity, allowed to
foreign corporation’s intention to do business in the Philippines. 10 transact business in other states and to sue in the courts of such fora. In the Philippines foreign
corporations are allowed such privileges, subject to certain restrictions, arising from the state’s sovereign
There is no sufficient basis in the records which would merit the institution of this collection suit in the right of regulation.
Philippines. More specifically, there is nothing to substantiate the private respondent’s submission that
the petitioners had engaged in business activities in this country. This is not an instance where the Before a foreign corporation can transact business in the country, it must first obtain a license to transact
erroneous service of summons upon the defendant can be cured by the issuance and service of alias business here 17 and secure the proper authorizations under existing law.
summons, as in the absence of showing that petitioners had been doing business in the country, they
cannot be summoned to answer for the charges leveled against them. If a foreign corporation engages in business activities without the necessary requirements, it opens itself
to court actions against it, but it shall not be allowed to maintain or intervene in an action, suit or
The Court is cognizant of the doctrine in Signetics Corp. v. Court of Appeals 11 that for the purpose of proceeding for its own account in any court or tribunal or agency in the Philippines. 18
acquiring jurisdiction by way of summons on a defendant foreign corporation, there is no need to prove
first the fact that defendant is doing business in the Philippines. The plaintiff only has to allege in the The purpose of the law in requiring that foreign corporations doing business in the country be licensed
complaint that the defendant has an agent in the Philippines for summons to be validly served thereto, to do so, is to subject the foreign corporations doing business in the Philippines to the jurisdiction of the
even without prior evidence advancing such factual allegation. courts, 19 otherwise, a foreign corporation illegally doing business here because of its refusal or neglect
to obtain the required license and authority to do business may successfully though unfairly plead such
As it is, private respondent has made no allegation or demonstration of the existence of petitioners’ neglect or illegal act so as to avoid service and thereby impugn the jurisdiction of the local courts.
domestic agent, but avers simply that they are doing business not only abroad but in the Philippines as
well. It does not appear at all that the petitioners had performed any act which would give the general The same danger does not exist among foreign corporations that are indubitably not doing business in
public the impression that it had been engaging, or intends to engage in its ordinary and usual business the Philippines. Indeed, if a foreign corporation does not do business here, there would be no reason for
undertakings in the country. The reinsurance treaties between the petitioners and Worldwide Surety and it to be subject to the State’s regulation. As we observed, in so far as the State is concerned, such foreign
Insurance were made through an international insurance broker, and not through any entity or means

Page 95 of 97
corporation has no legal existence. Therefore, to subject such corporation to the courts’ jurisdiction
would violate the essence of sovereignty. In civil cases, jurisdiction over the person of the defendant is acquired either by his voluntary appearance
in court and his submission to its authority or by service of summons. 22
In the alternative, private respondent submits that foreign corporations not doing business in the
Philippines are not exempt from suits leveled against them in courts, citing the case of Facilities Fundamentally, the service of summons is intended to give official notice to the defendant or respondent
Management Corporation v. Leonardo Dela Osa, et. al. 20 where we ruled "that indeed, if a foreign that an action has been commenced against it. The defendant or respondent is thus put on guard as to
corporation, not engaged in business in the Philippines, is not barred from seeking redress from Courts the demands of the plaintiff as stated in the complaint. 23 The service of summons upon the defendant
in the Philippines, a fortiori, that same corporation cannot claim exemption from being sued in Philippine becomes an important element in the operation of a court’s jurisdiction upon a party to a suit, as service
Courts for acts done against a person or persons in the Philippines." of summons upon the defendant is the means by which the court acquires jurisdiction over his person.
24 Without service of summons, or when summons are improperly made, both the trial and the
We are not persuaded by the position taken by the private Respondent. In Facilities Management case, judgment, being in violation of due process, are null and void, 25 unless the defendant waives the service
the principal issue presented was whether the petitioner had been doing business in the Philippines, so of summons by voluntarily appearing and answering the suit. 26
that service of summons upon its agent as under Section 14, Rule 14 of the Rules of Court can be made
in order that the Court of First Instance could assume jurisdiction over it. The Court ruled that the When a defendant voluntarily appears, he is deemed to have submitted himself to the jurisdiction of the
petitioner was doing business in the Philippines, and that by serving summons upon its resident agent, court. 27 This is not, however, always the case. Admittedly, and without subjecting himself to the court’s
the trial court had effectively acquired jurisdiction. In that case, the court made no prescription as the jurisdiction, the defendant in an action can, by special appearance object to the court’s assumption on
absolute suability of foreign corporations not doing business in the country, but merely discounts the the ground of lack of jurisdiction. If he so wishes to assert this defense, he must do so seasonably by
absolute exemption of such foreign corporations from liabilities particularly arising from acts done motion for the purpose of objecting to the jurisdiction of the court, otherwise, he shall be deemed to
against a person or persons in the Philippines. have submitted himself to that jurisdiction. 28 In the case of foreign corporations, it has been held that
As we have found, there is no showing that petitioners had performed any act in the country that would they may seek relief against the wrongful assumption of jurisdiction by local courts. In Time, Inc. v. Reyes,
place it within the sphere of the court’s jurisdiction. A general allegation standing alone, that a party is 29 it was held that the action of a court in refusing to rule or deferring its ruling on a motion to dismiss
doing business in the Philippines does not make it so. A conclusion of fact or law cannot be derived from for lack or excess of jurisdiction is correctable by a writ of prohibition or certiorari sued out in the
the unsubstantiated assertions of parties, notwithstanding the demands of convenience or dispatch in appellate court even before trial on the merits is had. The same remedy is available should the motion to
legal actions, otherwise, the Court would be guilty of sorcery; extracting substance out of nothingness. In dismiss be denied, and the court, over the foreign corporation’s objections, threatens to impose its
addition, the assertion that a resident of the Philippines will be inconvenienced by an out-of-town suit jurisdiction upon the same.
against a foreign entity, is irrelevant and unavailing to sustain the continuance of a local action, for
jurisdiction is not dependent upon the convenience or inconvenience of a party. 21 If the defendant, besides setting up in a motion to dismiss his objection to the jurisdiction of the court,
alleges at the same time any other ground for dismissing the action, or seeks an affirmative relief in the
It is also argued that having filed a motion to dismiss in the proceedings before the trial court, petitioners motion, 30 he is deemed to have submitted himself to the jurisdiction of the court.
have thus acquiesced to the court’s jurisdiction, and they cannot maintain the contrary at this juncture.

This argument is at the most, flimsy.

Page 96 of 97
In this instance, however, the petitioners from the time they filed their motions to dismiss, their
submissions have been consistently and unfailingly to object to the trial court’s assumption of jurisdiction,
anchored on the fact that they are all foreign corporations not doing business in the Philippines.

As we have consistently held, if the appearance of a party in a suit is precisely to question the jurisdiction
of the said tribunal over the person of the defendant, then this appearance is not equivalent to service of
summons, nor does it constitute an acquiescence to the court’s jurisdiction. 31 Thus, it cannot be argued
that the petitioners had abandoned their objections to the jurisdiction of the court, as their motions to
dismiss in the trial court, and all their subsequent posturings, were all in protest of the private
respondent’s insistence on holding them to answer a charge in a forum where they believe they are not
subject to. Clearly, to continue the proceedings in a case such as those before Us would just "be useless
and a waste of time." 32

ACCORDINGLY, the decision appealed from dated October 11, 1990, is SET ASIDE and the instant petition
is hereby GRANTED. The respondent Regional Trial Court of Manila, Branch 51 is declared without
jurisdiction to take cognizance of Civil Case No. 86-37932, and all its orders and issuances in connection
therewith are hereby ANNULLED and SET ASIDE. The respondent court is hereby ORDERED to DESIST
from maintaining further proceeding in the case aforestated.

SO ORDERED.

Page 97 of 97

You might also like